Barbri Civ Pro Outline

Download as docx, pdf, or txt
Download as docx, pdf, or txt
You are on page 1of 155

CIVIL PROCEDURE FEDERAL

CIVIL PROCEDURE-FEDERAL 1.

CIVIL PROCEDURE-FEDERAL
TABLE OF CONTENTS
INTRODUCTION....................................................................................................................................... 1
A. PERSONAL JURISDICTION.........................................................................................1
SUBJECT MATTER JURISDICTION..........................................................................1
1. Diversity of Citizenship Jurisdiction.............................................................................1
2. Federal Question Jurisdiction..................................................................................1
3. Removal Jurisdiction......................................................................................................1
4. Supplemental Jurisdiction.............................................................................................1
C. VENUE...............................................................................................................................1
D. DISCOVERY.....................................................................................................................2
E. MULTIPLE PARTIES.....................................................................................................2
I. PERSONAL JURISDICTION.................................................................................................3
A. OVERVIEW......................................................................................................................3
1. Limitations on Personal Jurisdiction.......................................................................3
a. Statutory Limitations........................................................................................3
b. Constitutional Limitations................................................................................3
c. Personal Jurisdiction in Federal Courts.........................................................3
2. Three Types of Personal Jurisdiction......................................................................3
a. In Personam Jurisdiction..................................................................................3
b. In Rem Jurisdiction...........................................................................................4
c. Quasi In Rem Jurisdiction................................................................................4
1) Defendant Is Not Bound Personally.........................................................4
STATUTORY LIMITATIONS ON IN PERSONAM JURISDICTION.....................4
1. Physical Presence at Time of Personal Service......................................................4
2. State Law Exceptions to Traditional Rule...............................................................5
a. Service by Fraud or Force Invalid...................................................................5
b. Immunity of Parties and Witnesses.................................................................5
3. Domicile......................................................................................................................5
a. Defined....................................................................................................................5
b. Citizenship..........................................................................................................5
4. Consent.......................................................................................................................5
a. Express Consent.................................................................................................5
1) By Contract................................................................................................5
2) By Appointment of Agent to Accept Service of Process..............................5
Appointment Required by State...........................................................6
b. Implied Consent.................................................................................................6
c. Voluntary Appearance......................................................................................6
5. Long Arm Statutes....................................................................................................6
a. Unlimited Long Arm Statutes..........................................................................6
b. Limited (or Specific) Long Arm Statutes........................................................6
1) Limitations in Tort Cases..........................................................................6
2) Limitations in Contract Cases..................................................................7
Limitations in Property Actions...............................................................7
11. CIVIL PROCEDURE-FEDERAL
4) Limitations in Marital Dissolution Cases.............................................................7
C. CONSTITUTIONAL LIMITATIONS ON IN PERSONAM JURISDICTION........................7
1. Sufficient Contacts with the Forum...............................................................................7
a. Traditional Rule: Physical Power..........................................................................7
b. Modern Due Process Standard: Contact, Relatedness, and Fairness........................7
1) Contact........................................................................................................8
a) Purposeful Availment....................................................................................8
(1) "Stream of Commerce" Cases......................................................9
(2) Internet Cases................................................................................ 9
b) Foreseeability .............. 10
2) Relatedness of Claim to Contact ....... 10
a) Claim Arising from Activity in the State (Specific
Jurisdiction) ..................... 10
b) "At Home" in the State (General Jurisdiction) .. 11
3) Fairness ........... 11
a) Convenience ...... 11
b) Forum State's Interest . 11
c) Other Factors...................................................................................12
2. Notice............................................................................................................................. 12
a. Traditional Methods of Personal Service Satisfy Due Process Notice
Requirements........................................................................................................12
b. Requirement that Agent Notify Defendant.................................................................12
c. Requirements for Cases Involving Multiple Parties or Unknown
Parties .......................... 13
d. Knowledge that Notice by Mail Was Not Received .. 13
D. IN REM JURISDICTION ..... . 13
1. Statutory Limitations ..................... . 13
2. Constitutional Limitations .................. . 13
a. Nexus ........................... 13
1) No Jurisdiction If Property Not Located in State 14
2) No Jurisdiction If Property Brought in by Fraud or Force .. 14
b. Notice ........... 14
QUASI IN REM JURISDICTION . 14
1. Statutory Limitations ..... . 14
2. Constitutional Limitations .. . 14
a. Nexus ...........
Quasi In Rem Type I 14
14
2) Quasi In Rem Type II ...................... 15
Procedural Requirements .... 15

II. b. Notice .................. 15


DIVERSITY OF CITIZENSHIP JURISDICTION . A. 15
INTRODUCTION ................ . 15
B. DIVERSITY AMONG THE PARTIES .....
. 15
1. Complete Diversity When Action Is Commenced ...
a. Multiple Parties-Complete Diversity 1) . 15
But Note ........... 16
lnterpleader Exception ...... 16
16
CIVIL PROCEDURE-FEDERAL 111.

Contents
CIVIL PROCEDURE-FEDERAL TABLE OF CONTENTS.............................................................2
INTRODUCTION 1..............................................................................................................2
A. PERSONAL JURISDICTION 1...................................................................................2
SUBJECT MATTER JURISDICTION 1.........................................................................2
1. Diversity of Citizenship Jurisdiction 1............................................................................2
2. Federal Question Jurisdiction 1....................................................................................2
3. Removal Jurisdiction 1....................................................................................................2
4. Supplemental Jurisdiction 1............................................................................................2
C. VENUE 1.........................................................................................................................2
D. DISCOVERY 2...............................................................................................................2
E. MULTIPLE PARTIES 2...............................................................................................2
I. PERSONAL JURISDICTION 3...................................................................................2
A. OVERVIEW 3................................................................................................................2
1. Limitations on Personal Jurisdiction 3........................................................................2
a. Statutory Limitations 3..................................................................................................2
b. Constitutional Limitations 3.........................................................................................2
c. Personal Jurisdiction in Federal Courts 3...................................................................2
2. Three Types of Personal Jurisdiction 3.......................................................................2
a. In Personam Jurisdiction 3...........................................................................................2
b. In Rem Jurisdiction 4....................................................................................................2
c. Quasi In Rem Jurisdiction 4..........................................................................................2
1) Defendant Is Not Bound Personally 4..........................................................................2
STATUTORY LIMITATIONS ON IN PERSONAM JURISDICTION 4......................2
1. Physical Presence at Time of Personal Service 4........................................................2
2. State Law Exceptions to Traditional Rule 5................................................................2
a. Service by Fraud or Force Invalid 5.............................................................................2
b. Immunity of Parties and Witnesses 5...........................................................................2
3. Domicile 5........................................................................................................................2
a. Defined 5..........................................................................................................................2
b. Citizenship 5....................................................................................................................2
4. Consent 5.........................................................................................................................2
a. Express Consent 5..........................................................................................................2
1) By Contract 5..................................................................................................................2
2) By Appointment of Agent to Accept Service of Process 5...............................................2
Appointment Required by State 6........................................................................................2
b. Implied Consent 6..........................................................................................................2
c. Voluntary Appearance 6................................................................................................2
5. Long Arm Statutes 6......................................................................................................2
a. Unlimited Long Arm Statutes 6....................................................................................2
b. Limited (or Specific) Long Arm Statutes 6.................................................................2
1) Limitations in Tort Cases 6...........................................................................................2
2) Limitations in Contract Cases 7...................................................................................2
Limitations in Property Actions 7......................................................................................2
4) Limitations in Marital Dissolution Cases 7........................................................................3
C. CONSTITUTIONAL LIMITATIONS ON IN PERSONAM JURISDICTION 7...............3
1. Sufficient Contacts with the Forum 7..............................................................................3
a. Traditional Rule: Physical Power 7.................................................................................3
b. Modern Due Process Standard: Contact, Relatedness, and Fairness 7...............................3
1) Contact 8.........................................................................................................................3
a) Purposeful Availment 8.....................................................................................................3
(1) "Stream of Commerce" Cases 9......................................................................................3
(2) Internet Cases 9...............................................................................................................3
b) Foreseeability .............. 10.......................................................................3
2) Relatedness of Claim to Contact ....... 10........................................................3
a) Claim Arising from Activity in the State (Specific.....................................................3
Jurisdiction) ..................... 10........................................................3
b) "At Home" in the State (General Jurisdiction) .. 11.............................................3
3) Fairness ........... 11......................................................................................3
a) Convenience ...... 11............................................................................................3
b) Forum State's Interest . 11........................................................................................3
c) Other Factors 12.............................................................................................................3
2. Notice 12..........................................................................................................................3
a. Traditional Methods of Personal Service Satisfy Due Process Notice Requirements 12
3
b. Requirement that Agent Notify Defendant 12....................................................................3
c. Requirements for Cases Involving Multiple Parties or Unknown............................3
13 3
2) Quasi In Rem Type II .............................................................3
G. PROCEDURE FOR REMOVAL 39.............................................................................20
1. Notice of Removal 39.....................................................................................................20
a. Allegation of Amount in Controversy 39.........................................................................20
2. Thirty Day Rule 39.......................................................................................................20
3. Procedure After Removal 40.........................................................................................20
4. Right to Jury Trial 40...................................................................................................20
a. Demand for Jury Trial 40...........................................................................................20
b. Demand Not Required 40................................................................................................20
5. Remand 40....................................................................................................................20
a. Subject Matter Jurisdiction Generally Considered First 41.............................................20
H. SPECIFIC TYPES OF ACTIONS 41........................................................................20
1. Removable 41.................................................................................................................20
2. Nonremovable 41............................................................................................................20
a. All Writs Act Not a Basis for Removal 41....................................................................20
III. CONFLICT OF JURISDICTION BETWEEN STATE AND FEDERAL
COURTS 41...................................................................................................................20
A. FULL FAITH AND CREDIT EXTENDED TO FEDERAL COURTS 41..................20
B. INJUNCTIONS AGAINST PENDING STATE PROCEEDINGS 42..............................20
C. INJUNCTIONS AGAINST THREATENED STATE CRIMINAL
PROSECUTIONS 42.....................................................................................................20
D. INJUNCTIONS AGAINST STATE TAX PROCEDURES 42.....................................20
IV. THE FEDERAL RULES OF CIVIL PROCEDURE 42..................................20
A. COMMENCEMENT OF THE ACTION 42.................................................................20
B. SERVICE OF PROCESS 42.......................................................................................20
1. Who May Serve 42.......................................................................................................20
2. How Service Is Made 42................................................................................................20
a. Service Under State Rules 43........................................................................................20
b. Waiver of Service (Service by Mail) 43..........................................................................20
1) Effect of Waiver 43........................................................................................................20
2) Effect of Failure to Waive 43.........................................................................................20
3. Parties Served Outside State 43.....................................................................................20
4. Parties Served in Foreign Country 44...........................................................................20
5. Immunity from Process 44.............................................................................................20
C. TIME PERIODS 44.....................................................................................................20
1. Counting Time 44..........................................................................................................20
2. Extensions of Time 44....................................................................................................20
D. INTERLOCUTORY INJUNCTIONS 44......................................................................20
1. Preliminary Injunction 45...............................................................................................20
a. Requirements for a Preliminary Injunction 45................................................................20
2. Temporary Restraining Order 45.................................................................................20
a. Requirements for Ex Parte Temporary Restraining Orders 45..................................20
1) Specific Facts Showing Immediate and Irreparable Injury 45.........................................20
2) Efforts to Give Notice 45................................................................................................20
Security 45....................................................................................................................20
b. Notice of Hearing vs.Actual Notice 45............................................................................20
c. Time Limit 45...............................................................................................................20
E. PROVISIONAL REMEDIES 46...................................................................................20
F. PLEADINGS...................................................................................................................21
Rule 12(b)...............................................................................................................................21
a. Must Contain Denials or Admissions and Any Affirmative Defenses ...47......................21
c. Counterclaims...............................................................................................................21
4) Setting Aside a Default or a Default Judgment Inconsistent Claims or Defenses.........21
• 48....................................................................................................................................21
Capacity ....................................................................................................................21
Special Damages....................................................................................................................21
.49 21
7. Amendment and Supplemental Pleadings ......................................................................21
Certification upon Presenting Paper to Court Sanctions ...........................................21
.52 21
1. Joinder of Parties.........................................................................................................21
.52 21
Capacity .......................................................................................................................21
.52 21
a. Successive Claims 54.......................................................................................................22
b. Jurisdiction 54................................................................................................................22
c. Class Actions 54.............................................................................................................22
1) Prerequisites 54..............................................................................................................22
2) Consideration in Treating Case as a Class Action 55...................................................22
a) Court Must Define Class Claims, Issues, or Defenses 55..................................................22
b) Appointment of Class Counsel 55....................................................................................22
3) Effect of Judgment 55....................................................................................................22
a) Personal Jurisdiction Over Absent Class Members Not..........................................22
Required 56.....................................................................................................................22
4) Notice 56.........................................................................................................................22
a) Notice Required in Common Question Suits 56...............................................................22
b) Notice Discretionary in Other Types of Class Action Suits 56.........................................22
5) Jurisdiction 56................................................................................................................22
a) Diversity Action 56..........................................................................................................22
b) Federal Question Action 56.............................................................................................22
6) Court Approval 56.........................................................................................................22
a) Notice of Dismissal or Settlement 57................................................................................22
b) Procedures for Settlements of Class Action Suits 57.....................................................22
(1) "Opt Out" Provision 57...............................................................................................22
c) Appeal of Approval of Settlement 57................................................................................22
7) Appeal of Class Action Certification Decision 57..............................................................22
d. Class Action Fairness Act 57..........................................................................................22
1) Subject Matter Jurisdiction Under the CAFA 57.........................................................22
2) Removal Under the CAFA 58.........................................................................................22
3) Excluded Actions 58........................................................................................................22
a) Primary Defendants Are States or Governmental Entities 58..........................................22
b) Claims Based on Securities Laws or Regarding Corporate Governance 58...........22
4) Local Considerations May Defeat Jurisdiction 58...........................................................22
a) Mandatory Decline of Jurisdiction 58..............................................................................22
b) Discretionary Decline of Jurisdiction 58..........................................................................22
5) Protections Under the CAFA 58....................................................................................22
a) Coupon Settlements 59....................................................................................................22
b) Protection Against Loss by Class Members 59.............................................................22
c) Protection Against Discrimination Based on Geographic........................................22
Location 59.....................................................................................................................22
d) Notification of Federal and State Officials 59...................................................................22
e. Shareholder Derivative Suits 59......................................................................................22
1) Minority Shareholder Allegations 59...............................................................................22
2) Corporation Named as Defendant 60...............................................................................22
3) Jurisdictional Amount and Venue 60...............................................................................22
4) Court Approval 60.........................................................................................................22
f. Interpleader 60..............................................................................................................22
1) Purpose Is to Avoid Double Liability 60...........................................................................22
2) Rights of Plaintiff Stakeholder 60....................................................................................22
3) Jurisdiction 60................................................................................................................22
a) Rule 22 Interpleader 60.................................................................................................22
b) Federal lnterpleader Statute 60.....................................................................................23
g. Intervention 61...............................................................................................................23
1) Intervention of Right 61.................................................................................................23
2) Permissive Intervention 61..............................................................................................23
3) Caveat 61.......................................................................................................................23
h. Third-Party Practice (lmpleader) 61.............................................................................23
1) Claims for Indemnity or Contribution 61........................................................................23
2) Non-Indemnity or Non-Contribution Claims 61..............................................................23
3) Severance of Third-Party Claims 62.............................................................................23
4) Response of Impleaded Party 62...................................................................................23
5) Impleading Insurance Companies 62...............................................................................23
i. Cross-Claims 62.............................................................................................................23
H. DISCOVERY 62...........................................................................................................23
1. Duty of Disclosure 62.....................................................................................................23
a. Types of Disclosure Required 62......................................................................................23
1) Initial Disclosures 62.......................................................................................................23
a) Exemptions from Initial Disclosure Requirement 63........................................................23
2) Disclosure of Expert Testimony 63...................................................................................23
3) Pretrial Disclosures 63...................................................................................................23
2. Discovery of Electronically Stored Data 64...................................................................23
a. Format for Producing Electronic Documents 64...........................................................23
b. Destruction of Electronically Stored Information 64........................................................23
3. Scope of Disclosure and Discovery 64...............................................................................23
a. In General 64................................................................................................................23
b. Trial Preparation Materials 65.....................................................................................23
1) Procedure for Claiming Work Product 65....................................................................23
c. Inadvertent Disclosure of Trial Preparation or Privileged Materials 65...........................23
d. Experts 65......................................................................................................................23
e. Protective Orders 65......................................................................................................23
f. Supplementation of Disclosures and Discovery Responses 65...........................................23
4. Types of Discovery 66....................................................................................................23
a. Pre-Action Depositions 66...............................................................................................23
1) Contents of Petition 66...................................................................................................23
2) Notice and Appointed Counsel 66....................................................................................23
3) Court Order 66.............................................................................................................23
b. Depositions 66.................................................................................................................23
1) Oral Deposition of a Witness, Including a Party-Witness 66............................................23
a) Compulsory Appearance of Witnesses 66........................................................................23
(1) Subpoena Not Needed for Parties 66................................................................................23
(2) Nonparties Should Be Subpoenaed 67..............................................................................23
(3) Costs When Notifying Party Fails to Attend 67............................................................23
2) Deposition of Witnesses on Written Questions 67............................................................23
c. Interrogatories to the Parties 67....................................................................................23
1) Option to Produce Business Records 67...........................................................................23
d. Production of Physical Material; Inspection 68...............................................................23
e. Physical and Mental Examinations 68.............................................................................23
1) Order for Examination 68.............................................................................................23
2) Report of Findings 68....................................................................................................23
f. Requests for Admission 68..............................................................................................24
5. Enforcing Disclosure and Discovery 68............................................................................24
a. Motion for an Order Compelling Disclosure or Discovery 68...........................................24
1) Motion Granted or Discovery Is Provided After Filing 69...............................................24
2) Motion Denied 69............................................................................................................24
3) Motion Granted In Part and Denied In Part 69............................................................24
b. Failure to Comply with a Court Order 69.....................................................................24
c. Failure to Disclose or Supplement an Earlier Response 69...............................................24
d. Failure to Admit 69.......................................................................................................24
e. Party's Failure to Attend His Own Deposition, Serve Answers to Interrogatories,
or Respond to a Request for Inspection 70....................................................................24
6. Use of Depositions at Trial or Hearing 70.....................................................................24
7. Errors and Irregularities in Depositions 70..................................................................24
a. As to Notice 70..............................................................................................................24
b. As to Manner of Taking 70............................................................................................24
c. As to Completion and Return 70...................................................................................24
d. As to Form of Written Questions .71..............................................................................24
I. PRETRIAL CONFERENCES 71...............................................................................24
Rule 26(f) Conference of Parties-Planning for Discovery................................................24
2. Rule 16(b) Scheduling Conference 71..............................................................................24
3. Pretrial Conferences 71.................................................................................................24
4. Sanctions 71...................................................................................................................24
J. ALTERNATIVE DISPUTE RESOLUTION 71...........................................................24
1. Contractual Arbitration 72............................................................................................24
a. Procedure 72.................................................................................................................24
1) Judicial Review of Award 72..........................................................................................24
2. Judicial Arbitration 72..................................................................................................24
3. Mediation 72..................................................................................................................24
K. TRIAL 72......................................................................................................................24
1. Jury Trial Problems 72................................................................................................24
a. Right to Jury Trial 73..................................................................................................24
b. Jury Trials in Diversity Cases 73..................................................................................24
1) Right to a Jury Trial 73...............................................................................................24
2) Motion for New Trial Based on Excessiveness of Verdict 74............................................24
c. Jury Size 74...................................................................................................................24
d. Jury Selection 74............................................................................................................24
1) Jury Venire 74...............................................................................................................24
2) Voir Dire 74...................................................................................................................24
3) Jury Challenges 74.........................................................................................................24
e. Jury Instructions 74.......................................................................................................24
f. Jury Deliberations 74.....................................................................................................24
g. Jury Verdicts 75............................................................................................................24
1) General Verdict 75.........................................................................................................24
2) Special Verdict 75...........................................................................................................24
3) General Verdict with Special Interrogatories 75..............................................................24
4) Erroneous Verdicts and Inconsistent Findings 75............................................................24
5) Juror Misconduct 76......................................................................................................24
Consolidation and Separate Trials 76..................................................................................24
Involuntary Dismissals 76...............................................................................................................................25
Voluntary Dismissals 76..................................................................................................................................25
a. Without Leave of Court 76..........................................................................................25
b. With Leave of Court 76...............................................................................................25
5. Offer of Judgment 77.....................................................................................................25
6. Summary Judgment 77..................................................................................................25
a. Standard 77...................................................................................................................25
b. Applicable to All Civil Actions 77....................................................................................25
c. Time 77..........................................................................................................................25
d. Partial 77.......................................................................................................................25
f. Affidavits 78............................................................................................................................................25
g. Nonappealability 78.........................................................................................................25
h. Relationship to Motion to Dismiss 78...............................................................................25
i. Relationship to Motion for Judgment on the Pleadings 78...............................................25
7. Motion for Judgment as a Matter of Law (Formerly Directed Verdict) 78.................25
8. Renewed Motion for Judgment as a Matter of Law (Formerly Judgment
Notwithstanding the Verdict ("JNOV")) 79.................................................................25
9. Motion for New Trial 79................................................................................................25
a. Reasons for Granting New Trial 79..............................................................................25
1) Remittitur 79..................................................................................................................25
Additur 79.....................................................................................................................25
b. Renewed Motion for Judgment as a Matter of Law with Motion for New Trial 80.....25
10.Effect of Failure to Move for a Renewed Judgment as a Matter of........................25
Law or for a New Trial 80...................................................................................................25
11.Judgment on Partial Findings 80..................................................................................25
V. ATTACK ON THE JUDGMENT AT THE TRIAL COURT LEVEL 80....................25
A. RELIEF FROM JUDGMENT OR ORDER 80........................................................25
1. Clerical Mistakes 80.......................................................................................................25
2. Motions to Amend Prior Orders or Renew Prior Motions 80......................................25
3. Other Grounds for Relief from Judgment 80...............................................................25
B. INDEPENDENT ACTION IN EQUITY TO SET ASIDE THE JUDGMENT 81.......25
VI. FINAL JUDGMENT AND APPELLATE REVIEW 81......................................25
A. JUDGMENT 81.............................................................................................................25
1. Relief that May Be Given 81..........................................................................................25
2. Judgment on Multiple Claims or Parties 81.................................................................25
3. Final Decision on Merits May Be Valid Despite Lack of Subject Matter
Jurisdiction 82................................................................................................................25
B. TIME FOR APPEALS 82...........................................................................................25
1. Extension of Time for Appeal 82...................................................................................25
C. REVIEWABLE ORDER 82........................................................................................25
1. Interlocutory Orders as of Right 82.............................................................................25
a. Injunction 82..................................................................................................................25
b. Receivers 82...................................................................................................................25
c. Admiralty 83..................................................................................................................25
d. Patent Infringement 83................................................................................................26
e. Property Possession 83.................................................................................................26
2. Interlocutory Appeals Act 83......................................................................................26
3. Fewer than All Claims or Parties 83...........................................................................26
4. Collateral Order Rule 83.............................................................................................26
5. Review of Nonappealable Orders by Writ 83...............................................................26
6. Certification of Class Actions 83.................................................................................26
D. STANDARDS OF REVIEW 84..................................................................................26
1. On Matters of Law 84..................................................................................................26
2. On Questions of Fact 84.................................................................................................26
3. On Mixed Questions of Law and Fact 84......................................................................26
4. On Discretionary Matters 84.......................................................................................26
E. STAY PENDING APPEAL 84....................................................................................26
1. Execution 84..................................................................................................................26
2. Bond 84..........................................................................................................................26
3. Injunction Order 84.....................................................................................................26
a. Power of Trial Court 84...............................................................................................26
b. Power of Appellate Court 85.........................................................................................26
F. SUPREME COURT JURISDICTION 85.................................................................26
1. Court of Appeals Cases 85.............................................................................................26
2. Cases from Highest State Court 85.............................................................................26
EFFECTS OF JUDGMENT ON FUTURE CASES 85.................................................26
A. CLAIM PRECLUSION (RES JUDICATA) 85........................................................26
1. Definition 85...................................................................................................................26
2. Terminology Used to Describe Effect-"Merger" and "Bar" 86..............................26
3. Requirements for ''Merger" and "Bar" 86...............................................................26
4. Valid, Final Judgment 86.............................................................................................26
5. ''On the Merits'' 86......................................................................................................26
6. Same Claimant Versus Same Defendant 86..............................................................26
7. ''Cause of Action'' 87...................................................................................................26
a. Common Examples 87.................................................................................................26
1) Accidents 87..................................................................................................................26
2) Installment Obligations 87..........................................................................................26
B. ISSUE PRECLUSION (COLLATERAL ESTOPPEL) 87......................................26
1. Definition 87..................................................................................................................26
2. Requirements 88...........................................................................................................26
a. Final Judgment 88........................................................................................................26
b. Issue Actually Litigated and Determined 88.............................................................26
c. Issue Was Essential to the Judgment 88.....................................................................26
d. Due Process and Mutuality Considerations 88.........................................................26
1) Against Whom Is Issue Preclusion Used? 88............................................................26
2) By Whom Is Issue Preclusion Used? 89....................................................................26
Exceptions to Mutuality When Judgment Used Defensively............................................... 89
26
4) Exceptions to Mutuality When Judgment Used Offensively-..................................26
Consider Fairness 89.........................................................................................................26
C. CLAIM AND ISSUE PRECLUSION IN SPECIAL SITUATIONS 89.................26
1. Judgments for Specific Performance 89....................................................................26
2. In Rem Judgments 90....................................................................................................27
3. Quasi In Rem Judgments 90..........................................................................................27
D. WIDCH PERSONS ARE BOUND BY A JUDGMENT? 90........................................27
1. Parties Are Bound 90...................................................................................................27
2. Privies to Parties Are Bound by Issue Preclusion 90....................................................27
3. Represented Parties May Be Bound by Claim Preclusion 90.......................................27
E. WIDCH CHOICE OF LAW RULES APPLY TO PRECLUSION QUESTIONS? 91
1. Case One Decided in State Court 91.....................................................................27
2. Case One Decided in Federal Court Under Diversity Jurisdiction 91.............................27
CIVIL PROCEDURE-FEDERAL INTRODUCTION......................................................29
A. PERSONAL JURISDICTION.....................................................................................29
B. SUBJECT MATTER JURISDICTION.......................................................................29
1. Diversity of Citizenship Jurisdiction..............................................................................29
2. Federal Question Jurisdiction........................................................................................29
3. Removal Jurisdiction......................................................................................................29
4. Supplemental Jurisdiction..............................................................................................29
C. VENUE..........................................................................................................................29
D. DISCOVERY.................................................................................................................30
E. MULTIPLE PARTIES..................................................................................................30
I. PERSONAL JURISDICTION.....................................................................................31
1. Limitations on Personal Jurisdiction.............................................................................31
a. Statutory Limitations.....................................................................................................31
b. Constitutional Limitations..............................................................................................31
c. Personal Jurisdiction in Federal Courts........................................................................31
2. Three Types of Personal Jurisdiction..........................................................................31
b. In Rem Jurisdiction.......................................................................................................32
c. Quasi In Rem Jurisdiction............................................................................................32
1) Defendant Is Not Bound Personally..............................................................................32
B. STATUTORY LIMITATIONS ON IN PERSONAM JURISDICTION...................32
1. Physical Presence at Time of Personal Service.............................................................32
State Law Exceptions to Traditional Rule.........................................................................33
a. Service by Fraud or Force Invalid...............................................................................33
b. Immunity of Parties and Witnesses...............................................................................33
3. Domicile..........................................................................................................................33
a. Defined............................................................................................................................33
b. Citizenship......................................................................................................................33
4. Consent...........................................................................................................................33
a. Express Consent.............................................................................................................33
1) By Contract....................................................................................................................33
2) By Appointment of Agent to Accept Service of Process...............................................33
a) Appointment Required by State....................................................................................34
b. Implied Consent.............................................................................................................34
c. Voluntary Appearance...................................................................................................34
5. Long Arm Statutes.........................................................................................................34
a. Unlimited Long Arm Statutes.......................................................................................34
b. Limited (or Specific) Long Arm Statutes......................................................................34
1) Limitations in Tort Cases..............................................................................................34
2) Limitations in Contract Cases.......................................................................................35
3) Limitations in Property Actions.....................................................................................35
4) Limitations in Marital Dissolution Cases.........................................................................35
C. CONSTITUTIONAL LIMITATIONS ON IN PERSONAM JURISDICTION.......35
1. Sufficient Contacts with the Forum...............................................................................35
b. Modern Due Process Standard: Contact, Relatedness, and Fairness.........................35
1) Contact...........................................................................................................................36
a) Purposeful Availment.....................................................................................................36
(1) "Stream of Commerce" Cases...........................................................................................37
(2) Internet Cases................................................................................................................37
b) Foreseeability.................................................................................................................38
Relatedness of Claim to Contact.........................................................................................38
a) Claim Arising from Activity in the State (Specific Jurisdiction)..................................38
b) "At Home" in the State (General Jurisdiction).............................................................39
3) Fairness...........................................................................................................................39
a) Convenience.....................................................................................................................39
b) Forum State's Interest....................................................................................................39
c) Other Factors.................................................................................................................40
b. Requirement that Agent Notify Defendant....................................................................40
c. Requirements for Cases Involving Multiple Parties or Unknown Parties...................41
d. Knowledge that Notice by Mail Was Not Received....................................................41
D. IN REM JURISDICTION............................................................................................41
1. Statutory Limitations.....................................................................................................41
2. Constitutional Limitations.............................................................................................41
1) No Jurisdiction If Property Not Located in State........................................................42
2) No Jurisdiction If Property Brought in by Fraud or Force.........................................42
b. Notice..............................................................................................................................42
E. QUASI IN REM JURISDICTION...............................................................................42
1. Statutory Limitations.....................................................................................................42
2. Constitutional Limitations..............................................................................................42
1) Quasi In Rem 'fype I.....................................................................................................42
Quasi In Rem Type II..........................................................................................................43
Procedural Requirements...................................................................................................43
b. Notice..............................................................................................................................43
II. DIVERSITY OF CITIZENSHIP JURISDICTION....................................................43
B. DIVERSITY AMONG THE PARTIES.......................................................................43
a. Multiple Parties-Complete Diversity.............................................................................44
1) But Note.........................................................................................................................44
2) lnterpleader Exception...................................................................................................44
a) Federal lnterpleader Statute-Minimal Diversity............................................................44
b) Interpleader Under Federal Rules-Complete Diversity................................................44
b. "Alienage" Jurisdiction.................................................................................................44
1) Aliens as Additional Parties..........................................................................................45
c. Diversity When Action Is Commenced.........................................................................45
2. Questions of Citizenship.................................................................................................45
a. State Citizenship of an Individual-Domicile.................................................................45
b. Citizenship of a Corporation-Possible Multiple Citizenships........................................45
1) Special Rule for Direct Actions.....................................................................................46
2) Incorporation or Principal Place of Business in Foreign Country..............................46
c. Unincorporated Associations.........................................................................................46
1) Citizenship......................................................................................................................46
2) Class Action...................................................................................................................46
3) Partnerships...................................................................................................................46
4) Limited Liability Companies.........................................................................................46
d. Business Trusts..............................................................................................................47
e. Legal Representatives....................................................................................................47
f. Class Actions..................................................................................................................47
g. Nonresident United States Citizens................................................................................47
3. Collusion and Devices to Create or Defeat Diversity....................................................47
a. Assignment of Claims....................................................................................................47
b. Class Actions..................................................................................................................47
c. Voluntary Change of State Citizenship.........................................................................47
4. Realignment According to Interest................................................................................47
a. May Create or Destroy Diversity...................................................................................47
b. Shareholder Derivative Actions.....................................................................................48
5. "Supplemental" Jurisdiction.........................................................................................48
6. Joinder or Subsequent Addition of Parties..................................................................48
a. Restriction on the Use of Supplemental Jurisdiction in Diversity Cases.....................48
b. Intervention of Right......................................................................................................49
c. Permissive Intervention..................................................................................................49
d. Substitution of Parties....................................................................................................49
e. Third-Party Practice-Impleader....................................................................................49
1) Subject Matter Jurisdiction Required...........................................................................50
f. Cross- Claims.................................................................................................................50
1) Subject Matter Jurisdiction Required...........................................................................50
C. JURISDICTIONAL AMOUNT: IN EXCESS OF $75,000........................................51
What Is "In Controversy"?................................................................................................51
a. Collateral Consequences of the Judgment...................................................................51
b. Interest and Costs..........................................................................................................51
c. Equitable Relief..............................................................................................................51
d. Punitive Damages...........................................................................................................52
Aggregation of Separate Claims.........................................................................................52
a. One Plaintiff Against One Defendant...........................................................................52
b. One Plaintiff Against Several Defendants.....................................................................52
c. Several Plaintiffs Against One Defendant......................................................................52
4. Counterclaims................................................................................................................53
a. Compulsory Counterclaim Need Not Meet Jurisdictional Amount.............................53
b. Permissive Counterclaim Must Meet Jurisdictional Amount......................................53
c. No Removal to Federal Court Based on Counterclaim...............................................53
1. Federal Statutes or Federal Rules of Civil Procedure...................................................54
a. Caution...........................................................................................................................54
b. Recent Application.........................................................................................................54
2. If There Is No Federal Statute or Rule on Point, Is the Issue Substantive or
Procedural?....................................................................................................................54
a. Some Situations Are Clearly Established......................................................................55
b. Law Is Unclear in Other Situations..............................................................................55
3. Statutes Involving Both Substance and Procedure.......................................................55
4. Interpreting State Law..................................................................................................56
a. De Novo Review of District Court's Decision...............................................................56
b. Subsequent State Court Decisions.................................................................................56
E. FEDERAL COMMON LAW......................................................................................56
1. When Federal Courts Create Federal Common Law Rules........................................56
a. Interpretation of Federal Statute or Constitution..........................................................56
b. Creating Rules to Fill Gaps in Federal Regulatory Schemes............................................56
1) Formulating Uniform Federal Standard.......................................................................56
2) Borrowing State Standard.............................................................................................57
When Federal Courts Create Federal Implied Rights of Action..........................................57
a. Implied Right of Action Based on Federal Statute......................................................57
1) Need for Affirmative Congressional Intent...................................................................57
b. Implied Right of Action Based on Constitution...........................................................57
F. EXCEPTIONS TO DIVERSITY OF CITIZENSHIP JURISDICTION....................58
1. Domestic Relations.........................................................................................................58
2. Probate Proceedings......................................................................................................58
G. MULTIPARTY, MULTIFORUM TRIAL JURISDICTION ACT.............................58
1. Requirements.................................................................................................................58
a. The Action......................................................................................................................58
b. Minimal Diversity..........................................................................................................58
c. One Additional Condition..............................................................................................58
2. Intervention....................................................................................................................59
3. Service of Process..........................................................................................................59
III.
FEDERAL QUESTION JURISDICTION..................................................................59
A. INTRODUCTION.........................................................................................................59
B. FEDERAL QUESTION MUST APPEAR IN THE COMPLAINT..........................59
1. Defendant's Answer or Defense Is Irrelevant................................................................59
2. Anticipation of a Defense...............................................................................................59
C. IMPLIED FEDERAL RIGHT OF ACTION..............................................................59
D. FEDERAL CORPORATIONS.......................................................................................60
E. SUPPLEMENTAL (PENDENT) JURISDICTION OVER STATE CLAIMS..........60
1. Supplemental (Pendent) Claims.....................................................................................60
a. Effect of Dismissal of Federal Claim on Supplemental (Pendent) Claim...........................60
2. Pendent Parties..............................................................................................................61
F. SPECIFIC STATUTORY GRANTS...........................................................................61
1. Amount in Controversy..................................................................................................61
2. Exclusive Jurisdiction....................................................................................................61
c. Many Cases Where United States Is Involved.............................................................61
e. Antitrust Cases..............................................................................................................62
f. Admiralty Cases-Caveat................................................................................................62
g. Foreign State-Caveat....................................................................................................62
IV.VENUE..........................................................................................................................62
A. SUBJECT MATTER JURISDICTION DISTINGUISHED.......................................62
B. GENERAL RULES.......................................................................................................62
1. General Rules for Most Civil Actions...........................................................................62
2. Special Venue Provisions...............................................................................................63
C. RESIDENCE.................................................................................................................63
1. Natural Persons..............................................................................................................63
2. Business Entities.............................................................................................................63
a. Note for Corporations....................................................................................................63
3. Nonresident of United States..........................................................................................63
D. IMPROPER VENUE MAY BE WAIVED..................................................................64
E. TRANSFER...................................................................................................................64
1. Original Venue Proper..................................................................................................64
2. Original Venue Improper..............................................................................................64
3. Effect of Forum Selection Clauses.................................................................................64
4. Original Court Lacks Personal Jurisdiction.................................................................64
F. LAW APPLICABLE UPON TRANSFER...................................................................64
1. Original Venue Proper..................................................................................................64
2. Original Venue Improper...............................................................................................65
V. REMOVAL JURISDICTION......................................................................................65
A. ORIGINAL JURISDICTION NECESSARY.............................................................65
1. When..............................................................................................................................65
2. Federal Defense Insufficient...........................................................................................65
3. State Court Need Not Have Had Jurisdiction...............................................................65
B. ONLY DEFENDANT MAY REMOVE; ALL MUST SEEK REMOVAL................65
C. VENUE..........................................................................................................................66
D. DEFENDANT MAY REMOVE SEPARATE AND INDEPENDENT FEDERAL
QUESTION CLAIM.....................................................................................................66
E. DISMISSAL OF NONDIVERSE PARTY ALLOWS REMOVAL............................66
F. LIMITATIONS ON REMOVAL IN DIVERSITY OF CITIZENSHIP CASES........66
1. Defendant Citizen of Forum State..................................................................................66
2. One Year Rule...............................................................................................................66
G. PROCEDURE FOR REMOVAL.................................................................................67
a. Allegation of Amount in Controversy............................................................................67
2. Thirty Day Rule.............................................................................................................67
3. Procedure After Removal..............................................................................................68
4. Right to Jury Trial.........................................................................................................68
b. Demand Not Required..................................................................................................68
5. Remand..........................................................................................................................68
Subject Matter Jurisdiction Generally Considered First................................................69
H. SPECIFIC TYPES OF ACTIONS...............................................................................69
2. Nonremovable................................................................................................................69
All Writs Act Not a Basis for Removal..............................................................................69
VI. CONFLICT OF JURISDICTION BETWEEN STATE AND FEDERAL COURTS
69
B. INJUNCTIONS AGAINST PENDING STATE PROCEEDINGS..............................70
D. INJUNCTIONS AGAINST STATE TAX PROCEDURES.......................................70
VII. THE FEDERAL RULES OF CIVIL PROCEDURE..................................................70
B. SERVICE OF PROCESS.............................................................................................70
2. How Service Is Made.....................................................................................................70
a. Service Under State Rules.............................................................................................71
b. Waiver of Service (Service by Mail).............................................................................71
1) Effect of Waiver..............................................................................................................71
2) Effect of Failure to Waive..............................................................................................71
3. Parties Served Outside State.........................................................................................71
4. Parties Served in Foreign Country...............................................................................72
5. Immunity from Process..................................................................................................72
C. TIME PERIODS...........................................................................................................72
2. Extensions of Time.........................................................................................................72
D. INTERLOCUTORY INJUNCTIONS.........................................................................72
1. Preliminary Injunction...................................................................................................73
a. Requirements for a Preliminary Injunction..................................................................73
2. Temporary Restraining Order.......................................................................................73
a. Requirements for Ex Parte Temporary Restraining Orders........................................73
1) Specific Facts Showing Immediate and Irreparable Injury.........................................73
2) Efforts to Give Notice.....................................................................................................73
3) Security...........................................................................................................................73
b. Notice of Hearing vs. Actual Notice...............................................................................73
c. Time Limit.....................................................................................................................73
E. PROVISIONAL REMEDIES.......................................................................................74
F. PLEADINGS..................................................................................................................74
1. Complaint.......................................................................................................................74
2. Pre-Answer Motions......................................................................................................74
b. Motion for More Definite Statement.............................................................................75
c. Motion to Strike.............................................................................................................75
3. Answer...........................................................................................................................75
b. Time...............................................................................................................................75
c. Counterclaims................................................................................................................76
d. Effect of Failure to Answer-Default and Default Judgment.........................................76
1) Default.............................................................................................................................76
2) Default Judgment...........................................................................................................76
a) Default Judgment Entered by the Clerk.......................................................................76
3) Notice Required.............................................................................................................76
4) Setting Aside a Default or a Default Judgment............................................................77
4. Inconsistent Claims or Defenses....................................................................................77
5. Special Pleading.............................................................................................................77
a. Capacity..........................................................................................................................77
b. Fraud or Mistake...........................................................................................................77
c. Conditions of the Mind..................................................................................................77
d. Conditions Precedent.....................................................................................................77
e. Official Document or Act...............................................................................................77
3) Conform to Evidence.....................................................................................................79
4) Due Process Limitation.................................................................................................79
b. Supplemental Pleadings.................................................................................................79
8. Rule 11............................................................................................................................79
2) Can the Absentee Be Joined?.......................................................................................81
3) Should the Action Proceed Without the Absentee?.....................................................81
c. Permissive Joinder-Requirements..................................................................................81
2. Joinder of Claims...........................................................................................................82
a. Successive Claims...........................................................................................................82
b. Jurisdiction.....................................................................................................................82
c. Class Actions..................................................................................................................82
2) Consideration in Treating Case as a Class Action.......................................................83
a) Court Must Define Class Claims, Issues, or Defenses.................................................83
b) Appointment of Class Counsel.......................................................................................83
3) Effect of Judgment.........................................................................................................83
a) Personal Jurisdiction Over Absent Class Members Not Required.............................84
4) Notice..............................................................................................................................84
b) Notice Discretionary in Other Types of Class Action Suits.........................................84
5) Jurisdiction.....................................................................................................................84
b) Federal Question Action................................................................................................84
6) Court Approval..............................................................................................................84
a) Notice of Dismissal or Settlement..................................................................................85
b) Procedures for Settlements of Class Action Su.its.......................................................85
(1) "Opt Out" Provision.....................................................................................................85
c) Appeal of Approval of Settlement.................................................................................85
7) Appeal of Class Action Certification Decision.............................................................85
d. Class Action Fairness Act.............................................................................................85
1) Subject Matter Jurisdiction Under the CAFA...............................................................85
a) Coupon Settlements.......................................................................................................87
b) Protection Against Loss by Class Members................................................................87
c) Protection Against Discrimination Based on Geographic Location............................87
d) Notification of Federal and State Officials...................................................................87
e. Shareholder Derivative Suits.........................................................................................87
2) Corporation Named as Defendant.................................................................................88
3) Jurisdictional Amount and Venue.................................................................................88
4) Court Approval..............................................................................................................88
f. lnterpleader....................................................................................................................88
2) Rights of Plaintiff Stakeholder......................................................................................88
3) Jurisdiction.....................................................................................................................88
b) Federal lnterpleader Statute..........................................................................................88
g. Intervention....................................................................................................................89
1) Intervention of Right......................................................................................................89
2) Permissive Intervention..................................................................................................89
3) Caveat.............................................................................................................................89
h. Third-Party Practice (Impleader).................................................................................89
2) Non-Indemnity or Non-Contribution Claims................................................................89
3) Severance of Third-Party Claims..................................................................................90
4) Response of Impleaded Party........................................................................................90
5) Impleading Insurance Companies.................................................................................90
i. Cross-Claims.................................................................................................................90
H. DISCOVERY.................................................................................................................90
a. Types of Disclosure Required.......................................................................................90
1) Initial Disclosures...........................................................................................................90
a) Exemptions from Initial Disclosure Requirement.........................................................91
2) Disclosure of Expert Testimony.....................................................................................91
3) Pretrial Disclosures........................................................................................................91
Discovery of Electronically Stored Data............................................................................92
a. Format for Producing Electronic Documents...............................................................92
b. Destruction of Electronically Stored Information.........................................................92
3. Scope of Disclosure and Discovery................................................................................92
4. 'fypes of Discovery.........................................................................................................94
1) Contents of Petition........................................................................................................94
Notice and Appointed Counsel...........................................................................................94
3) Court Order...................................................................................................................94
b. Depositions.....................................................................................................................94
1) Oral Deposition of a Witness, Including a Party-Witness............................................94
a) Compulsory Appearance of Witnesses..........................................................................94
(2) Nonparties Should Be Subpoenaed...............................................................................95
(3) Costs When Notifying Party Fails to Attend.................................................................95
2) Deposition of Witnesses on Written Questions.............................................................95
c. Interrogatories to the Parties.........................................................................................95
1) Option to Produce Business Records............................................................................95
d. Production of Physical Material; Inspection.................................................................96
e. Physical and Mental Examinations................................................................................96
2) Report of Findings.........................................................................................................96
f. Requests for Admission.................................................................................................96
5. Enforcing Disclosure and Discovery.............................................................................96
Motion Granted or Discovery Is Provided After Filing...................................................97
Motion Denied......................................................................................................................97
3) Motion Granted In Part and Denied In Part...............................................................97
b. Failure to Comply with a Court Order.........................................................................97
c. Failure to Disclose or Supplement an Earlier Response..............................................97
d. Failure to Admit.............................................................................................................97
e. Party's Failure to Attend His Own Deposition, Serve Answers to Interrogatories, or
Respond to a Request for Inspection............................................................................98
6. Use of Depositions at Trial or Hearing.........................................................................98
7. Errors and Irregularities in Depositions.......................................................................98
a. As to Notice....................................................................................................................98
b. As to Manner of Taking...............................................................................................98
c. As to Completion and Return..........................................................................................98
d. As to Form of Written Questions..................................................................................99
I. PRETRIAL CONFERENCES.....................................................................................99
2. Rule 16(b) Scheduling Conference................................................................................99
3. Pretrial Conferences.......................................................................................................99
4. Sanctions........................................................................................................................99
J. ALTERNATIVE DISPUTE RESOLUTION...............................................................99
1. Contractual Arbitration...............................................................................................100
a. Procedure.....................................................................................................................100
1) Judicial Review of Award............................................................................................100
2. Judicial Arbitration......................................................................................................100
3. Mediation......................................................................................................................100
K. TRIAL.........................................................................................................................100
2) Motion for New Trial Based on Excessiveness of Verdict.........................................102
c. Jury Size......................................................................................................................102
d. Jury Selection..............................................................................................................102
2) Voir Dire......................................................................................................................102
3) Jury Challenges...........................................................................................................102
e. Jury Instructions..........................................................................................................102
f. Jury Deliberations........................................................................................................102
g. Jury Verdicts................................................................................................................103
1) General Verdict............................................................................................................103
2) Special Verdict.............................................................................................................103
3) General Verdict with Special Interrogatories.............................................................103
4) Erroneous Verdicts and Inconsistent Findings...........................................................103
5) Juror Misconduct........................................................................................................104
2. Consolidation and Separate Trials..............................................................................104
3. Involuntary Dismissals.................................................................................................104
4. Voluntary Dismissals...................................................................................................104
a. Without Leave of Court...............................................................................................104
b. With Leave of Court...................................................................................................104
5. Offer of Judgment........................................................................................................105
6. Summary Judgment.....................................................................................................105
b. Applicable to All Civil Actions....................................................................................105
c. Time.............................................................................................................................105
d. Partial...........................................................................................................................105
e. Support.........................................................................................................................105
78. CIVIL PROCEDURE-FEDERAL.............................................................................106
g. Nonappealability..........................................................................................................106
h. Relationship to Motion to Dismiss...............................................................................106
i. Relationship to Motion for Judgment on the Pleadings.............................................106
7. Motion for Judgment as a Matter of Law (Formerly Directed Verdict)...................106
8. Renewed Motion for Judgment as a Matter of Law (Formerly Judgment
Notwithstanding the Verdict ("JNOV"))....................................................................107
9. Motion for New Trial..................................................................................................107
a. Reasons for Granting New Trial.................................................................................107
1) Remittitur.....................................................................................................................107
2) Additur.........................................................................................................................107
10. Effect of Failure to Move for a Renewed Judgment as a Matter of Law or for a New
Trial..............................................................................................................................108
11. Judgment on Partial Findings.....................................................................................108
VIII.ATTACK ON THE JUDGMENT AT THE TRIAL COURT LEVEL..................108
2. Motions to Amend Prior Orders or Renew Prior Motions.......................................108
3. Other Grounds for Relief from Judgment..................................................................108
B. INDEPENDENT ACTION IN EQillTY TO SET ASIDE THE JUDGMENT.......109
IX. FINAL JUDGMENT AND APPELLATE REVIEW...............................................109
2. Judgment on Multiple Claims or Parties....................................................................109
B. TIME FOR APPEALS...............................................................................................110
1. Extension of Time for Appeal.....................................................................................110
C. REVIEWABLE ORDER............................................................................................110
1. Interlocutory Orders as of Right.................................................................................110
b. Receivers.......................................................................................................................110
c. Admiralty.....................................................................................................................111
d. Patent Infringement.....................................................................................................111
e. Property Possession......................................................................................................111
2. Interlocutory Appeals Act............................................................................................111
3. Fewer than All Claims or Parties................................................................................111
4. Collateral Order Rule..................................................................................................111
5. Review of Nonappealable Orders by Writ..................................................................111
6. Certification of Class Actions......................................................................................111
D. STANDARDS OF REVIEW.........................................................................................112
2. On Questions of Fact...................................................................................................112
3. On Mixed Questions of Law and Fact........................................................................112
4. On Discretionary Matters............................................................................................112
E. STAY PENDING APPEAL.........................................................................................112
1. Execution......................................................................................................................112
2. Bond.............................................................................................................................112
3. Injunction Order..........................................................................................................112
b. Power of Appellate Court............................................................................................113
F. SUPREME COURT JURISDICTION......................................................................113
1. Court of Appeals Cases...............................................................................................113
2. Cases from Highest State Court..................................................................................113
X. EFFECTS OF JUDGMENT ON FUTURE CASES................................................113
2. Terminology Used to Describe Effect-"Merger" and "Bar".....................................114
3. Requirements for "Merger" and "Bar"....................................................................114
4. Valid, Final Judgment..................................................................................................114
5. "On the Merits"..........................................................................................................114
6. Same Claimant Versus Same Defendant.....................................................................114
7. "Cause of Action"........................................................................................................115
a. Common Examples......................................................................................................115
2) Installment Obligations...............................................................................................115
B. ISSUE PRECLUSION (COLLATERAL ESTOPPEL)............................................115
2. Requirements...............................................................................................................116
b. Issue Actually Litigated and Determined...................................................................116
c. Issue Was Essential to the Judgment..........................................................................116
d. Due Process and Mutuality Considerations.................................................................116
By Whom Is Issue Preclusion Used?................................................................................117
Exceptions to Mutuality When Judgment Used Defensively.........................................117
Exceptions to Mutuality When Judgment Used Offensively-Consider Fairness......................117
C. CLAIM AND ISSUE PRECLUSION IN SPECIAL SITUATIONS........................117
2. In Rem Judgments.......................................................................................................118
3. Quasi In Rem Judgments............................................................................................118
D. WHICH PERSONS ARE BOUND BY A JUDGMENT?........................................118
2. Privies to Parties Are Bound by Issue Preclusion.....................................................118
3. Represented Parties May Be Bound by Claim Preclusion.........................................118
1. Case One Decided in State Court................................................................................119
2. Case One Decided in Federal Court Under Diversity Jurisdiction...........................119
TIMING OF PRE-ANSWER MOTIONS.......................................................................121
FEDERAL CLASS ACTION REQUIREMENTS...........................................................................................122
CIVIL PROCEDURE MULTIPLE CHOICE QUESTIONS INTRODUCTORY NOTE...124
Question 1.........................................................................................................................................................124
Question 2........................................................................................................................................................124
Question 3.........................................................................................................................................................125
Question 4.........................................................................................................................................................125
Question 5.........................................................................................................................................................126
Question 6.........................................................................................................................................................126
Question 9............................................................................................................................128
Question 10..........................................................................................................................128
Question 11..........................................................................................................................129
Question 12............................................................................................................................129
ANSWERS TO MULTIPLE CHOICE QUESTIONS...................................................130
Answer to Question 2.........................................................................................................130
Answer to Question 3..........................................................................................................130
Answer to Question 4.......................................................................................................................................131
Answer to Question 5........................................................................................................................................131
Answer to Question 6.......................................................................................................................................131
Answer to Question 7.........................................................................................................132
Answer to Question 8.........................................................................................................132
Answer to Question 9.........................................................................................................132
Answer to Question 10.....................................................................................................................................133
Answer to Question 11.....................................................................................................................................133
Answer to Question 12.....................................................................................................................................133
APPROACH TO EXAMS CIVIL PROCEDURE..............................................................134
I. JURISDICTION OVER THE PERSON...................................................................134
A. Types of Personal Jurisdiction....................................................................................134
B. Statutory Limitations on Personal Jurisdiction..........................................................134
Constitutional Limitations on Personal Jurisdiction...............................................................134
II. SUBJECT MATTER JURISDICTION IN FEDERAL COURTS...........................135
B. Federal Question Jurisdiction......................................................................................136
III. VENUE.........................................................................................................................136
C. Residence for Venue Purposes......................................................................................136
D. Transfer........................................................................................................................136
IV. REMOVAL JURISDICTION.....................................................................................137
A. Requirements...............................................................................................................137
B. Procedure for Removal................................................................................................137
V. CONFLICT OF JURISDICTION BETWEEN STATE AND FEDERAL COURTS
......................................................................................................................................137
VI. FEDERAL RULES OF CIVIL PROCEDURE.........................................................137
C. Interlocutory Injunctions-Maintain Status Quo Until Trial........................................138
D. Pleadings......................................................................................................................138
E. Joinder..........................................................................................................................138
VII. DUTY OF DISCLOSURE AND DISCOVERY........................................................140
B. Types of Discovery upon Request...............................................................................140
C. Enforcing Disclosure and Discovery............................................................................140
D. Use of Depositions at Trial or Hearing.......................................................................140
E. Pretrial Conferences....................................................................................................141
VIII. TRIAL..................................................................................................................141
B. Trial..............................................................................................................................141
IX. POST-TRIAL MOTIONS AND APPEALS..............................................................141
B. Final Judgment and Appellate Review.......................................................................141
X. CLAIM AND ISSUE PRECLUSION.......................................................................142
B. Who Is Bound by Judgment?......................................................................................142
ESSAY EXAM QUESTIONS...........................................................................................144
EXAM QUESTION NO. 1................................................................................................145
EXAM QUESTION NO. 2................................................................................................146
EXAM QUESTION NO. 3................................................................................................147
EXAM QUESTION NO. 4................................................................................................148
ANSWERS TO ESSAY EXAM QUESTIONS ANSWER TO EXAM QUESTION NO. 1
......................................................................................................................................150
Motion to Dismiss for Lack of Personal Jurisdiction.....................................................150
ANSWER TO EXAM QUESTION NO. 2.......................................................................151
Statutorily Authorized.......................................................................................................151
Service of Process...............................................................................................................152
Constitutionality.................................................................................................................152
ANSWER TO EXAM QUESTION NO. 3.......................................................................152
(2) Joinder of Defendants..................................................................................................153
Joinder of Claims...............................................................................................................153
(3) Ped's Requests for Admissions....................................................................................153
(4) Motion for New Trial..................................................................................................153
ANSWER TO EXAM QUESTION NO. 4.......................................................................154
(2) Peter and Quincy v. Valeo and Mity..........................................................................154
(3) Peter v. Valeo and Mity...............................................................................................156
v1. CIVIL PROCEDURE-FEDERAL ti) barbrr

G. PROCEDURE FOR REMOVAL........................................................................................39


1. Notice of Removal.........................................................................................................39
a. Allegation of Amount in Controversy.........................................................................39
2. Thirty Day Rule.......................................................................................................39
3. Procedure After Removal............................................................................................40
4. Right to Jury Trial...................................................................................................40
a. Demand for Jury Trial....................................................................................40
b. Demand Not Required..................................................................................................40
5. Remand.....................................................................................................................40
a. Subject Matter Jurisdiction Generally Considered First..........................................41
H. SPECIFIC TYPES OF ACTIONS.................................................................................41
1. Removable..................................................................................................................... 41
2. Nonremovable........................................................................................................................41
a. All Writs Act Not a Basis for Removal...............................................................41
III. CONFLICT OF JURISDICTION BETWEEN STATE AND FEDERAL COURTS..............41
A. FULL FAITH AND CREDIT EXTENDED TO FEDERAL COURTS...........................41
B. INJUNCTIONS AGAINST PENDING STATE PROCEEDINGS............................................42
C. INJUNCTIONS AGAINST THREATENED STATE CRIMINAL PROSECUTIONS
42
D. INJUNCTIONS AGAINST STATE TAX PROCEDURES..............................................42
IV. THE FEDERAL RULES OF CIVIL PROCEDURE...........................................................42
A. COMMENCEMENT OF THE ACTION...........................................................................42
B. SERVICE OF PROCESS...............................................................................................42
1. Who May Serve.......................................................................................................42
2. How Service Is Made....................................................................................................42
a. Service Under State Rules...................................................................................43
b. Waiver of Service (Service by Mail)....................................................................43
1) Effect of Waiver...........................................................................................43
2) Effect of Failure to Waive............................................................................43
3. Parties Served Outside State........................................................................................43
4. Parties Served in Foreign Country.............................................................................44
5. Immunity from Process................................................................................................44
C. TIME PERIODS..............................................................................................................44
1. Counting Time..............................................................................................................44
2. Extensions of Time.......................................................................................................44
D. INTERLOCUTORY INJUNCTIONS.................................................................................44
1. Preliminary Injunction..........................................................................................................45
a. Requirements for a Preliminary Injunction...............................................................45
2. Temporary Restraining Order....................................................................................45
a. Requirements for Ex Parte Temporary Restraining Orders............................45
1) Specific Facts Showing Immediate and Irreparable Injury.............................45
2) Efforts to Give Notice...................................................................................45
Security.....................................................................................................45
b. Notice of Hearing vs.Actual Notice .....................................................................45
c. Time Limit.......................................................................................................45
E. PROVISIONAL REMEDIES..............................................................................................46
@borbrr CIVIL PROCEDURE-FEDERAL vn.

F. PLEADINGS
•• 46
1. Complaint •• 46
2. Pre-Answer Motions . • 46
a. Rule 12(b) • 46
bc.. Motion for More Definite Statement .47
Motion to Strike .47
3. Answer . . . . . . . . .47
a. Must Contain Denials or Admissions and Any Affirmative Defenses ...47
b. Time...... .47
c. Counterclaims • 48
d. Effect of Failure to Answer-Default and Default Judgment . • 48
1) Default ...... • 48
2) Default Judgment • 48
a) Default Judgment Entered by the Clerk • 48
3) Notice Required . . . . . . . . . . . . . . . . • 48
4) Setting Aside a Default or a Default Judgment .49
4 Inconsistent Claims or Defenses .49
5. Special Pleading .....
. .49
a. Capacity ...... .49
b. Fraud or Mistake . .49
c. Conditions of the Mind .. .49
d. Conditions Precedent ..
Official Document or Act .49
fe.. Judgment .... .49
g. Timing...... • 50
h. Special Damages • 50
6. • 50
Reply . . . . . . . . . • 50
7. Amendment and Supplemental Pleadings . • 50
a. Amendment .... • 50
1) Relation Back • 50
2) Changing Party . • 50
3) Conform to Evidence . . 51
4) Due Process Limitation . . 51
b. Supplemental Pleadings . 51
8. Rile 11 . . . . . . . . . . . . . . 51
a. Certification upon Presenting Paper to Court .. 5521
b. Sanctions ....... .52
1) Court's Initiative .52
2) Party's Motion .
G. JOINDER . . . . . . . . .52
1. Joinder of Parties .52
a. Capacity .... .52
b. Compulsory Joinder (Indispensable Parties) . .52
1) Should the Absentee Be Joined? .52
2. c. 2) Can the Absentee Be Joined? .. ..53
3) of
Joinder Should
Claims the Action Proceed Without the Absentee?
............. ..53
Permissive Joinder-Requirements . ..53
••
54
v111. CIVIL PROCEDURE-FEDERAL ti) barbrr

a. Successive Claims..........................................................................................................54
b. Jurisdiction........................................................................................................... 54
c. Class Actions......................................................................................................... 54
1) Prerequisites.................................................................................................54
2) Consideration in Treating Case as a Class Action.....................................55
a) Court Must Define Class Claims, Issues, or Defenses..............................55
b) Appointment of Class Counsel...................................................................55
3) Effect of Judgment.......................................................................................55
a) Personal Jurisdiction Over Absent Class Members Not
Required...............................................................................................56
4) Notice............................................................................................................56
a) Notice Required in Common Question Suits............................................56
b) Notice Discretionary in Other Types of Class Action Suits....................56
5) Jurisdiction...................................................................................................56
a) Diversity Action...........................................................................................56
b) Federal Question Action..............................................................................56
6) Court Approval............................................................................................56
a) Notice of Dismissal or Settlement...............................................................57
b) Procedures for Settlements of Class Action Suits...............................57
(1) "Opt Out" Provision...............................................................57
c) Appeal of Approval of Settlement..............................................................57
7) Appeal of Class Action Certification Decision....................................................57
d. Class Action Fairness Act.....................................................................................57
1) Subject Matter Jurisdiction Under the CAFA...........................................57
2) Removal Under the CAFA...........................................................................58
3) Excluded Actions..................................................................................................58
a) Primary Defendants Are States or Governmental Entities.....................58
b) Claims Based on Securities Laws or Regarding Corporate
Governance...........................................................................................58
4) Local Considerations May Defeat Jurisdiction.................................................58
a) Mandatory Decline of Jurisdiction............................................................58
b) Discretionary Decline of Jurisdiction........................................................58
5) Protections Under the CAFA......................................................................58
a) Coupon Settlements.....................................................................................59
b) Protection Against Loss by Class Members.......................................59
c) Protection Against Discrimination Based on Geographic
Location............................................................................................59
d) Notification of Federal and State Officials................................................59
e. Shareholder Derivative Suits........................................................................................59
1) Minority Shareholder Allegations......................................................................59
2) Corporation Named as Defendant......................................................................60
3) Jurisdictional Amount and Venue......................................................................60
4) Court Approval............................................................................................60
f. Interpleader..........................................................................................................60
1) Purpose Is to Avoid Double Liability..................................................................60
2) Rights of Plaintiff Stakeholder............................................................................60
3) Jurisdiction...................................................................................................60
a) Rule 22 Interpleader............................................................................60
@borbrr CIVIL PROCEDURE-FEDERAL 1x.

b) Federal lnterpleader Statute...............................................................60


g. Intervention...........................................................................................................61
1) Intervention of Right....................................................................................61
2) Permissive Intervention.......................................................................................61
3) Caveat.......................................................................................................61
h. Third-Party Practice (lmpleader)........................................................................ 61
1) Claims for Indemnity or Contribution...............................................................61
2) Non-Indemnity or Non-Contribution Claims....................................................61
3) Severance of Third-Party Claims...............................................................62
4) Response of Impleaded Party......................................................................62
5) Impleading Insurance Companies......................................................................62
i. Cross-Claims.........................................................................................................62
H. DISCOVERY...................................................................................................................62
1. Duty of Disclosure.........................................................................................................62
a. Types of Disclosure Required.......................................................................................62
1) Initial Disclosures.................................................................................................62
a) Exemptions from Initial Disclosure Requirement....................................63
2) Disclosure of Expert Testimony..........................................................................63
3) Pretrial Disclosures...................................................................................... 63
2. Discovery of Electronically Stored Data.....................................................................64
a. Format for Producing Electronic Documents....................................................64
b. Destruction of Electronically Stored Information.....................................................64
3. Scope of Disclosure and Discovery.......................................................................................64
a. In General.........................................................................................................64
b. Trial Preparation Materials................................................................................ 65
1) Procedure for Claiming Work Product......................................................65
c. Inadvertent Disclosure of Trial Preparation or Privileged Materials......................65
d. Experts..............................................................................................................65
e. Protective Orders..................................................................................................65
f. Supplementation of Disclosures and Discovery Responses.......................................65
4. Types of Discovery........................................................................................................66
a. Pre-Action Depositions.................................................................................................66
1) Contents of Petition......................................................................................66
2) Notice and Appointed Counsel............................................................................66
3) Court Order.............................................................................................66
b. Depositions.....................................................................................................................66
1) Oral Deposition of a Witness, Including a Party-Witness................................66
a) Compulsory Appearance of Witnesses......................................................66
(1) Subpoena Not Needed for Parties.....................................................66
(2) Nonparties Should Be Subpoenaed...................................................67
(3) Costs When Notifying Party Fails to Attend.............................67
2) Deposition of Witnesses on Written Questions..................................................67
c. Interrogatories to the Parties...............................................................................67
1) Option to Produce Business Records..................................................................67
d. Production of Physical Material; Inspection..............................................................68
e. Physical and Mental Examinations.............................................................................68
1) Order for Examination................................................................................68
2) Report of Findings.......................................................................................68
x. CIVIL PROCEDURE-FEDERAL ti) barbrr

f. Requests for Admission................................................................................................68


5.
Enforcing Disclosure and Discovery....................................................................................68
a. Motion for an Order Compelling Disclosure or Discovery.......................................68
1) Motion Granted or Discovery Is Provided After Filing....................................69
2) Motion Denied......................................................................................................69
3) Motion Granted In Part and Denied In Part..............................................69
b. Failure to Comply with a Court Order................................................................69
c. Failure to Disclose or Supplement an Earlier Response............................................69
d. Failure to Admit...............................................................................................69
e. Party's Failure to Attend His Own Deposition, Serve Answers to
Interrogatories, or Respond to a Request for Inspection..................................70
6. Use of Depositions at Trial or Hearing........................................................................70
7. Errors and Irregularities in Depositions.....................................................................70
a. As to Notice......................................................................................................70
b. As to Manner of Taking....................................................................................... 70
c. As to Completion and Return.............................................................................. 70
d. As to Form of Written Questions .71
I. PRETRIAL CONFERENCES....................................................................................... 71
Rule 26(f) Conference of Parties-Planning for Discovery .71
2. Rule 16(b) Scheduling Conference.......................................................................................71
3. Pretrial Conferences..................................................................................................... 71
4. Sanctions.......................................................................................................................71
J. ALTERNATIVE DISPUTE RESOLUTION...................................................................... 71
1. Contractual Arbitration...............................................................................................72
a. Procedure.........................................................................................................72
1) Judicial Review of Award............................................................................72
2. Judicial Arbitration......................................................................................................72
3. Mediation......................................................................................................................72
K. TRIAL..............................................................................................................................72
1. Jury Trial Problems................................................................................................72
a. Right to Jury Trial...........................................................................................73
b. Jury Trials in Diversity Cases.............................................................................73
1) Right to a Jury Trial...............................................................................73
2) Motion for New Trial Based on Excessiveness of Verdict................................74
c. Jury Size...........................................................................................................74
d. Jury Selection........................................................................................................74
1) Jury Venire...............................................................................................74
2) Voir Dire...................................................................................................74
3) Jury Challenges............................................................................................74
e. Jury Instructions..................................................................................................74
f. Jury Deliberations................................................................................................ 74
g. Jury Verdicts....................................................................................................75
1) General Verdict............................................................................................75
2) Special Verdict..............................................................................................75
3) General Verdict with Special Interrogatories....................................................75
4) Erroneous Verdicts and Inconsistent Findings..................................................75
5) Juror Misconduct......................................................................................... 76
Consolidation and Separate Trials..............................................................................76
@borbrr
CIVIL PROCEDURE-FEDERAL x1.
3.
4.

Involuntary Dismissals............................................................................................76
Voluntary Dismissals...............................................................................................76
a. Without Leave of Court..................................................................................76
b. With Leave of Court........................................................................................76
5. Offer of Judgment........................................................................................................77
6. Summary Judgment.....................................................................................................77
a. Standard...........................................................................................................77
b. Applicable to All Civil Actions.....................................................................................77
c. Time..................................................................................................................77
d. Partial...............................................................................................................77
e. Support..............................................................................................................77
f. Affidavits...........................................................................................................78
g. Nonappealability...........................................................................................................78
h. Relationship to Motion to Dismiss...............................................................................78
i. Relationship to Motion for Judgment on the Pleadings............................................78
7. Motion for Judgment as a Matter of Law (Formerly Directed Verdict)..................78
8. Renewed Motion for Judgment as a Matter of Law (Formerly Judgment
Notwithstanding the Verdict ("JNOV"))....................................................................79
9. Motion for New Trial...................................................................................................79
a. Reasons for Granting New Trial.........................................................................79
1) Remittitur.....................................................................................................79
Additur..........................................................................................................79
b. Renewed Motion for Judgment as a Matter of Law with Motion for New Trial
...........................................................................................................................80
10. Effect of Failure to Move for a Renewed Judgment as a Matter of
Law or for a New Trial............................................................................................80
11. Judgment on Partial Findings.....................................................................................80
V. ATTACK ON THE JUDGMENT AT THE TRIAL COURT LEVEL......................................80
A. RELIEF FROM JUDGMENT OR ORDER................................................................80
1. Clerical Mistakes..........................................................................................................80
2. Motions to Amend Prior Orders or Renew Prior Motions........................................80
3. Other Grounds for Relief from Judgment..................................................................80
B. INDEPENDENT ACTION IN EQUITY TO SET ASIDE THE JUDGMENT................81
VI. FINAL JUDGMENT AND APPELLATE REVIEW.................................................................81
A. JUDGMENT......................................................................................................................... 81
1. Relief that May Be Given.............................................................................................81
2. Judgment on Multiple Claims or Parties....................................................................81
3. Final Decision on Merits May Be Valid Despite Lack of Subject Matter
Jurisdiction....................................................................................................................82
B. TIME FOR APPEALS....................................................................................................82
1. Extension of Time for Appeal......................................................................................82
C. REVIEWABLE ORDER................................................................................................82
1. Interlocutory Orders as of Right.................................................................................82
a. Injunction..............................................................................................................82
b. Receivers...............................................................................................................82
c. Admiralty..............................................................................................................83
xu. CIVIL PROCEDURE-FEDERAL ti) barbrr

d. Patent Infringement........................................................................................83
e. Property Possession.........................................................................................83
2. Interlocutory Appeals Act......................................................................................83
3. Fewer than All Claims or Parties...........................................................................83
4. Collateral Order Rule.............................................................................................83
5. Review of Nonappealable Orders by Writ..................................................................83
6. Certification of Class Actions.................................................................................83
D. STANDARDS OF REVIEW..........................................................................................84
1. On Matters of Law..................................................................................................84
2. On Questions of Fact....................................................................................................84
3. On Mixed Questions of Law and Fact.........................................................................84
4. On Discretionary Matters.......................................................................................84
E. STAY PENDING APPEAL............................................................................................84
1. Execution..................................................................................................................84
2. Bond..........................................................................................................................84
3. Injunction Order.....................................................................................................84
a. Power of Trial Court.......................................................................................84
b. Power of Appellate Court....................................................................................85
F. SUPREME COURT JURISDICTION..........................................................................85
1. Court of Appeals Cases................................................................................................85
2. Cases from Highest State Court.............................................................................85
EFFECTS OF JUDGMENT ON FUTURE CASES............................................................85
A. CLAIM PRECLUSION (RES JUDICATA).................................................................85
1. Definition....................................................................................................................... 85
2. Terminology Used to Describe Effect-"Merger" and "Bar"...............................86
3. Requirements for ''Merger" and "Bar"...............................................................86
4. Valid, Final Judgment.............................................................................................86
5. ''On the Merits''.......................................................................................................86
6. Same Claimant Versus Same Defendant...............................................................86
7. ''Cause of Action''...................................................................................................87
a. Common Examples..........................................................................................87
1) Accidents...................................................................................................87
2) Installment Obligations...........................................................................87
B. ISSUE PRECLUSION (COLLATERAL ESTOPPEL)...............................................87
1. Definition..................................................................................................................87
2. Requirements...........................................................................................................88
a. Final Judgment................................................................................................88
b. Issue Actually Litigated and Determined......................................................88
c. Issue Was Essential to the Judgment.............................................................88
d. Due Process and Mutuality Considerations..................................................88
1) Against Whom Is Issue Preclusion Used?.............................................88
2) By Whom Is Issue Preclusion Used?.....................................................89
Exceptions to Mutuality When Judgment Used Defensively...............89
4) Exceptions to Mutuality When Judgment Used Offensively-
Consider Fairness....................................................................................89
C. CLAIM AND ISSUE PRECLUSION IN SPECIAL SITUATIONS..........................89
1. Judgments for Specific Performance.....................................................................89
@borbrr CIVIL PROCEDURE-FEDERAL xm.

2. In Rem Judgments........................................................................................................90
3. Quasi In Rem Judgments.............................................................................................90
D. WIDCH PERSONS ARE BOUND BY A JUDGMENT?...................................................90
1. Parties Are Bound...................................................................................................90
2. Privies to Parties Are Bound by Issue Preclusion......................................................90
3. Represented Parties May Be Bound by Claim Preclusion.........................................90
E. WIDCH CHOICE OF LAW RULES APPLY TO PRECLUSION QUESTIONS? 91 1.
Case One Decided in State Court..................................................................................91
2. Case One Decided in Federal Court Under Diversity Jurisdiction................................91
CIVIL PROCEDURE-FEDERAL 1.

CIVIL PROCEDURE-FEDERAL
INTRODUCTION
This outline is designed to acquaint you with commonly tested areas within the fields of federal juris
diction and procedure. These are: personal jurisdiction, subject matter jurisdiction, venue, discovery, and
joinder of multiple parties.

A. PERSONAL JURISDICTION
Personal jurisdiction refers to the ability of a court to exercise power over a particular defendant or
item of property. It may be categorized as in personam, in rem, or quasi in rem. The primary
limitations on a court's power to exercise personal jurisdiction are found in the United States
Constitution and state statutes.

B. SUBJECT MATTER JURISDICTION


The subject matter jurisdiction of the federal courts is limited to that authorized by the Constitution
as implemented by federal statute and decisional law. In general, it may be catego rized as follows:

1. Diversity of Citizenship Jurisdiction


Diversity of citizenship jurisdiction under 28 U.S.C. section 1332 is grounded historically in
the desire to protect out-of-state parties from local prejudice. Its main requirement is that there
be complete diversity between opposing parties. Each plaintiff must be of diverse citizenship
from each defendant. Also, the amount in controversy must exceed $75,000.

2. Federal Question Jurisdiction


Federal question jurisdiction under section 1331 presents fewer specific difficulties. The
principal problem in this area is to determine when an action "arises under" federal law. A
secondary problem is to know what types of actions are within the exclusive jurisdiction of the
federal courts under other specific statutes.

3. Removal Jurisdiction
Removal jurisdiction allows defendants to remove an action brought in a state court to a federal
court if the federal court would have had original jurisdiction over the action.

4. Supplemental Jurisdiction
The doctrine of supplemental jurisdiction is codified under section 1367 and includes, under a
single name, the concepts of "ancillary" and "pendent" jurisdiction. In any form, supple mental
jurisdiction allows a federal court to entertain certain claims over which it would have no
independent basis of subject matter jurisdiction, i.e., claims that do not satisfy diver sity or
federal question jurisdiction requirements. It is important to note that supplemental jurisdiction
operates only after a claim has invoked federal subject matter jurisdiction, after the case is
properly in federal court. Supplemental jurisdiction operates to bring additional claims into that
case that arise from the same transaction or occurrence as the original claim, but it cannot be
used to get the case into federal court in the first instance.

C. VENUE
Venue is the designation of the proper geographic district in which to bring an action. Venue will
2. CIVIL PROCEDURE-FEDERAL

depend on where the cause of action arose and on the nature of the parties (i.e., whether corporate or natural persons).
D. DISCOVERY
Discovery issues principally revolve around the scope of the examination allowed in discovery, the uses of depositions at trial,
and the available methods of enforcing discovery rights.
E. MULTIPLE PARTIES
Multiple party questions concern whether various types ofjoinder are permitted under federal law and, if so, whether there is a
jurisdictional basis for a particular attempted joinder. The majority of the issues that arise in this area are grounded in the
interpretation or application of statutes and the Federal Rules of Civil Procedure ("Federal Rules"), and also require knowledge of
subject matter jurisdictional bases, especially supplemental jurisdiction.
CIVIL PROCEDURE-FEDERAL 3.

I. PERSONAL JURISDICTION
A. OVERVIEW
There are two branches ofjurisdiction: subject matter jurisdiction and personal jurisdiction. Subject
matterjurisdiction involves the court's power over a particular type ofcase. Personal jurisdiction
involves the ability of a court having subject matterjurisdiction to exercise power over aparticular
defendant or item ofproperty. This section discusses personal jurisdiction.
1. Limitations on Personal Jurisdiction
The exercise of personal jurisdiction generally must be authorized by statute and constitu tional.

a. Statutory Limitations
States have the power to decide over whom their courts may exercise jurisdiction.
Therefore, the first place to look to determine whether the court has properly exercised
personal jurisdiction usually is state law. If no state statute grants the court the power
over the parties before the court, the court lacks personal jurisdiction.
b. Constitutional Limitations
The Due Process Clause of the Constitution places two restrictions on the exercise of
personal jurisdiction. First, the defendant must have such contacts with the forum state
that the exercise ofjurisdiction would be fair and reasonable. Second, the defendant must
be given appropriate notice of the action and an opportunity to be heard. Exercise of
personal jurisdiction over a defendant in violation of these constitutional require ments is
not valid, even if a statute purports to grant the court jurisdiction.
c. Personal Jurisdiction in Federal Courts
The main jurisdictional problem in state courts arises when the defendant over whom power
is sought lives in another state. Since the federal borders encompass all states, one might
expect that federal courts would encounter problems of personal jurisdic- tion only when
the defendants were foreign nationals. However, Rule 4 of the Federal Rules provides that,
absent some special federal statute, each federal court must analyze personal jurisdiction as
ifit were a court ofthe state in which it is located. Thus, in most cases, the assessment of
whether the court has personal jurisdiction over the defen dant will be exactly the same in
federal court as it is in state court. Rule 4 also autho rizes jurisdiction without regard to
state long arm statutes over third-party defendants and parties required to be joined under
the compulsory joinder rules, provided the party
is served within 100 miles from the place where the summons was issued. (See VII.B.3.,
infra.)
2. Three Types of Personal Jurisdiction
a. In Personam Jurisdiction
In personam jurisdiction exists when the forum has power over the person of a partic ular
defendant. (Jurisdiction over a plaintiff is generally not an issue because the plain tiff
accedes to the court's jurisdiction by bringing suit in that court.) In these cases, the court
may render a money judgment against the defendant or may order the defendant to perform
acts or refrain from acting. Such a judgment creates a personal obligation on the defendant
and is entitled to full faith and credit in all other states; i.e., if a defendant
4. CIVIL PROCEDURE-FEDERAL

is ordered to pay a sum of money to a plaintiff, the plaintiff may enforce the judgment
against the defendant's property in any other state where that property is located.

b. In Rem Jurisdiction
In rem jurisdiction exists when the court has power to adjudicate the rights of all persons
in the world with respect to a particular item of property. This jurisdiction is limited to
situations where the property is located within the physical borders of the state and where
it is necessary for the state to be able to bind all persons regarding the property's ownership
and use. This occurs with respect to actions for condemnation (eminent domain cases),
forfeiture of property to the state (e.g., when the property is used for the unlawful
transportation of narcotics), and settlement of decedents' estates.

c. Quasi In Rem Jurisdiction


One type of quasi in rem jurisdiction exists when the court has power to determine
whether particular individuals own specific property within the court's control. Unlike in rem
jurisdiction, however, it does not permit the court to determine the rights of
all persons in the world with respect to the property. A second type of quasi in rem
jurisdiction permits the court to adjudicate disputes other than ownership based on the presence of
the defendant's property in the forum (see E.2.a.2), infra, regarding appli cable constitutional
limitations).

1) Defendant Is Not Bound Personally


The basis of a court's power to exercise quasi in rem jurisdiction is the property
within the state. (See E., infra.) The judgment does not bind the defendant person ally and
cannot be enforced against any other property belonging to the defendant.

B. STATUTORY LIMITATIONS ON IN PERSONAM JURISDICTION


Each state is free to prescribe its own statutory bases for personal jurisdiction. Of course, the exercise
ofjurisdiction in a given case must also satisfy the constitutional requirements. (See C., infra .) Most
states have statutes granting their courts in personam jurisdiction in the following four situations:

(i) Where the defendant is present in the forum state and is personally served with process;

(ii) Where the defendant is domiciled in the forum state;

(iii) Where the defendant consents to jurisdiction; and

(iv) Where the defendant has committed acts bringing him within the forum state's long arm
statutes.

Each of these bases ofin personam jurisdiction will be discussed in detail below.

1. Physical Presence at Time of Personal Service


Most states grant their courts in personam jurisdiction over any defendant who can be served with
process within the borders ofthe state, no matter how long he was present (i.e., even if merely passing
through). The Supreme Court has upheld this type ofjurisdiction, allowing
a transient defendant to be served with process for a cause of action unrelated to his brief
presence in the state. [Burnham v. Superior Court, 495 U.S. 604 (1990)]

CIVIL PROCEDURE-FEDERAL 5.

State Law Exceptions to Traditional Rule


Even though jurisdiction through presence at the time of service has been upheld under the
Constitution, state statutes and court decisions have limited the power of their courts in certain
situations.

a. Service by Fraud or Force Invalid


If a plaintiff brings a defendant into a state by fraud or force to serve process, most
courts will find the service invalid. [See, e.g. , Copas v. Anglo-American Provision Co.,
73 Mich. 541 (1889)]
b. Immunity of Parties and Witnesses
Most states likewise grant immunity from personal jurisdiction to nonresidents who are
present in the state solely to take part in a judicial proceeding, or who are passing
through the state on their way to a judicial proceeding elsewhere.
3. Domicile
Most states grant their courts in personam jurisdiction over persons who are domiciliaries of
the state, even when the defendant is not physically within the state when served with process.

a. Defined
Domicile refers to the place where a person maintains her permanent home. If a person
has legal capacity, her domicile is the place she has chosen through presence (even for a
moment), coupled with the intention to make that place her home. If a person lacks
capacity, domicile is determined by law (e.g., infant is domiciliary of custodial parent's
home state).

b. Citizenship
A United States citizen, even though domiciled abroad, is subject to personal jurisdic tion
in the United States. The scope of this basis for jurisdiction is unclear, because states
have never attempted to enact laws or rules enabling their courts to obtain juris diction
solely on the basis of citizenship.

4. Consent
Virtually every state provides for in personam jurisdiction through the defendant's consent.
Such consent may be express or implied or through the making of a general appearance.

a. Express Consent
A party's express consent to the jurisdiction of local courts, whether given before or after
suit is commenced, serves as a sufficient basis for in personam jurisdiction.

1) By Contract
A person can, by contract, give advance consent to jurisdiction in the event a suit is
brought against him.

2) By Appointment of Agent to Accept Service of Process


A person can, by contract, appoint an agent in a particular state to receive service in
that state in an action against him. The terms of the contract determine the extent of
the agent's power and, thus, the scope of the jurisdiction conferred.

6. CIVIL PROCEDURE-FEDERAL

a) Appointment Required by State


When the state heavily regulates a type of business (e.g., sale of securities) to
protect its citizens, it can require a nonresident engaged in that business to appoint
an agent for service of process in the state. Note: The state cannot require every
nonresident businessperson to appoint such an agent, because the state lacks power to
exclude individuals from the state. However, a state can require nonresident
corporations to make such an appointment before doing business in the state.
b. Implied Consent
When the state has substantial reason to regulate the in-state activity of a nonresi- dent of
the state, it may provide that by engaging in such activity, the nonresident
thereby appoints a designated state official as his agent for service of process. Thus, for
example, the Supreme Court has upheld statutes that use such implied consent to subject a
nonresident motorist to jurisdiction in any state in which he has an accident. [Hess v.
Pawloski, 274 U.S. 352 (1927)]

c. Voluntary Appearance
A defendant may consent to jurisdiction by a voluntary appearance, i.e., by contesting the case
without challenging personal jurisdiction. Generally, any sort of appearance provides a sufficient
basis for jurisdiction, but many states allow "special appearances" through which a defendant
can object to the court's exercise ofjurisdiction. The defen dant usually must make this special
appearance-by stating grounds for his objection
to jurisdiction-in his initial pleading to the court; otherwise, the defendant will be deemed to
have consented to jurisdiction.

5. Long Arm Statutes


Most states also grant their courts in personam jurisdiction over nonresidents who perform or cause
to be performed certain acts within the state or who cause results within the state by acts performed
out of the state. In personam jurisdiction is granted regardless of whether the defendant is served
within or outside the forum, but is limited to causes of action arising from the acts performed or
results caused within the state.

a. Unlimited Long Arm Statutes


A few states, such as California, have long arm statutes that give their courts power over any
person or property over which the state can constitutionally exercise jurisdiction. (See C.,
infra .) These are known as unlimited long arm statutes.

b. Limited (or Specific) Long Arm Statutes


Most states, however, have long arm statutes that specify in detail the situations in which their
courts can exercise jurisdiction.

1) Limitations in Tort Cases


Some statutes permit jurisdiction when a "tort" occurs within the state, while others
require a "tortious act." The latter language has caused problems where an out-of-
state manufacturer puts his products into the stream of commerce
knowing that some items will end up in the forum state. When the gravamen of the
complaint is negligent manufacture, some courts have read "tortious act" narrowly
CIVIL PROCEDURE-FEDERAL 7.

and confined jurisdiction to the place of manufacture; others have read it to mean
"the place the tort occurred," interpreting that to be the place of injury.

2) Limitations in Contract Cases


Many statutes permit jurisdiction if the cause of action arises out of the "transac tion
of business" in the state. Some states require the defendant or his agent to have been
physically present in the state at the time the transaction took place, but others have
taken a broader view-e.g., New York has upheld jurisdiction over a California
resident who made telephone bids from California on paintings being sold in New
York.

3) Limitations in Property Actions


Many state statutes permit jurisdiction over a nonresident defendant when the cause
of action arises from ownership of property within the state-as in the case of a tort
action based on negligent maintenance of realty or a contract action
regarding the sale of the property. Some statutes include chattels, while others are
confined to realty.

4) Limitations in Marital Dissolution Cases


All states provide that when a married couple last lived together in the state and one
spouse then abandons the other, the remaining spouse may obtain personal
jurisdiction over the absent spouse for divorce or legal separation proceedings.
States vary on whether the plaintiff spouse must be living in the state at the time of
abandonment (or other cause for dissolution) or whether jurisdiction may be acquired
whenever the plaintiff has acquired domicile in the state.
C. CONSTITUTIONAL LIMITATIONS ON IN PERSONAM JURISDICTION
Once it is determined that a state has a statute that allows the court to exercise in personam juris
diction over the parties before it, the constitutionality of the exercise must next be determined.
There are two components of the constitutional aspect: contacts with the forum and notice.
1. Sufficient Contacts with the Forum
a. Traditional Rule: Physical Power
Traditionally, jurisdiction over a person (or res) was a consequence of the state's physical
power to carry out its judgment; i.e., it was based on the power to arrest the person to
force compliance with a judgment. Accordingly, the Supreme Court upheld exercises
ofjurisdiction whenever the defendant was served with process within the forum state.
[See Pennoyer v. Neff, 95 U.S. 714 (1878)] The Court later expanded the states'
physical power to extend not only to those defendants who were served within the state,
but also to those defendants who consented to the state's power or who were domiciled
in the state, regardless of where they were served.

b. Modern Due Process Standard: Contact, Relatedness, and Fairness


The concept of power by which a state could enforce its judgments was greatly
expanded by the Supreme Court in International Shoe Co. v. Washington, 326 U.S. 310
(1945). No longer was power controlled solely by whether one of the traditional
bases of presence, residence, or consent was present. Instead, the focus became whether
8. CIVIL PROCEDURE-FEDERAL

sufficient minimum contacts exist between the defendant and the forum so that mainte
nance of the suit against the defendant does not offend "traditional notions of fair play and
substantial justice." The Supreme Court has listed a series of factors by which to assess
the constitutionality of personal jurisdiction. In general, the factors fall under three
headings: contact, relatedness, and fairness.

1) Contact
International Shoe requires that the defendant have "such minimum contacts" with the
forum that the exercise ofjurisdiction would be fair and reasonable. In consid ering
whether there are such contacts, a court will look to two factors: purposeful availment
and foreseeability.

a) Purposeful Availment
Defendant's contact with the forum must result from her purposeful availment
with that forum. The contacts cannot be accidental. Defendant must reach out to
the forum in some way, such as to make money there or to use the roads there.
The court must find that through these contacts the defendant purpose fully
availed herself "of the privilege of conducting activities within the forum state,
thus invoking the benefits and protections of its laws." [Hanson v. Denckla, 357
U.S. 235 (1958)]
Examples: 1) Defendants, Michigan residents, entered into a franchise
contract with a Florida corporation. The agreement required,
among other things, that fees be sent to the franchisor's home
office in Florida, and provided that Florida law would govern
any dispute. The Court held that the defendants could be sued in
Florida; their contact with Florida resulted from their
purposeful availment of that state. [Burger King v. Rudzewicz, 471
U.S. 462 (1985)]

2) Defendant manufactures widgets in Alabama and markets


them to customers in Mississippi. Plaintiff, a resident of
Mississippi, purchases a widget from Defendant. Defendant
accepts the order and ships the widget to Plaintiff in
Mississippi. If the widget explodes and injures Plaintiff,
she can probably sue Defendant in Mississippi. Defendant
purposefully availed itself of the market in Mississippi. [See
Compare: International Shoe Co. v. Washington, supra]

1) Father, in New York, agreed to give up custody of Daughter to


Mother in California. Mother sued Father in California for
additional support. Father's only contact with California was
letting Daughter go there. The Court held that California could not
obtain in personam jurisdiction over Father because, in acting in
the interest of family harmony, Father could not be said to have
purposefully availed himself of the benefits and protections of
California laws. [Kulko v. Superior Court, 436 U.S. 84 (1978)]
CIVIL PROCEDURE-FEDERAL 9.

2) Defendant, a New York car dealer, was sued in Oklahoma


based on an injury that Plaintiff received from an accident in
Oklahoma. The only basis for jurisdiction over Defendant was
the sale of the allegedly defective car in New York by
Defendant, who knew no more than that any vehicle sold might
be driven elsewhere. The Court found that there was no
purposeful availment of the privileges or protections of
Oklahoma. [World-Wide Volkswagen Corp. v. Woodson, 444
U.S. 286 (1980)]

(1) "Stream of Commerce" Cases


There is great difficulty in assessing purposeful availment in "stream of
commerce" cases. Stream of commerce cases typically arise when
Defendant manufactures its product in State A (or even Country
A) and sells them to a second party in State B, thereby placing the
product in the stream of commerce. The product eventually winds up in
another state (State C) and causes an injury therein. The question is
whether Defendant purposefully availed itself of State C. The Supreme
Court has addressed such a scenario twice, but it has failed to reach a
consensus both times. [Asahi Metal Industry Co. v. Superior Court, 480
U.S. 102 (1987); J. McIntyre Machinery, Ltd. v. Nicastro, 564 U.S. 873
(2011)] For bar exam purposes, the important points to remember are:

(i) Merely placing an item in the stream ofcommerce, by itself, is


not a sufficient basis for personal jurisdiction.
(ii) It is unresolved whether (probably unlikely that) placing an item in
the stream of commerce with the knowledge or hope that it will
wind up in a particular state would be a sufficient basis for
personal jurisdiction. If you encounter such a question on the exam, you
should scour the question for facts showing an intentional targeting
(purposeful availment) of the forum. (See (iii), below.)
If there is no targeting, you should discuss the connections the
defendant has with the forum, and fairness to the defendant, while
mentioning the state's interest in providing a forum for its citizens and
noting that the question has not been definitively resolved. Your
conclusion will be less important than the discussion.

(iii) Placing an item in the stream ofcommerce, coupled with some


other act that shows the intent to serve a particular state (e.g.,
modifying its product to comply with state law, maintaining a sales
office within the state, etc.) is a sufficient basis for personal juris
diction.

(2) Internet Cases


The Supreme Court has not set out a specific test or standard for assessing
purposeful availment based on the defendant's Internet activity. Many courts
will look at whether the defendant has a passive website
10. CIVIL PROCEDURE-FEDERAL

that allows people to view only content, an active website that allows
people to order and download products, or something in between. The
maintenance of a website for only informational purposes, without more
activity in the forum, is insufficient to exercise jurisdiction over the
defendant for all causes of action (i.e., general jurisdiction), but it may be
sufficient for a claim arising from the maintenance of the website itself and
brought under the state's long arm statute (thus invoking specific
jurisdiction) if the defendant is specifically targeting readers in the forum.
On the other hand, maintenance of an active website alone would be
sufficient for the exercise of general jurisdiction (i.e., a claim unrelated to
the website activities) if the "at home" test is satisfied.
Specific jurisdiction hinges on whether the defendant was purposefully
directing his activities to the forum. [See, e.g., Snowney v. Harrah's
Entertainment, Inc., 35 Cal. 4th 1054 (2006)-Nevada hotel subject
to personal jurisdiction in California when it specifically targeted California
consumers by providing rate information to and accepting reservations on
its website, by touting its proximity to California, and by providing driving
directions from California]

b) Foreseeability
In addition to purposeful availment, the contact requirement of International
Shoe requires that it be foreseeable that the defendant's activities make her
amenable to suit in the forum. The defendant must know or reasonably antici
pate that her activities in the forum render it foreseeable that she may be "haled
into court" there.
Example: A national magazine is probably subject to in personam juris
diction for libel cases in every state in which the magazine is
marketed. Its publishers may reasonably anticipate causing
injury in every state in which the magazine is sold, and thus
should reasonably anticipate being haled into court in each state.
[Keeton v. Hustler Magazine, 465 U.S. 770 (1984);
Calder v. Jones, 465 U.S. 783 (1984)]

Relatedness of Claim to Contact


One important factor is whether the claim asserted against the defendant arises in
some way from the defendant's contacts with the forum. If it does, the court is
more likely to find that jurisdiction is fair and reasonable. This assessment requires the
court to determine the nature and quality of the defendant's contacts with
the state. Some authorities consider this factor to be part of the "contact" or the
"fairness" assessment; others consider it, as we do here, to be part of the "related ness"
assessment. The important point is that you address the issue in your answer, whether
under the contact, relatedness, or fairness prong of the analysis.

a) Claim Arising from Activity in the State (Specific Jurisdiction)


If the defendant's in-state activity is less than systematic or continuous (e.g.,
isolated acts), in personam jurisdiction over the defendant will be proper only for
causes of action arising from that in-state activity; i.e., the court will have
"specific jurisdiction."

CIVIL PROCEDURE-FEDERAL 11.

b) "At Home" in the State (General Jurisdiction)


General jurisdiction-in personam jurisdiction for any cause of action against the
defendant, regardless of where the cause of action arose-requires that
a home" in the state in which he is domiciled, and a corporation is "at home" in
defe the state in which it was incorporated and the state in which it has its principal
ndan place
t be of business. [Goodyear Dunlop Tires Operations, S.A. v. Brown, 564 U.S.
"at 915 (2011); Daimler AG v. Bauman, 134 S. Ct. 746 (2014)] A number of
hom states have statutes that authorize personal jurisdiction over a defendant that
e" conducts systematic and continuous activity ("doing business") within the
in state. Such "authorization" statutes must be read with this constitutional
the requirement in mind.
juris
dicti Note: Remember that the discussion here is about general jurisdiction, not
on. specific jurisdiction. If a plaintiff is injured in State A by a defendant, he may
A still sue in State A regardless of whether the defendant is "at home" in State A,
pers because there will very likely be specific (long arm) jurisdiction. Should you
on is need to discuss general jurisdiction, make sure to discuss whether the defendant
"at is "at home" in the state, as discussed above.

3) Fairness
In addition to the defendant's having relevant contacts with the forum, International
Shoe requires that the exercise ofjurisdiction not offend "traditional notions of fair play
and substantial justice." The Court has listed several factors relevant to assessing
whether jurisdiction would be fair. It is possible that an especially strong showing of
fairness might make up for a lesser amount of contact (although minimum contacts are
always required).

a) Convenience
A defendant will often complain that the forum is inconvenient. The Supreme
Court has emphasized, however, that the Constitution does not require that the
forum be the best of several alternatives. The forum is constitutionally accept
able unless it is "so gravely difficult and inconvenient that a party is unfairly put at
a severe disadvantage in comparison to his opponent." [Burger King v.
Rudzewicz, supra] This is a very difficult standard to meet, and the defendant
usually will not be able to meet it simply by showing that the plaintiff has
superior economic resources.

b) Forum State's Interest


The forum may have a legitimate interest in providing redress for its residents.
Examples: 1) Decedent, a California resident, purchased a life insurance
policy by mail from a Texas company. Decedent regularly
mailed his premiums from California to the Texas company,
which had no other contacts with California. In a suit brought by
the beneficiary of the life insurance policy, the Supreme Court
held that California had personal jurisdiction over the Texas
company. Among other things, the Court noted that California
had a strong interest in protecting its citizens from
12. CIVIL PROCEDURE-FEDERAL

alleged misfeasance by insurance companies. [McGee v. International Insurance Co.,


355 U.S. 220 (1957)]
2) Asahi, a Japanese manufacturer oftire valves, shipped valves to a
Taiwanese manufacturer of motorcycle tire tubes. The valves were incorporated
into tires and sold in California, where a resident was injured by a defective tire.
The Taiwanese manufacturer was sued in a California court, where it sought to
implead Asahi. The main case was settled, leaving only
the indemnity claim by the tire manufacturer against Asahi pending. Held:
Even though Asahi placed the defective goods in the stream of commerce knowing
that some would be used in California, exercise ofjurisdiction by the California
court would be unreasonable considering the severe burdens of Asahi in
defending in a foreign legal system, the slight interest of the Taiwanese
manufacturer and California in the exercise ofjurisdiction, and the international
interest in not subjecting an alien corporation to United States jurisdiction. [Asahi
Metal Industry Co. v. Superior Court, supra]
c) Other Factors
The Supreme Court has listed other factors relevant to the assessment of whether the exercise of
jurisdiction would be fair and reasonable, but has not discussed these factors in detail: (i) the plaintiff's
interest in obtaining convenient and effective relief, (ii) the interstate judicial system's interest in
obtaining the most efficient resolution of controversies, and (iii) the shared interest of the states in
furthering fundamental substantive social policies.
2. Notice
In addition to the requirement that the defendant have such minimum contacts with the forum to render the exercise
ofjurisdiction there fair and reasonable, due process also requires that a reasonable method be used to notify
the defendant ofa pending lawsuit so that she may have an opportunity to appear and be heard. Due
process requires that notice be "reasonably calculated, under all the circumstances, to apprise interested parties of the pendency
of the action and afford them an opportunity to present their objections." [Dusenbery v. United States, 534 U.S.
161 (2002)-quoting Mullane v. Central Hanover Bank & Trust Co., 339 U.S. 306 (1950)]
a. Traditional Methods of Personal Service Satisfy Due Process Notice Requirements Any of the
traditional methods of personal service satisfy due process notice require ments. These include personal delivery to
the defendant; leaving papers with a respon sible person at the defendant's residence or place of business; delivery to
an agent appointed to accept service; or delivery by registered mail, return receipt requested. (See VII.B., infra ,
for discussion of methods of service of process.)

b. Requirement that Agent Notify Defendant


If an agent is appointed by contract, in a case where the plaintiff chose the agent for his own benefit, or the agent is
appointed by operation of law (as under a nonresident motor
CIVIL PROCEDURE-FEDERAL 13.

vehicle statute), the failure of the agent to notify the defendant will prohibit jurisdic tion-
since the defendant will in fact be deprived of an opportunity to be heard. (This is not true
when the defendant voluntarily selects his own agent, since any failure of the agent can
and will be attributed to the principal.)

c. Requirements for Cases Involving Multiple Parties or Unknown Parties


In Mullane v. Central Hanover Bank & Trust Co., supra, an action was brought against a
number of trust beneficiaries scattered throughout the world. The Supreme Court held that
the Constitution did not require personal service on each beneficiary since the cost would
have been prohibitive. However, every beneficiary had to be notified by the best practical
means available. Thus, those whose addresses were known or could reason ably be
ascertained had to be notified by ordinary mail, while those whose names or addresses
were unknown could be notified by publication. Such methods of notice are valid only if
all defendants have substantially identical interests.

d. Knowledge that Notice by Mail Was Not Received


Although Mullane v. Central Hanover Bank & Trust Co., supra, does not require actual
notice, if a party knows that the notice by mail was not received, he may not proceed
in the face of such knowledge if practicable alternatives to apprise the defendant of the action
exist.
Example: In Jones v. Flowers, 541 U.S. 220 (2006), the state sent a certified letter
to a homeowner to inform him that he was delinquent on taxes and that
failure to pay would make his property subject to public sale. By statute,
the taxpayer was required to keep his address updated. The letter was
returned "unclaimed," after which the state took no further steps (such as
using first class mail or posting notice on the property) to notify the
taxpayer. The Court held that taking no further steps to provide notice
with the knowledge that notice had not been received violated due process.

D. IN REM JURISDICTION
As stated in I.A.2.b., supra, in rem actions adjudicate rights of all persons with respect to property
located in the state. An in rem judgment does not bind the parties personally, but is binding as to the
disposition of the property in the state.

1. Statutory Limitations
Most states have statutes providing for in rem jurisdiction in actions for condemnation, title
registration, confiscation of property (such as vehicles used to transport narcotics), forfei ture
of a vessel, distribution of the assets of an estate, and a grant of divorce when only
the complaining spouse is present and subject to personal jurisdiction. In the last case, the
"property" is the marital status of the complainant.
2. Constitutional Limitations
a. Nexus
In in rem actions the basis ofjurisdiction is the presence of the property in the state. The
state has a great interest in adjudicating the rights of all the world regarding this
property. Therefore, the presence of the property in the state is constitutionally suffi
cient for the exercise ofjurisdiction over the property.

14. CIVIL PROCEDURE-FEDERAL

1) No Jurisdiction If Property Not Located in State


A court has no in rem power over property outside the state; e.g., in settling a decedent's estate, the court
has no in rem power over property in other jurisdictions.
2) No Jurisdiction If Property Brought in by Fraud or Force
The exercise of in rem power is prohibited when the property is brought into the state by fraud or force.
b. Notice
The early view held that attachment of property, when supplemented by publication of notice in a local
newspaper or by posting of notice on the property, would give all interested persons sufficient notice of the
action. However, such procedures are no longer adequate, and the requirements of Mullane v. Central
Hanover Bank & Trust Co., supra, apply to in rem actions. Thus, persons whose interests are affected and
whose addresses are known must at least be notified by ordinary mail. [Walker v. City of Hutchinson, 352 U.S.
112 (1956)]
E. QUASI IN REM JURISDICTION
Quasi in remjurisdiction permits a court without in personam jurisdiction to determine certain disputes between a plaintiff and
defendant regarding property when the property is located in the forum state. (See I.A.2.c., supra.)
1. Statutory Limitations
There are two types of quasi in rem jurisdiction. The first type (type I) involves disputes between parties over their rights in
property within the state. The second type (type II) involves disputes unrelated to the in-state property and has been
severely limited by the Supreme Court. In quasi in rem cases, the plaintiff is unable to obtain personal jurisdiction over
the defendant, but the defendant has property in the state that the plaintiff attaches.
The court then adjudicates the dispute between the parties on the basis of its power over the property. Since the
court's sole basis ofjurisdiction is the property, anyjudgment against the defendant can be satisfied only
out ofthatproperty.
2. Constitutional Limitations
a. Nexus
Before 1977, a state clearly had power over all persons and property found within its borders. A defendant with no
other connections with the state could be sued in the state for any dispute simply because he owned property there.
However, in 1977, the Supreme Court held that the minimum contacts standard is applicable to every exercise
ofjuris diction. The Court further found that the mere presence of property within a state is not itself sufficient to
permit a court to exercise quasi in rem jurisdiction over property in a quasi in rem action. [Shaffer v. Heitner,
433 U.S. 186 (1977)]
1) Quasi In Rem 'fype I
Thus, when the dispute involves the rights ofthe parties in the property itself (quasi in rem type I),
jurisdiction based upon the presence of the property in the state is proper. The close connection between
the litigation and the property provides the necessary minimum contacts.

2) CIVIL PROCEDURE-FEDERAL 15.

Quasi In Rem Type II


When the dispute is unrelated to the ownership of property (quasi in rem type II),
jurisdiction cannot be based solely on the presence of property in the forum state;
there must be minimum contacts between the defendant and the forum. However, if
the defendant has minimum contacts with the forum, it is also likely that a court
could exercise in personam jurisdiction over a defendant under the forum's long
arm statute, thus removing the limit on recovery to the defendant's in-state property.
As a result, use of quasi in rem jurisdiction type II will be rare.
3) Example: A, a Maine resident, flies to Ohio and enters into a contract with B, an
Ohio resident. All performance is to occur in Ohio. A flies home. B
breaches. A does not want to fly to Ohio to sue B, but he
discovers that B has a boat docked in Maine. Traditionally, A could
have sued on his contract claim in Maine by attaching the boat (his
remedy being limited by the value of the boat). Today, he would have
to show minimum contacts between B and Maine.

Procedural Requirements
To obtain quasi in rem jurisdiction, a plaintiff must "bring the asset before the court"
by attachment (or garnishment). This will inhibit the sale or mortgage of the
defendant's interest, since a new owner must take subject to the decision of the court.
Serious whether such a pretrial interference with a defendant's property rights is constitutional
questions unless the defendant is afforded a hearing on the necessity of such procedures. Most
have been commentators think the process is valid, but the Supreme Court has thus far avoided
raised as to the issue.

b. Notice
As in in rem cases, quasi in rem cases require the best practical notice. Therefore, posting
of notice or notice by publication will be insufficient where the addresses of persons
affected by the action are known or reasonably ascertainable. The federal statute for the
enforcement of liens or other claims to real or personal property requires personal service
if practicable and service by publication if personal service is not practicable. If the
defendant is not personally served, he may appear within one year of final judgment, and
the court must set aside the judgment on payment of costs as the court deems just. [28
U.S.C. § 1655]

II. DIVERSITY OF CITIZENSHIP JURISDICTION


A. INTRODUCTION
The federal courts have been given subject matter jurisdiction over controversies between citizens of
different states, even though the controversies do not involve questions of federal substantive law, in
order to protect an out-of-state party from possible local bias in state courts.
B. DIVERSITY AMONG THE PARTIES
1. Complete Diversity When Action Is Commenced
16. CIVIL PROCEDURE-FEDERAL

a. Multiple Parties-Complete Diversity


Diversity jurisdiction requires "complete diversity," meaning that no plaintiff may be a
citizen of the same state as any defendant. If one defendant and one plaintiff are co-
citizens of the same state, complete diversity is lacking and there is no diversity
jurisdiction.
Example: A, B, and C bring an action against X, Y, and Z. A and B are citizens of
New York; X and Y are citizens of Florida; and C and Z are citizens of
Texas. Since no diversity exists between C and Z, the requirement of
complete diversity is not satisfied, and, as structured, the case cannot be
brought in federal court under diversity jurisdiction.
1) But Note
The rule of complete diversity does not require that every party be of diverse citizenship
from every other party. It requires only that no plaintiff be a co-citizen with any
defendant. Thus, two plaintiffs who are both citizens of Missouri may invoke diversity
of citizenship jurisdiction against three defendants, all three of whom are citizens of
Kansas.
2) lnterpleader Exception
a) Federal lnterpleader Statute-Minimal Diversity
The federal interpleader statute [28 U.S.C. § 1335] requires only that among the
parties there be "two or more adverse claimants, of diverse citizenship." Thus,
"minimal diversity" is sufficient to conferjurisdiction under the statute. If there
is diversity between any two ofthe claimants, all other claimants may be citizens
of the same state. (Also, section 1335 only requires that the money or property at
issue be valued at $500 or more.)
b) Interpleader Under Federal Rules-Complete Diversity
Interpleader pursuant to Rule 22 of the Federal Rules, on the other hand,
requires the usual diversity between all the plaintiffs (stakeholders) and all the
defendants (claimants).
b. "Alienage" Jurisdiction
Most bar exam questions in this general area involve basic diversity of citizenship
jurisdiction, in which the dispute involves "citizens of different states," as discussed
immediately above. However, section 1332(a)(2) grants subject matter jurisdiction over
"alienage" cases, in which the dispute is between a citizen of a U.S. state and an
"alien"-meaning a citizen or subject of a foreign country. Jurisdiction is denied,
however, if the case is between a citizen of a state and a citizen of a foreign country who has
been admitted to the United States for permanent residence and domiciled in the same state as
the U.S. citizen. Also note that the U.S. Constitution does not provide for federal jurisdiction
over cases by an alien against an alien; there must be a citizen of a
U.S. state on one side of the suit to qualify for alienage jurisdiction.
Examples: 1) A, a citizen of Venezuela, sues B, a citizen of New York. This dispute
would invoke alienage jurisdiction (assuming the amount in controversy
requirement was also met), because it is between a citizen of a state and a
citizen of a foreign country.
CIVIL PROCEDURE-FEDERAL 17.

2) A, a citizen of Venezuela, sues B, a citizen of France. This dispute


would not invoke alienage jurisdiction, because it is not between a citizen
of a state and a citizen of a foreign country. There is no citizen of a state
involved here.

3) A, a citizen of New York, sues B, a permanent resident alien domiciled


in New York. Alienage jurisdiction would be denied because B has the
same U.S. domicile as A.

1) Aliens as Additional Parties


28 U.S.C. section 1332(a)(3) grants jurisdiction in a case between citizens of
different states in which citizens or subjects of a foreign country are additional parties.
The foreign parties are disregarded for jurisdictional purposes. The restric tion in
example 2) above does not apply-it applies only to section 1332(a)(2) actions.
Although not entirely clear from case law, there appears to be no subject matter
jurisdiction when there are U.S. citizens on one side ofthe action and aliens on both
sides.

c. Diversity When Action Is Commenced


Diversity of citizenship (or alienage) must exist as of the time the suit is instituted. [Grupo
Dataflux v. Atlas Global Group, 541 U.S. 567 (2004)] It need not exist at the time the
cause of action arose, and it is not defeated if, after commencement of the action, a party
later becomes a citizen of the same state as one of his opponents.
2. Questions of Citizenship
a. State Citizenship of an Individual-Domicile
The determination of the state of citizenship of a natural person depends on the perma nent
home to which he intends to return. The concept is the same, except in name, as domicile.
A new state citizenship may be established by (i) physicalpresence in a new place and (ii)
the intention to remain there, i.e., no present intent to go elsewhere. The citizenship of a
child is that of her parents. In most cases, the citizenship of a party will be determined by
the court, but it may be left to the jury.

b. Citizenship of a Corporation-Possible Multiple Citizenships


For diversity purposes, a corporation's citizenship is defined by 28 U.S.C. section 1332.
Under this statute, a corporation is deemed to be a citizen of every state and foreign country
in which it is incorporated and the one state or foreign country in which it
has its principal place of business. The Supreme Court has held that a corporation's
"principal place of business" is the state from which the corporation's high level officers
direct, control, and coordinate the corporation's activities (i.e., its "nerve center," which will
usually be the corporation's headquarters). [Hertz Corp. v. Friend, 559 U.S. 77 (2010)]
Thus, many corporations have two citizenships-their state of incorporation and the state in
which their principal place of business is located. Although rare, it
also is possible for a corporation to have more than two state citizenships because a
corporation may be incorporated in more than one state. It is impossible, however, for a
corporation to have more than one principal place of business. If an opposing party is a
citizen of any of the corporate party's states of citizenship, there is no diversity.

18. CIVIL PROCEDURE-FEDERAL

1) Special Rule for Direct Actions


The rules of corporate citizenship are subject to a special rule in direct action cases. When a plaintiff
sues an insurer on a policy or contract of liability insur ance, and does not also join the insured, the
insurer (whether incorporated or not) is treated as a citizen of all of the following: (i) the state or foreign
country in which the insurer is incorporated (if it is), (ii) the state or foreign country in which the insurer
has its principal place of business, and (iii) the state or foreign country of which the insured is a citizen.
2) Incorporation or Principal Place of Business in Foreign Country
Because a corporation is a citizen of both its place of incorporation and its principal place of business,
and either of those places may be in a foreign country, a corporation might simultaneously be an alien and a
citizen of a U.S. state. The fact that a corporation is an alien will defeat jurisdiction in a suit involving solely
another alien, even though the corporation may also have U.S. state citizenship. (See Lb., supra.)
c. Unincorporated Associations
1) Citizenship
In claims based on federal law for or against an unincorporated association, the association has entity
capacity, but the question of its citizenship is normally irrelevant because the court will have federal question
jurisdiction. When diversity jurisdiction is involved, an unincorporated association:
(i) May sue or be sued in its own name if local state law so permits; or
(ii) Is an aggregate of individuals if local state law follows the common law rule.
In either case, the unincorporated association's citizenship is that of each and every one of its
members.
2) Class Action
If the association is large, a class action is possible. If a class action is brought, the relevant citizenship is
that of the named members who sue or are sued on behalf of the members of the association. (See f.,
infra.)
3) Partnerships
The citizenship of a general partnership is that of each and every general partner, and the citizenship of a
limited partnership is that of each and every partner, both limited and general. [Carden v. Arkoma
Associates, 494 U.S. 185 (1990)]

4) Limited Liability Companies


Although limited liability companies ("LLCs") are formed in a manner similar to corporations, they are
treated as unincorporated associations for citizenship
purposes. Thus, an LLC is a citizen of all states of which its members are citizens. [See, e.g., Belleville
Catering Co. v. Champaign Market Place LLC, 350 F.3d 691 (7th Cir. 2003)]
CIVIL PROCEDURE-FEDERAL 19.

d. Business Trusts
The trustees of a business trust are the real parties in interest and their citizenship, not
that of the individual shareholders, determines whether there is diversity. [Navarro Savings
v. Lee, 446 U.S. 458 (1980)]

e. Legal Representatives
A legal representative of an infant, an incompetent, or an estate of a decedent is deemed to
be a citizen of the same state as the infant, incompetent, or decedent.

f. Class Actions
If suit is brought by several named persons on behalf of a class under Rule 23, diver sity
is determined on the basis of the citizenship of the named members of the class who are
suing. Thus, there is considerable room for maneuvering to create diversity if the class has
members who are citizens of several different states. The Class Action Fairness Act (see
VII.G.2.d., infra) also expands the federal court's jurisdiction over class actions.

g. Nonresident United States Citizens


A United States citizen domiciled abroad is not a citizen of any state and also is not an
alien. (Alien status depends on nationality, not domicile.)

3. Collusion and Devices to Create or Defeat Diversity


The federal court does not have jurisdiction if a party "by assignment or otherwise, has been
improperly or collusively made or joined to invoke jurisdiction." [28 U.S.C. § 1359]

a. Assignment of Claims
The assignment of a claim to another party for collection only is clearly within this
section. [Kramer v. Caribbean Mills, Inc., 394 U.S. 823 (1969)] Thus, the assignment
would be ignored in determining whether diversity exists. But note: There is no collu sion
if an absolute assignment of a claim is made and the assignor retains no interest in the
assigned claim.

b. Class Actions
No rule prevents achieving diversity by the adroit selection of named plaintiffs to bring a
proper class action on behalf of others. The naming of only members of the class who are
not co-citizens of the defendants will create diversity even though other unnamed members
of the class are co-citizens who would defeat diversity if named.

c. Voluntary Change of State Citizenship


A plaintiff can create diversity by changing his state citizenship after the cause of action
accrued but before suit is commenced, but the change must be genuine. In other words, a
true change of citizenship can create or destroy diversity. The party's motive for changing
citizenship is irrelevant.
4. Realignment According to Interest
a. May Create or Destroy Diversity
In determining whether diversity exists, the court will look beyond the nominal
20. CIVIL PROCEDURE-FEDERAL

designation of the parties in the pleadings and realign them according to their true inter ests in
the dispute. Thus, realignment may create diversity or destroy it.
b. Shareholder Derivative Actions
Taking the view that the shareholder's alignment of the corporation as a party plaintiff or
defendant is not controlling insofar as diversity jurisdiction is affected by the citizen ship of
the corporation, the federal courts have established the rule (at least when align ment of
the corporation in a shareholder's derivative suit is not specifically provided for by state law)
that the corporation is to be aligned as a party defendant. Federal diver sity jurisdiction is
determined in accordance with that alignment when, with respect to the claim sought to be
enforced by the shareholder's derivative suit, the corporation is "antagonistic" to the
shareholder. [See Smith v. Sperling, 354 U.S. 91 (1957)]

5. "Supplemental" Jurisdiction
Occasionally, a claim may be joined that could not, by itself, invoke federal question jurisdic
tion or diversity jurisdiction (because, for example, it is a state claim between parties who are
citizens of the same state or because it does not involve the requisite amount in controversy).
(See C., infra .) The federal court may nonetheless entertain such claims under its supple mental
jurisdiction. (This type of supplemental jurisdiction used to be known as "ancillary
jurisdiction.") Supplemental jurisdiction requires that the supplemental claim arise from a common
nucleus of operative fact as the claim that invoked original (diversity or federal question, usually)
federal subject matter jurisdiction. Some courts consider the common nucleus test to mean that
the claims must arise from the same transaction or occurrence, but the growing trend is that the
common nucleus test is broader than that.
6. Joinder or Subsequent Addition of Parties
The Federal Rules permit numerous methods by which multiple claims or parties may be joined
or added to the case. A claim by or against such a party, like any claim in federal court, must
satisfy some basis of federal subject matter jurisdiction, such as diversity of citizenship or federal
question. If the claim does not satisfy either of those, and it arises from a common nucleus of
operative fact (see above), the party asserting the claim might invoke supplemental jurisdiction.
Joinder of claims or parties is also discussed in VII.G., infra.
a. Restriction on the Use of Supplemental Jurisdiction in Diversity Cases
For cases that are in federal court based solely on diversity, supplemental jurisdiction
may not be used to support:
(i) Claims by plaintiffs against persons made parties under Rules 14 (impleader), 19
(compulsory joinder), 20 (permissive joinder), or 24 (intervention);
(ii) Claims by persons proposed to be joined as plaintiffs under Rule 19; and
(iii) Claims by persons seeking to intervene as plaintiffs under Rule 24;
When the exercise of supplemental jurisdiction would be inconsistent with the require ments for
complete diversity.
[28 U.S.C. §1367(b)]
CIVIL PROCEDURE-FEDERAL 21.

b. Intervention of Right
Intervention of right is given under Rule 24(a) where the intervenor claims an interest
relating to the property or transaction that is the subject of the action and the disposition of
the action may adversely affect that interest. Traditionally, intervention of right has not
required any showing of independent jurisdiction; the intervenor's claim was consid ered to
be within the court's supplemental (ancillary) jurisdiction if the requirements
for intervention of right were met. Under the supplemental jurisdiction statute, however,
there is no supplemental jurisdiction for claims by or against intervenors. Thus, such a
claim could proceed only if there were an independent basis ofjurisdiction, e.g., diver sity
or federal question.
c. Permissive Intervention
Under Rule 24(b), permissive intervention may be permitted in the court's discretion when
the intervenor's action and the main action have a claim or defense involving a common
question oflaw orfact. The claim by a permissive intervenor must invoke either diversity
of citizenship or federal question jurisdiction.
d. Substitution of Parties
Substitution under Rule 25 involves changes in parties to a lawsuit necessitated by death,
incompetency, etc., of an original party after an action has been commenced. The
citizenship of the substituted party is disregarded; that of the original party controls.
Substitution should be distinguished from an amendment that allows "replacement" of
an original party by the party in whom or against whom the action properly lies. A
"replacement" party must be diverse to the party or parties on the opposing side.
Example: A v. B. A dies and the administrator of his estate is substituted as plain tiff.
Jurisdiction is not destroyed even though B and the administrator are co-
citizens. However, if A sues B and subsequently discovers that C-not B-is
the proper defendant, an amendment to the complaint by which B is
replaced by C must show that diversity exists between A and C.
e. Third-Party Practice-Impleader
A third-party claim under Rule 14 is the joinder by the defendant in the original action
(who is usually called the third-party plaintiff) of another person not originally a party to
the action (who is called the third-party defendant) . The impleader claim asserts that the
third-party defendant is or may be liable to the defendant for all or part of the plain tiff's
claim against the defendant. In other words, an impleader claim is for indemnity or
contribution.
Example: P sues D for $500,000 for personal injuries allegedly inflicted by joint
tortfeasors D and X. Applicable law provides thatjoint tortfeasors have a
right of contribution against each other. D may implead X into the pending
case. D is seeking to deflect her liability on P's claim, in part, to
X. (If X owed D indemnity for some reason, then D could implead X to
deflect her entire liability on the underlying claim to X.)
Under Rule 14, after the third-party defendant is impleaded, he may assert a claim
against the plaintiff in the pending case if the claim arises from the same transaction or
occurrence as the underlying suit. In addition, under Rule 14, after the third-party
defendant is impleaded, the plaintiff may assert a claim against him if it arises from the same
transaction or occurrence as the underlying suit.
22. CIVIL PROCEDURE-FEDERAL

1) Subject Matter Jurisdiction Required


Of course, every claim asserted in federal court must have a basis of subject matter
jurisdiction.
Examples: 1) P, a citizen of Illinois, sues D, a citizen of Wisconsin, asserting a
state law claim of more than $75,000. Thus, the case invokes diver
sity of citizenship jurisdiction and is properly brought in federal court.
Now D impleads X, who is also a citizen of Illinois, on an indemnity
claim of more than $75,000. That claim invokes diver sity of
citizenship jurisdiction, because it is asserted by a citizen
of Wisconsin (D) against a citizen of Illinois (X) and exceeds
$75,000. The fact that P is also a citizen of Illinois is irrelevant; the
claim is not by or against her, so her citizenship does not affect the
impleader claim. If P wanted to assert a claim against X in this
situation, however, there would not be diversity because P and X are
co-citizens of Illinois. In addition, the claim would not invoke
supplemental jurisdiction, because in diversity of citizenship cases, the
supplemental jurisdiction statute cannot be used to override
the complete diversity rule. Thus, unless the claim by P against X
invoked federal question jurisdiction, it could not be asserted in the
pending case; it would have to be asserted in state court.

2) P, a citizen of Alabama, sues D, a citizen of Maine, asserting a


state law claim of more than $75,000. Thus, the case invokes
diversity of citizenship jurisdiction and is properly brought in
federal court. Now D impleads X, who is also a citizen of Maine,
on a state law contribution claim. The impleader claim does not
invoke diversity of citizenship jurisdiction, because it is asserted by a
citizen of Maine (D) against another citizen of Maine (X). It does not
invoke federal question jurisdiction because it is based on state law.
The claim invokes the ancillary form of supplemental jurisdic tion,
however, because it arises from a common nucleus of operative fact as
the underlying case and is asserted by the defendant, not the plaintiff,
thus avoiding the restriction on the use of supplemental jurisdiction in
28 U.S.C. section 1367(b).

f. Cross- Claims
Rule 13 (g) allows a party to assert a claim in a pending case against a co-party, but only
if the claim arises from the same transaction or occurrence as the underlying dispute. So,
in a lawsuit of A v. B and C, a claim by B against C (or C against B) that arises from the
same transaction or occurrence as the underlying case would be a cross-claim.

1) Subject Matter Jurisdiction Required


Cross-claims, like all claims in federal court, must invoke subject matter jurisdic tion.
Therefore, after determining that a cross-claim would be filed, assess whether that claim
could invoke diversity of citizenship or federal question jurisdiction.
If so, the claim may be asserted in federal court. However, if a cross-claim does not
invoke diversity of citizenship or federal question jurisdiction, the cross-claim
CIVIL PROCEDURE-FEDERAL 23.

could nonetheless be asserted in federal court through supplemental (ancillary)


jurisdiction.
C. JURISDICTIONAL AMOUNT: IN EXCESS OF $75,000
Actions brought in a federal court under the diversity statute must meet the jurisdictional amount
requirement. The matter in controversy must be in excess of$75,000, exclusive of interest and costs.
[28 U.S.C. § 1332] The amount is determined from what is claimed in the complaint, disre garding
potential defenses or counterclaims. Usually, all that is necessary is a goodfaith allegation that the
amount of the damages or injuries in controversy exceeds, exclusive of interest and costs, the sum of
$75,000. Good faith means that there must be a legally tenable possibility that recovery will exceed
the jurisdictional amount. The complaint can be dismissed only if it appears there is no legal possibility
of a recovery exceeding the jurisdictional amount. Jurisdiction is not retroactively defeated by the fact
that the amount actually recovered is less than the jurisdictional amount.
What Is "In Controversy"?
a. Collateral Consequences of the Judgment
Does the collateral effect of the judgment sought by the plaintiff bring into controversy the
value of other claims that may be governed by the judgment? The Supreme Court has held
that the collateral effects of ajudgment may not be considered.
Examples: 1) If an insured asserts a claim for installments due under a disability
policy, only the installments due may be considered, even though the
judgment may control the insured's rights to payment of future install ments.
However, if the insurance company sues to cancel the contract for fraud,
the value of the entire contract is brought into controversy.
2) If a bondholder sues to collect amounts due on coupons that have
matured, only the amount of the coupons is in controversy, even though the
judgment will determine the validity of the entire bond issue. However, if
the issuer of the bonds seeks a declaratory judgment that the bonds are
properly issued, the value of the entire bond issue would be in controversy.

b. Interest and Costs


The statute excludes interest and costs in determining the jurisdictional amount. However,
attorneys' fees that are recoverable by contract or by statute are considered part of the
matter in controversy rather than as costs. Similarly, interest that constitutes a partofthe
claim itself, as distinguished from interest payable by virtue of a delay in payment, is part
of the jurisdictional amount.
Example: Plaintiff sues on a three-year note with face value of $70,000 and
interest at 5% (an additional $10,500). Since the interest on the note is
part of the claim, the jurisdictional amount is satisfied. [See, e.g., Brainin
v. Melikian, 396 F.2d 153 (3d Cir. 1968)] But if the interest accrued
between maturity and filing, the additional interest after maturity is not
part of the claim.
c. Equitable Relief
There may be difficulty calculating an amount in controversy for a claim for equitable
relief, given that the claimant does not seek money damages. For example, suppose P
24. CIVIL PROCEDURE-FEDERAL

sues D for an injunction ordering D to remove part of D's house that blocks P's view. What
is the value of the injunction and, therefore, the claim? Some courts look at the issue from
the plaintiff's viewpoint, and ask what the value of the harm caused by the blocked view is.
Other courts look at the issue from the defendant's viewpoint, and ask what it would cost
the defendant to comply with the injunction if it were ordered. Some courts conclude that
the amount in controversy requirement is satisfied if the amount under either test-plaintiff's
viewpoint or defendant's viewpoint-exceeds $75,000.
d. Punitive Damages
If a punitive damage claim is permitted under state substantive law, it may be used in
making the dollar amount requirement because there is "no legal certainty" that the
amount will not be recovered.
Aggregation of Separate Claims
a. One Plaintiff Against One Defendant
For purposes of meeting the jurisdictional amount, the plaintiff may aggregate all her
claims against a single defendant. This aggregation is permitted regardless of whether the
claims are legally or factually related to each other.

b. One Plaintiff Against Several Defendants


A plaintiff who has an action against several defendants cannot aggregate claims based on
separate liabilities. Thus, if P had a claim of $50,000 against D-1 and a separate claim of
$30,000 against D-2, she may not aggregate those claims. Note, however, that there is no
aggregation problem if plaintiff asserts a joint claim against multiple defen dants. With a
joint claim, courts look to the total value of the claim.
Example: P sues alleged joint tortfeasors X, Y, and Z for damages of $76,000. This
claim satisfies the amount in controversy requirement. Because this is a
claim based onjoint liability, any of the three defendants might be held
liable for the total amount of the claim. This is not a case of trying to
aggregate three separate claims. Because ofjoint liability, courts see this as
one claim. Because it exceeds $75,000, it meets the amount require ment.

c. Several Plaintiffs Against One Defendant


Several plaintiffs can aggregate their claims only where they are seeking "to enforce a
single title or right in which they have a common or undivided interest............."
Example: Ifjoint owners of real estate file suit to quiet title, the right asserted is
held jointly and the amount in controversy is the total value of the
land. However, if several victims of the same accident sue for personal
injuries, their claims are separate and distinct from one another and
aggregation is not allowed.

This rule has special importance in class actions, in which the rule is that the claims of the
class members cannot be aggregated if their rights are "separate" rather than 'joint" or
"common." One class representative's claim must exceed $75,000, and the court will have
supplemental jurisdiction over the claims that do not exceed $75,000. [Snyder v.
Harris, 394 U.S. 332 (1969)]
CIVIL PROCEDURE-FEDERAL 25.

3. Supplemental Jurisdiction over Claims Not Exceeding $75,000 in Diversity Cases


Claims that do not meet the amount in controversy requirement for diversity of citizenship
jurisdiction may invoke supplemental jurisdiction if they arise from a common nucleus of
operative fact as a claim that invoked diversity of citizenship. However, the supplemental
jurisdiction cannot be used to override the complete diversity rule. [Exxon Mobil Corp. v.
Allapattah Services, 545 U.S. 546 (2005)]
Example: P-1, a citizen of California, asserts a claim for $ 100,000 against D, a citizen of
Arizona. That claim invokes diversity of citizenship jurisdiction. In the same case,
P-2 asserts a claim against the same D for $50,000. The claims by P-1 and P-2
arise from the same transaction and are based on state law. The claim by P-2
cannot invoke diversity of citizenship jurisdiction because it does not exceed
$75,000. Nonetheless, the claim by P-2 can be heard in federal court under
supplemental jurisdiction.

Compare: P-1, a citizen of California, asserts a claim for $ 100,000 against D, a citizen
of Arizona. That claim invokes diversity of citizenship jurisdiction. In the
same case, P-2, a citizen of Arizona, asserts a claim against the same D for
$100,000. The claim by P-1 and the claim by P-2 arise from the same transac
tion and are based on state law. The claim by P-2 cannot invoke diversity of
citizenship jurisdiction because it is by a citizen of Arizona against a citizen of
Arizona, and supplemental jurisdiction cannot override the complete diver sity
requirement. Thus, that claim may be asserted only in state court.

4. Counterclaims
A defendant's counterclaim [see Fed. R. Civ. P. 13] cannot be combined with the plaintiff's claim
to reach the jurisdictional amount; e.g., if the plaintiff claims $20,000, there is no jurisdictional
amount even though the defendant counterclaims for $100,000. Does a counter claim itself have to
meet the requirements of the jurisdictional amount?
a. Compulsory Counterclaim Need Not Meet Jurisdictional Amount
A compulsory counterclaim (arising out of the same transaction or occurrence) does not
need to meet the jurisdictional amount requirement. The court has ancillary (supple mental)
jurisdiction over such a counterclaim just as it does over a third-party claim under Rule 14
impleader.

b. Permissive Counterclaim Must Meet Jurisdictional Amount


A defendant's permissive counterclaim (arising out of a completely unrelated trans
action) must have an independent jurisdictional basis, and thus must meet thejuris
dictional amount requirement. A growing body of case law holds that supplemental
jurisdiction is available if there is some sort of factual relationship between the two
claims.

c. No Removal to Federal Court Based on Counterclaim


A plaintiff who claims $75,000 or less in a state court action who is met with a counter
claim for more than $75,000 may not remove the suit to federal court, regardless of
whether the counterclaim is compulsory or permissive, because removal is permitted only
to defendants. The weight of authority also holds that in a situation where the plaintiff has
not met the jurisdictional amount, the defendant who must assert a
26. CIVIL PROCEDURE-FEDERAL

compulsory counterclaim in the state suit may not remove the action, even though the
counterclaim is over $75,000 and there is complete diversity. Thus, a plaintiff with a small
claim can require a defendant with a large claim to litigate it in state court simply by being
the first to file. But note: Even though this is the traditional rule, there is a trend
allowing removal.
D. ERIE DOCTRINE AND THE LAW APPLIED UNDER DIVERSITY JURISDICTION A
federal court, in the exercise of its diversity jurisdiction, is required to apply the substantive law
ofthe state in which it is sitting, including that state's conflict of law rules. [Erie Railroad v.
Tompkins, 304 U. S. 64 (1938); Klaxon Co. v. Stentor Electric Manufacturing Co., 313 U.S. 487
(1941)] However, the federal courts applyfederalprocedural law.
1. Federal Statutes or Federal Rules of Civil Procedure
To determine whether federal law should be applied, the first question to ask is whether there is
a federal law (e.g., statute, Federal Rule of Civil Procedure) on point. If there is, the federal
statute or federal rule will apply, provided that it is valid. (Since the Supreme Court reviews and
sends proposed rules to Congress prior to enactment, it is very unlikely that a Federal Rule
would be held invalid.) Of course, if there is a federal constitutional provision (e.g., right to jury
trial in cases over $20), it applies.
Example: Federal Rule 4 permits substituted service of process. Suppose that state law
(of the state in which the federal court sits) does not permit substituted
service. The court will apply the Federal Rule, because it is on point and is
valid. A Federal Rule of Civil Procedure is valid if it is "arguably proce
dural." [Hanna v. Plumer, 380 U. S. 460 (1965)]
a. Caution
Sometimes it is difficult to determine whether a federal statute or rule is on point. For
example, Federal Rule 3 provides that a case is commenced when the complaint is filed.
Many people thought that the rule thus was a directive that the statute of limita tions
would be tolled from the date of filing the complaint. The Supreme Court held, however,
that Rule 3 did not address tolling at all, and thus did not constitute a federal directive on
the tolling question. [Walker v. Armco Steel, 446 U.S. 740 (1980)]
b. Recent Application
In Shady Grove Orthopedic Associates v. Allstate Insurance Co., 559 U.S. 393 (2010), a
plaintiff brought a diversity jurisdiction class action under a New York law for recovery of
interest on claims paid late by insurance companies. A New York statute would have denied
class action status to claims seeking such recovery. A majority of the Court
held that Federal Rule of Civil Procedure 23 governed regarding class action status
and refused to apply the New York statute. Under the Rules Enabling Act, a Federal
Rule is valid if it deals with "practice or procedure" and does not "abridge, enlarge,
or modify" a substantive right. A majority in Shady Grove concluded that Rule 23 is
valid. Only four Justices concluded, however, that a Rule's validity is determined
solely by refer ring to its terms (ignoring the state provision) and asking whether it is
"arguably proce dural." If so, according to those four Justices, the Federal Rule is
valid.
2. If There Is No Federal Statute or Rule on Point, Is the Issue Substantive or Procedural?
If there is no federal statute or rule on point, can a federal judge refuse to follow state law
CIVIL PROCEDURE-FEDERAL 27.

on a particular issue? The answer depends on whether the law on that issue is substantive or
procedural. If it is a matter of substance, the federal judge must follow state law in a diversity case.
If it is a matter of procedure, the federal judge may ignore state law.
Example: In medical malpractice actions, some states require that an affidavit addressing
the legitimacy of the claim from a qualified expert be attached to the complaint.
Must such an affidavit be attached to a complaint filed in federal court based
on diversity? Step one would be determining whether there is a federal
directive on point. (There is no such directive.) Step two is
determining whether the affidavit requirement is substantive or procedural.
a. Some Situations Are Clearly Established
In some instances, the characterization as substance or procedure is well established. For
example, the Supreme Court has established that statutes of limitations and rules for tolling
statutes oflimitations are substantive for Erie purposes; therefore, a federal judge in a
diversity case must follow state law on those issues. [Guaranty Trust Co. v.
York, 326 U.S. 99 (1945)] Choice oflaw rules are also substantive for Erie purposes,
and a federal judge in a diversity case must follow state law on that issue as well. [Klaxon
v. Stentor Electric Manufacturing Co., 313 U.S. 487 (1941)] Finally, of course, elements
ofa claim or defense are substantive.
Note: There can be two issues presented in a conflict of laws problem. The first issue
involves determining whetherfederal or state law applies. The second issue involves
determining which state's laws apply. Both questions are often resolved using "substan
tive" and "procedural" language, but a determination that a law is "substantive" for the
federal vs. state question does not mean that has to be "substantive" for the state vs. state
question.
b. Law Is Unclear in Other Situations
Outside these areas, when there is no federal directive on point, it is often difficult to
determine whether an issue is substantive or procedural for Erie purposes. The
Supreme Court has given different "tests" at different times on this point, and has
failed to integrate the tests comprehensively. One such test is outcome determination,
which holds that an issue is substantive if it substantially affects the outcome of the case.
[Guaranty Trust Co. v. York, supra] Another test is balance ofinterests, in which the court
weighs whether the state or federal judicial system has the greater interest in having its rule
applied. [Byrd v. Blue Ridge Electric Cooperative, Inc., 356 U.S. 525 (1958)] Yet another
test is forum shopping deterrence, which directs that the federal judge should follow state
law on the issue if failing to do so would cause litigants to flock to federal court. [Hanna
v. Plumer, supra]
3. Statutes Involving Both Substance and Procedure
Sometimes, a state statute or rule may be both substantive and procedural. In one case, the
state tort reform law relaxed the standard for granting a new trial, making it easier to grant a
new trial than under the basic federal standard. Also, the state appellate court was charged with
the responsibility to consider whether a new trial should be ordered. In a diversity case under
this state law, the standard for granting a new trial was held to be substantive, so the federal
court had to apply the state standard for granting a new trial. However, the require ment that the
appellate court consider whether a new trial should be ordered was held to be procedural, so a
federal trial court would determine whether a new trial should be ordered,
28. CIVIL PROCEDURE-FEDERAL

using the aforementioned state standard, rather than an appellate court. [Gasperini v. Center for
Humanities, Inc., 518 U.S. 415 (1996)-in diversity case, federal trial court applied New York
"excessive damages" standard for new trial rather than federal "shock the conscience" standard]

4. Interpreting State Law


The federal court is bound to apply the substantive state law that would be applied by the
highest court of the state. If the state courts have not decided the issue that is before the federal
court, or if the decisions on point are old and no longer current with the decisions of other
jurisdictions, the federal court may consider the law of other jurisdictions in reaching its
decision. However, the focus of the federal court is to determine what decision the highest
court ofthe state would reach if confronted with the issue.

a. De Novo Review of District Court's Decision


On appeal, the federal appellate court reviews the federal trial judge's decision as to state
law de novo. [See Salve Regina College v. Russell, 499 U.S. 225 (1991)]

b. Subsequent State Court Decisions


If the highest state court renders a decision on an issue after the federal court has made its
determination, the decision of the district court may be changed to conform to the new
decision of the highest state court until the disposition of the final federal appeal. [See
Thomas v. American Home Products, Inc., 519 U.S. 913 (1996)]

E. FEDERAL COMMON LAW


1. When Federal Courts Create Federal Common Law Rules
Although Erie held that federal courts cannot create "federal general common law" rules to govern
state-law claims, Erie did not change the authority of federal courts to create "federal common
law."

a. Interpretation of Federal Statute or Constitution


The federal courts create substantive rules of federal law when interpreting the meaning of
federal statutes or the federal Constitution.

b. Creating Rules to Fill Gaps in Federal Regulatory Schemes


The federal courts may also create substantive rules of federal law based on the deter
mination that Congress has expressly or by implication authorized the federal courts to do
so for the purpose of filling in gaps or silences in a federal regulatory statute.

1) Formulating Uniform Federal Standard


In some instances, the federal courts have determined that a uniform judge-made
standard is necessary as a matter of federal common law. Examples of such cases
include suits involving the rights and obligations of the United States, admiralty cases,
border disputes between states, and disputes involving relations with foreign states.
But ordinarily, in the absence of congressional authorization to formulate substantive
rules of decision, federal common law will not be created outside these areas.

CIVIL PROCEDURE-FEDERAL 29.

Example:
Even though no federal statute supplies a rule of decision, Clearfield
Trust Co. v. United States, 318 U.S. 744 (1943), held that rules for
the negotiability of checks payable by the United States should be
governed by uniform judge-made federal law rather than by state law.
2) Borrowing State Standard
When borrowing a legal rule from state law authority, a federal court may select a
rule that is used by a majority of state courts. Or, if there is little need for federal
uniformity, and if the parties might expect that state law would apply, then the
borrowed rule may be the one that would be applied under the law of the state forum.
Example: In De Sylva v. Ballentine, 351 U.S. 570 (1956), the Court held
that the meaning of the term "children" in the federal copyright
statute should be defined in accordance with ordinary usage under
state law, and that it would be up to the federal courts to determine
which state's definition would be applicable.

When Federal Courts Create Federal Implied Rights of Action


In some decisions, federal courts have exercised the authority to provide judicial recogni tion
for "implied" remedies in the form of causes of action that are not specified in federal statutes
or federal constitutional provisions that establish particular rights.

a. Implied Right of Action Based on Federal Statute


Initially, federal courts looked to several factors in determining whether an unspeci fied
remedy could be viewed as "implied" in a statute. These factors looked to the questions of
whether the plaintiff belonged to a class for whose special benefit the statute was enacted,
whether evidence of legislative intent supported an implied remedy, whether such a remedy
would be consistent with the underlying purposes of the legisla tive scheme, and whether a
remedy should not be implied because the cause of action was one traditionally relegated
to state law. This approach led to the recognition, for example, of an implied cause of
action under the nondiscrimination provision of Title IX (a law prohibiting discrimination
on the basis of gender in schools receiving federal funds).

1) Need for Affirmative Congressional Intent


More recently, federal courts have required a showing of affirmative congressional
intent for implied remedies in federal statutes, the absence of which led to the
judicial rejection of an implied cause of action under the nondiscrimination provi
sions of Title VI of the Civil Rights Act.

b. Implied Right of Action Based on Constitution


The federal courts have recognized several implied causes of action for damages against
federal officials for the violation of particular constitutional rights. These rights include the
Fourth Amendment protection from unreasonable searches and seizures, the Equal
Protection right to nondiscrimination in employment on the basis of gender under the Fifth
Amendment, and the right of a prisoner to not be deprived of medical treatment under the
Eighth Amendment's prohibition on cruel and unusual punishment. In other
30. CIVIL PROCEDURE-FEDERAL

cases, federal courts have declined to recognize additional implied remedies based on the
existence of alternate federal statutory remedies. Note that the violation of federal
constitutional rights by a federal official is not cognizable under 42 U.S.C. section 1983
(which provides for a cause of action for constitutional deprivations performed under color
of law because section 1983 applies only to defendants who are state or local officials).
F. EXCEPTIONS TO DIVERSITY OF CITIZENSHIP JURISDICTION
For historical reasons, even though the requirements for diversity of citizenship jurisdiction are
satisfied, federal courts will not exercise jurisdiction over domestic relations or probate proceed ings.
1. Domestic Relations
The federal court will not take jurisdiction over actions "involving the issuance of a divorce,
alimony or child custody decree." [Akenbrandt v. Richards, 504 U.S. 689 (1992)] Note that this
exception is quite narrow. Federal courts may maintain actions upon state court decrees, such as
those for alimony. They also may hear cases involving intra-family torts. They refuse only cases
involving issuance of decrees of divorce, alimony, or child custody.
2. Probate Proceedings
Federal courts will not entertain cases to probate a decedent's estate. To fall within this exception
to diversity of citizenship jurisdiction, however, the claim asserted must involve actualprobate or
annulment ofa will or seek to reach property in the custody ofa state probate court. [Marshall v.
Marshall, 547 U. S. 293 (2006)]
Example: The federal court had jurisdiction over a claim for damages for alleged
tortious interference with testator's efforts to create a trust benefiting the
plaintiff (who was Anna Nicole Smith). [Marshall v. Marshall, supra]
G. MULTIPARTY, MULTIFORUM TRIAL JURISDICTION ACT
The Multiparty, Multiforum Trial Jurisdiction Act applies to accidents meeting the statutory definition.
The principal points are these:
1. Requirements
a. The Action
The Act grants jurisdiction to federal district courts of civil actions that (i) arise "from a
single accident, (ii) where at least 75 naturalpersons have died in the accident (iii) at a
discrete location." [28 U.S.C. §1369(a)]
b. Minimal Diversity
Such jurisdiction attaches based on minimal diversity ofcitizenship; thus, all that is required
is that at least one plaintiff be of diverse citizenship from at least one defen dant.
c. One Additional Condition
In addition, however, one ofthree other conditions must be satisfied: either (i) a defendant
"resides" in a different state from the place where "a substantial part" of the accident took
place (even if the defendant also resides where the accident took place);
CIVIL PROCEDURE-FEDERAL 31.

(ii) any two defendants "reside" in different states; or (iii) substantial parts of the accident
took place in different states.
2. Intervention
Anyone "with a claim arising from the accident" is permitted to intervene as a plaintiff, even if
she could not have maintained an action in the district where the case is pending. [28 U.S.C.
§1369(d)]

3. Service of Process
Finally, the Act provides for nationwide service ofprocess. [28 U.S.C. §1697]

III. FEDERAL QUESTION JURISDICTION


A. INTRODUCTION
It is difficult to formulate a summary of the case holdings as to when an action "arises under"
federal law. The best one can do, perhaps, is the following: A case arises under federal law if the
plaintiff is alleging a right or interest that is substantially founded on federal law, which consists of
federal common law, federal constitutional law, federal statutory law, treaty law, and federal
administrative regulations.
B. FEDERAL QUESTION MUST APPEAR IN THE COMPLAINT
The federal question must appear as part of theplaintiff's cause ofaction as set out in a well
pleaded complaint. It is therefore sometimes necessary to determine whether certain allegations are
proper in pleading the cause of action, and whether the federal element is essential to the plaintiff's
case.

1. Defendant's Answer or Defense Is Irrelevant


The content of the defendant's answer is not relevant; the existence of a defense based on
federal law will not give federal question jurisdiction. Likewise, the court may not look to a
counterclaim asserted by the defendant to determine whether the plaintiff's complaint states a
federal question claim. [Holmes Group, Inc. v. Vornado Air Circulation System, Inc., 535 U.S.
826 (2002)]

2. Anticipation of a Defense
Similarly, a complaint does not create federal question jurisdiction if it alleges federal issues only
in anticipation of some defense.
Example: A sues B for specific performance ofa contract and alleges that B's refusal to
perform is based on B's erroneous belief that federal law prohibits his perfor
mance. No federal question jurisdiction exists because the federal question
presented by the plaintiff's complaint is merely in anticipation of B's defense.
[Louisville & Nashville Railroad v. Mottley, 211 U.S. 149 (1908)]

C. IMPLIED FEDERAL RIGHT OF ACTION


It is not essential that the federal statute expressly provide for a civil cause of action for an alleged
violation. Thus, federal question jurisdiction was held to exist in an action involving an alleged
violation of the Fourth and Fifth Amendments [Bell v. Hood, 327 U.S. 678 (1946)] and
32. CIVIL PROCEDURE-FEDERAL

an alleged violation of the Securities Exchange Acts of 1934 [J. I. Case v. Borak, 377 U.S. 426
(1964)] , although neither the Constitution nor the act involved created a "remedy" for the wrongs
complained of. However, not all federal provisions creating duties are held to create an implied
private right of action. [Cort v. Ash, 422 U.S. 66 (1975)]

D. FEDERAL CORPORATIONS
Federal question jurisdiction does not arise merely from the fact that a corporate party was incor
porated by an act of Congress unless the United States owns more than one-half of the corpora
tion's capital stock, in which case it is treated as a federal agency that can sue or be sued on that basis
in federal court. [28 U.S.C. §1349]

E. SUPPLEMENTAL (PENDENT) JURISDICTION OVER STATE CLAIMS


As previously discussed (supra, II.B.5.), claims sometimes can invoke supplemental jurisdic tion
when the supplemental claim arises from a common nucleus of operative fact as the original claim,
whether the case got into federal court by diversity of citizenship or federal question juris diction.
Supplemental jurisdiction in federal question cases is discussed here.

1. Supplemental (Pendent) Claims


In some cases, the plaintiff will have both federal and state claims against the defendant.
Although there may be no diversity, the federal court has discretion to exercise supple mental
(pendent) jurisdiction over the claim based on state law if the two claims "derive from a
common nucleus of operative fact" and are such that a plaintiff "would ordinarily be expected
to try them all in one judicial proceeding." [United Mine Workers of America v.
Gibbs, 383 U. S. 715 (1966)] Essentially, this means that the two claims must arise from the
same transaction or occurrence. The supplemental jurisdiction statute [28 U.S.C. §1367(a)]
adopts this standard for the grant of supplemental jurisdiction.
Example: P, a citizen of Arkansas, asserts two claims against D, who is also a citizen
of Arkansas, in federal court. Importantly, both claims arise from a
common nucleus of operative fact. Claim #1 is for violation of a federal
statute, and thus invokes federal question jurisdiction. Claim #2 is based on
state law, and thus does not invoke federal question jurisdiction (because it is
based on state, not federal, law). Also, Claim #2 does not invoke diversity of
citizenship (because P and D are citizens of the same state). Nonetheless,
Claim #2 invokes supplemental jurisdiction because it arises from a common
nucleus of operative fact as the claim that invoked federal question jurisdiction.

a. Effect of Dismissal of Federal Claim on Supplemental (Pendent) Claim


The court may exercise supplemental (pendent) jurisdiction over the state claim even
though the federal claim is dismissed on the merits. However, the state claim should
probably also be dismissed (without prejudice) if the federal claim is dismissed before
trial. Indeed, the supplemental jurisdiction statute provides that the court may refuse
supplemental jurisdiction if the federal claim is dismissed, if the state claims are complex
or novel, or if the state claims predominate substantially over federal claims. Note also
that a federal court may not award relief against a state official based solely on a state law
claim. [Pennhurst State School & Hospital v. Halderman, 465 U. S. 89 (1984)]
CIVIL PROCEDURE-FEDERAL 33.

2. Pendent Parties
Pendent party jurisdiction is relevant in cases in which the plaintiff sues more than one
defendant, there is federal jurisdiction over the claim against one defendant, and the claim
against the second defendant does not invoke federal question or diversity of citizenship
jurisdiction. Under the supplemental jurisdiction statute, the claim against the second defen dant
might invoke supplemental jurisdiction if it arises from a common nucleus of operative fact as
the claim against the first defendant.
Example: P asserts a federal question claim against D-1 and joins a transactionally
related state law (not federal question) claim against D-2. P and D-2 are
citizens of the same state. The claim against D-2 does not invoke federal
question jurisdiction (because it is based upon state law) and does not invoke
diversity of citizenship jurisdiction (because P and D-2 are citizens of the same
state) . The claim against D-2 invokes supplemental jurisdiction, however,
because it arises from a common nucleus of operative fact as the
claim that invoked federal question jurisdiction and is asserted by the plaintiff in
a federal question case.

Conversely, pendent party jurisdiction can arise when multiple plaintiffs assert claims against
one defendant.
Example: P-1 asserts a federal question claim against D. In the same case, P-2 asserts a
state law claim against D. P-2 and D are citizens ofthe same state. The claim
by P-2 invokes supplemental jurisdiction if it arises from a common nucleus of
operative fact as the federal question claim by P-1 against D.

F. SPECIFIC STATUTORY GRANTS

1. Amount in Controversy
There is no amount in controversy requirement in federal question cases, with the narrow exception
for cases brought against defendants other than the United States, its agencies, or employees
under section 23(a) of the Consumer Product Safety Act. That section authorizes action by any
person who sustains injury by reason of a knowing violation of a consumer product safety rule, or
any other rule issued by the Commission. In such actions, at least
$10,000 must be in controversy. [15 U.S.C. §2072]

2. Exclusive Jurisdiction
Congress has expressly provided that the jurisdiction of the federal courts shall be exclusive of
state courts in:
Bankruptcy Proceedings [28 U.S.C. §1334]
Patent and Copyright Cases [28 U.S.C. §1338]
c. Many Cases Where United States Is Involved
Cases involving fines, penalties, or forfeitures under the laws of the United States; crimes
against the United States; tort suits against the United States; or customs review. (Because
of the doctrine of sovereign immunity, there is no jurisdiction in the courts
to hear lawsuits against the United States unless the United States has consented to be
sued.)
34. CIVIL PROCEDURE-FEDERAL

Cases with Consuls and Vice-Consuls as Defendants [28 U.S.C. §1351]

e. Antitrust Cases
Although 28 U.S.C. section 1337 does not expressly make federal jurisdiction exclu sive
in actions arising under laws regulating interstate commerce, the federal antitrust statutes
are interpreted to place the remedy exclusively in the federal courts. [Freeman v. Bee
Machine Co., 319 U.S. 448 (1943)]

f. Admiralty Cases-Caveat
28 U.S.C. section 1333 grants exclusive jurisdiction in cases of admiralty and maritime
jurisdiction, but since the same section has a clause "saving to suitors in all cases all other
remedies," the result is that federal jurisdiction is exclusive only in limitation of liability
proceedings and in maritime actions in rem.

g. Foreign State-Caveat
28 U.S.C. section 144l(d) permits a foreign state (or agency thereof), if sued in state
court, to remove the action to federal court.

Postal Matters [28 U.S.C. § 1339]


i. Internal Revenue [28 U.S.C. § 1340]
j. Securities Exchange Act [15 U.S.C. §78aa]

IV. VENUE
A. SUBJECT MATTER JURISDICTION DISTINGUISHED
Subject matter jurisdiction and venue are very often confused. Subject matter jurisdiction is the
power of the court to adjudicate the matter before it, whereas venue relates to the proper
geographic district in which to bring the action. [28 U.S.C. § 1390] Subject matter jurisdiction is a
question of power or authority; venue is a question of geography. Subject matter jurisdiction
cannot be conferred by agreement; venue can be. A court can have subject matter jurisdiction
without having proper venue.
Example: Smith, a citizen of Georgia, brings a personal injury suit arising in Florida against
Jones, a citizen of New York. Suit is brought in the federal district court in
California. The amount in controversy exceeds $75,000. Under section 1332, the
district court has diversity jurisdiction, but venue is improper and the case is
subject to transfer or dismissal.
B. GENERAL RULES
1. General Rules for Most Civil Actions
Venue in civil actions in the federal courts is proper in:
(i) A judicial district in which any defendant resides, if all defendants are residents of the state
in which the district is located;
CIVIL PROCEDURE-FEDERAL 35.

(ii) A judicial district in which a substantialpart ofthe events or omissions giving rise to the
claim occurred, or a substantialpart ofproperty that is the subject ofthe action is situated;
or

(iii) If there is no district anywhere in the United States which satisfies (i) or (ii), ajudicial
district in which any defendant is subject to the court'spersonaljurisdiction with
respect to such action.
[28 u.s.c. §1391]
Note: Unlike past federal practice and the existing practice in many states, local actions (e.g.,
those involving real property) and transitory actions (e.g., tort actions) are treated under the same
venue provision in federal court.

2. Special Venue Provisions


There are many venue provisions applicable only to specified types of actions. One is worth
noting: Where the defendant is the United States or an agency thereof, or an officer, employee,
etc., of the United States acting in his official capacity, a civil action may be brought where: (i)
a defendant resides; (ii) a substantial part of the events or omissions giving rise to the action
occurred, or a substantial part of property that is the subject of the action is situated; or (iii) the
plaintiff resides if no real property is involved in the action [28 U.S.C. § 139l(e)] .

C. RESIDENCE

1. Natural Persons
For venue purposes, a natural person, including an alien lawfully admitted for permanent
residence in the United States, is deemed to reside in the judicial district in which that person is
domiciled. [28 U.S.C. §139l (c)(l)]

2. Business Entities
An entity with the capacity to sue and be sued in its common name under applicable law,
whether or not incorporated, is deemed to reside, if a defendant, in any judicial district in which
the defendant is subject to the court's personaljurisdiction with respect to the civil action in
question. [28 U.S.C. §139l (c)(2)]
a. Note for Corporations
In a state having more than one judicial district and in which a corporate defendant is
subject to personal jurisdiction at the time the action is commenced, the corporation is
deemed to reside in any district in that state within which the corporation's contacts
would be sufficient to subject the corporation to personal jurisdiction if the district
were a state. If there is no such district, the corporation is deemed to reside in the
district within which it has the most significant contacts. [28 U.S.C. §139l (d)]

3. Nonresident of United States


A defendant who is not a resident of the United States-whether a U.S. citizen or an alien may
be sued in any judicial district. The joinder of such a defendant, however, is disregarded in
determining where the action may be brought with respect to any other defendants. [28 U.S.C.
§139l(c)(3)]

36. CIVIL PROCEDURE-FEDERAL

D. IMPROPER VENUE MAY BE WAIVED


Unlike jurisdiction over the subject matter, venue may be waived by the parties. Venue is consid ered to be waived unless timely
objection (in a pre-pleading motion or, where no such motion is made, in the answer) is made to the improper venue.
E. TRANSFER
1. Original Venue Proper
Section 1404(a) allows transfer to another district where the action "might have been brought" or "to
which allparties have consented," even though venue has been properly laid in the court before which the
motion to transfer is made. The policy behind section
1404 is that while venue may be correct, the parties or the witnesses might be greatly incon venienced by the trial in
the original forum. By balancing the relative convenience offered by the alternative forums, the original court has
discretion to transfer the action to a court
in which the action "might have been brought" in conformity with the rules governing: (i) subject matter jurisdiction,
(ii) in personam jurisdiction over the defendant, and (iii) venue. Another alternative is transfer to a court to which all parties
have consented (even if venue ordinarily would not be proper there). If the superior forum is in another judicial system, the
court may dismiss or stay the action under the doctrine of forum non conveniens.
2. Original Venue Improper
When a case is filed in an improper venue, a court must dismiss, or, in "the interests of justice," transfer the
case to a venue in which it could have been brought (i.e., subject matter jurisdiction, personal jurisdiction, and proper
venue must exist). [28 U.S.C. §1406(a)] Transfer is more appropriate than dismissal except in extraordinary circumstances.
Some courts have held that section 1406(a) applies when the original court is a proper venue but
lacks personal jurisdiction over the defendant. This view seems contrary to the language and purpose of the statute.
3. Effect of Forum Selection Clauses
A forum selection clause represents the parties' agreement as to the most proper forum, and it will be enforced by means
of a motion to transfer "in the interests of justice" under section 1404 (if the venue selected by the plaintiff is proper
under section 1391) or under section
1406 (if the venue selected by the plaintiff is improper under section 1391) unless exceptional public interest factors dictate
otherwise. [Atlantic Marine Construction Co. v. United States District Court for the Western District of Texas, 134 S. Ct.
568 (2013)]
4. Original Court Lacks Personal Jurisdiction
The Supreme Court has held that the original court's lack of personal jurisdiction over the defendant does not affect its
power to transfer a case under section 1406(a). [Goldlawr, Inc. v. Heiman, 369 U.S. 463 (1962)] There is also authority
to support the conclusion that the same is true in transfers under section 1404(a). [See, e.g., United States v. Berkowitz,
328 F.2d 358 (3d Cir. 1964)]
F. LAW APPLICABLE UPON TRANSFER
1. Original Venue Proper
A transfer solely on convenience grounds carries to the transferee court the originally
CIVIL PROCEDURE-FEDERAL 37.

applicable (under Erie) rules (including choice of law); i.e., the law of the state in which the
transferor court sat unless transfer was ordered to enforce a forum selection clause. [Atlantic
Marine Construction Co., supra] . This is true even where the plaintiff initiates a transfer for
convenience after initially choosing the inconvenient forum. [Ferens v. Deere Co., 494 U.S. 516
(1990)]
Example: P sued D in a federal district court in Pennsylvania. Upon the transfer to
Massachusetts, the district court judge there must apply the law that would
have been applied in Pennsylvania. [Van Dusen v. Barrack, 376 U.S. 612
(1964)]

2. Original Venue Improper


A transfer on the ground that the original choice of venue was improper generally results in a
change of the law applicable under Erie; i.e, thelaw of the state in which the transferee court
sits.
Example: P sued D in the federal district court in Maryland. Upon transfer to New York
under section 1406(a), the law applied in the transferee court (New York) would
be its own law.

V. REMOVAL JURISDICTION
A. ORIGINAL JURISDICTION NECESSARY
Under section 144l(a), a defendant can remove an action that could have originally been brought by
the plaintiff in the federal courts.

1. When
The prevailing rule is that removal is tested only as of the date of removal. Some courts held
under the statute as it existed before 1989 that original jurisdiction must have existed both at the
time the suit was instituted in the state court and at the time of removal. [In re Carter, 618 F.2d
1093 (5th Cir. 1980)]

2. Federal Defense Insufficient


A defendant cannot remove on the ground that she has a defense grounded in federal law, since
the existence of a federal defense is insufficient to confer original federal question jurisdiction
under section 1331.
3. State Court Need Not Have Had Jurisdiction
By statute, the federal court may hear and decide a claim in a removed civil action even where
the state court had no jurisdiction because the action is exclusively federal. [28 U.S.C.
§144l(e)] Formerly, the federal court was required to dismiss.

B. ONLY DEFENDANT MAY REMOVE; ALL MUST SEEK REMOVAL


Only defendants can exercise the right of removal. Thus, a plaintiff cannot remove on the ground that
a counterclaim against him could have been brought independently in a federal court. If there is more
than one defendant, all defendants who have been properly joined and served must join in or
consent to the removal. (Note: The Class Action Fairness Act relaxes this rule for some class
actions. See VII.G.2.d., infra.)
38. CIVIL PROCEDURE-FEDERAL

C. VENUE
Venue for an action removed under section 144l (a) lies in the federal district court "embracing the
place where such [state] action is pending." In removal cases, section 144l(a) determines proper
venue, not section 139l (a). Thus, in a properly removed case, venue is proper in the federal court of
the state where the case was pending, even if venue would have been improper had the plaintiff
originally filed the action in the federal district court of that state.
Example: Linda, a citizen of State A, sues Jim, a citizen of State B, in the state court in State Z
in the amount of $2 million for negligent acts Jim committed in State B. Jim may
remove the case to the federal district court of State Z because the court has diversity
jurisdiction and Jim is not a citizen of State Z. Although under section 139l (a)
venue would have been improper if Linda had filed her case in the State Z federal
district court, under section 144l (a) venue is proper in the federal district court of
State Z because it "embraces the place" where the state court action was pending.

D. DEFENDANT MAY REMOVE SEPARATE AND INDEPENDENT FEDERAL


QUESTION CLAIM
If a case filed in state court contains a claim that would arise under federal law, and it is joined with
state law claims that do not invoke diversity or supplemental jurisdiction, the entire case can be
removed to federal court. The federal court, however, must then sever and remand the state law claims
to state court. Only those defendants against whom a federal claim is asserted are required to join in
the removal.

E. DISMISSAL OF NONDIVERSE PARTY ALLOWS REMOVAL


If no federal question is involved and diversity does not exist because a party is a co-citizen of
an opposing party, removal will be permitted if the nondiverse parties are thereafter dismissed
from the action and there is complete diversity between the remaining parties, subject to the
limitations discussed below.
F. LIMITATIONS ON REMOVAL IN DIVERSITY OF CITIZENSHIP CASES
1. Defendant Citizen of Forum State
When the jurisdiction of the federal court is based solely on diversity and one of the defen dants
is a citizen of the state in which the state action was brought, the action is not remov able. [28
U.S.C. §144l(a)(2)]
Example: Jones, a citizen of State A, sues Brown, a citizen of State B, and Smith, a
citizen of State C, in the state court in State B. Although diversity jurisdiction
would have existed originally (assuming the jurisdictional amount had been
met), Brown and Smith cannot remove. Had Jones brought the action in the
state court in State A, Brown and Smith could remove. [28 U.S.C. §144l(b)]
When the original jurisdiction of the district court would have been based on a federal
question, the defendants can remove without regard to the citizenship of the parties.
2. One Year Rule
A case may not be removed on the basis of diversity of citizenship jurisdiction more than one
year after it was commenced in state court. [28 U.S.C. § 1446(b)] Note also that a case must
be removed no later than 30 days after the defendant discovers, through service of an amended
pleading, order, etc., that the case has become removable (see G.2., infra). Because
CIVIL PROCEDURE-FEDERAL 39.

most cases will be removable, ifat all, at commencement of the action, the one year deadline
generally will not be difficult to meet. The provision may be important, however, if the case is
not removable at the outset, but becomes removable later. The one year rule does not apply to
removals based on federal question jurisdiction, nor does it apply if the district court finds that
theplaintiffhas acted in badfaith (e.g., by fraudulently joining a nondiverse party
or by intentionally failing to disclose the true amount in controversy) in order to prevent a
defendantfrom removing the action.
Examples: 1) A (a citizen of State A) sues B (a citizen of State B) in state court in State
A, seeking damages of $100,000. The case is removable at its commence ment,
since the case meets the requirements of diversity of citizenship juris diction and
no defendant is a citizen of the forum. B must remove the case within 30 days of
being served with process.

2) P (a citizen of State A) sues Dl (a citizen of State B) and D2 (a citizen of


State A) in state court in State C, seeking damages of $250,000. The case is not
removable at its commencement, since it does not satisfy the complete diversity
rule for diversity of citizenship jurisdiction. However, if P later voluntarily
dismisses the claim against D2, the case becomes removable. Dl must then remove
within 30 days. But if more than a year has passed since the state case was
commenced, Dl cannot remove on the basis of diversity of citizenship.
G. PROCEDURE FOR REMOVAL
1. Notice of Removal
A defendant seeking removal must file a notice of removal-containing a short and plain statement
of the grounds for removal and signed under Rule 11-in the federal district court in the district and
division within which the action is pending. A copy of the notice should be sent to the other
parties and to the state court. Once this is done, the state court can no longer deal with the case.
If the state court attempts to do so, the federal court can enjoin the state court's action.

a. Allegation of Amount in Controversy


In cases seeking nonmonetary relief, or in cases in which the plaintiff (under state law) is not
required to state an amount in controversy or in which recovery may be in excess ofthe
damages stated, the defendant may state that the amount in controversy exceeds
$75,000, and the district court may keep the case if it finds, by a preponderance of the
evidence, that the amount does exceed $75,000.

2. Thirty Day Rule


Generally, a defendant must file a notice of removal within 30 days "after receipt by or service
on that defendant of the initial pleading or summons." [28 U.S.C. § 1446 (b)] The statute is
intended to address different state approaches to the order of filing a case and serving process.
For instance, in some states, the defendant is served with a summons but not a copy of the
complaint. For such defendants, the 30-day removal period would start to run upon formal receipt
of the complaint. If defendants are served at different times, and a later served defendant initiates
timely removal, the earlier served defendant may join in the removal even though his 30-day
period for initiating removal may have expired.
40. CIVIL PROCEDURE-FEDERAL

Example: P files an action against B in state court. P faxes a "courtesy copy" of the
complaint to B, but does not have formal process (a summons and complaint) served
for two weeks. D removes the case within 30 days after being served with process,
but more than 30 days after receiving the faxed copy of the complaint. Was
removal timely? Yes. The 30 days ran from service of process on the defendant.
[Murphy Brothers v. Michetti Pipe Stringing, Inc., 526 U.S. 344 (1999)]

A defendant also may file a notice of removal within 30 days of receipt of an amended pleading,
motion, order, or other court paper (such as discovery or other state court plead ings) that shows
that a nonremovable case (or an apparently nonremovable case) is infact removable. [28
U.S.C. §1446(b)(3)] This provision is significant in states that either prohibit the plaintiff from
alleging or do not require the plaintiff to allege a specific amount of damages and in cases in which
a nondiverse defendant has been dismissed. In such cases, the 30-day window to remove may begin
much later than after the service of the initial pleading.

3. Procedure After Removal


After removal, the case proceeds according to the federal rules of procedure. Repleading is not
necessary unless the court so orders. If the defendant has not answered, she must answer or present the
other defenses or objections available to her under the Federal Rules within 21 days after being
served, or within seven days after filing the petition for removal, whichever period is longer.
Amendments may be made to pleadings filed before removal.
4. Right to Jury Trial
a. Demand for Jury Trial
The right to a jury trial in a case removed to a federal court may be waived unless a timely
demand for a jury trial is filed. If, at the time of removal, all necessary pleadings have been
served, a trial by jury will be granted to a party so entitled. The removing party must file a
demand for jury trial within 14 days after the notice of removal is filed. The nonremoving
party generally must file for jury trial within 14 days after service on her of the notice of
filing for removal.
b. Demand Not Required
A party who, prior to removal, has made an express demand for trial by jury in accor dance with
state law, need not make a demand after removal. In addition, if state law applicable in the court
from which the case is removed does not require the parties
to make an express demand in order to claim trial by jury, they need not make such demand
after removal unless the court directs they do so.

5. Remand
A plaintiff can file a motion to have the case remanded (sent back) to the state court. If the
plaintiff bases this motion on a defect other than subject matter jurisdiction (e.g., a defect in removal
procedures), the motion must be brought within 30 days of removal. The court must remand,
however, whenever it is shown that there was no federal subject matter jurisdiction. If the court
erroneously fails to remand, but the subject matter defect is cured before trial begins, failure to
remand does not require that the federal judgment be vacated. [Caterpillar Inc. v. Lewis, 519 U. S.
61 (1996)] The federal court has discretion to remand a case to state
CIVIL PROCEDURE-FEDERAL 41.

court once all federal claims have been resolved, leaving only state claims over which there
would be no diversity jurisdiction. [Carnegie-Mellon University v. Cohill, 484 U.S. 343 (1988)]
Appellate review of remand orders is generally barred [28 U.S.C. § 1447 (d)] ; however, appeal
is allowed where a case involving civil rights is remanded to state court. Remand orders can also
be reviewed by means of a mandamus if the remand represented a refusal to exercise plainly
proper jurisdiction. [Thermtron v. Hermansdorfer, 423 U.S. 336 (1976)]

Subject Matter Jurisdiction Generally Considered First


Ordinarily, a federal court will determine whether it has subject matter jurisdiction before
it considers the merits of the case. [Steel Co. v. Citizens for Better Environment, 523 U.S.
83 (1998)] However, the Supreme Court has held that when the subject matter jurisdiction
issue is complicated, and the personal jurisdiction issue is fairly easy, a federal court does
not abuse its discretion by addressing the personal jurisdiction issue first. [Ruhrgas AG v.
Marathon Oil Co., 526 U.S. 574 (1999)]
H. SPECIFIC TYPES OF ACTIONS
1. Removable
Statutes allow removal of certain actions againstfederal offi cers when they were allegedly acting
under color of law [28 U.S.C. §1442] andfederal employees for torts by motor vehicle committed in
the scope of employment [28 U.S.C. §2679(d)]; actions involving international banking; and
criminal or civil actions where the defendant will be denied a right protected under federal civil
rights statutes [28 U.S.C. §1443] , but only if the denial is inevitable under state law [Georgia v.
Rachel, 384 U.S. 780 (1966)] . Not all of these special types of cases could have been brought
originally in the federal courts.

2. Nonremovable
Actions under the Federal Employers Liability Act ("FELA'') are not removable; nor are
actions for less than $10,000 against carriers for losses to certain shipments in interstate
commerce or workers' compensation proceedings. [28 U.S.C. § 1445]

All Writs Act Not a Basis for Removal


Some courts permitted removal of a case under the All Writs Act, 28 U.S.C. section 1651
(permitting federal courts to "issue all writs necessary or appropriate in aid of their
respective jurisdictions and agreeable to the usages and principles of law"), even if the case
did not invoke federal subject matter jurisdiction. However, the Supreme Court held that the
All Writs Act does not provide an independent basis for removal jurisdic tion; a case thus
must satisfy some independent basis of subject matterjurisdiction, such as diversity of
citizenship or federal question jurisdiction. [Syngenta Crop Protection, Inc. v. Henson, 537
U.S. 28 (2002)]

VI. CONFLICT OF JURISDICTION BETWEEN STATE AND FEDERAL COURTS


A. FULL FAITH AND CREDIT EXTENDED TO FEDERAL COURTS
The Constitution's Full Faith and Credit Clause-which requires a state to recognize the legisla tive
acts and judicial decisions of its sister states-is applicable only where a state court judgment
42. CIVIL PROCEDURE-FEDERAL

is sought to be enforced in another state. However, an implementing federal statute provides that this
Clause is extended to the federal courts. Therefore, recognition ofjudgments is required between state
and federal courts and between federal courts.

B. INJUNCTIONS AGAINST PENDING STATE PROCEEDINGS


Potentially, a case also could be filed in state court by one party and in federal court by the other
party. In such a case, federal court is prohibited from enjoining pending state court proceedings unless
expressly authorized by statute (e.g., the interpleader provision expressly authorizes injunc tions against state
court proceedings), or "where necessary in aid of its jurisdiction, or to protect or effectuate its
judgments." [28 U.S.C. §2283] The case coming to a final decision first will have preclusive effect
on the other. (See X.B., infra .)
C. INJUNCTIONS AGAINST THREATENED STATE CRIMINAL PROSECUTIONS
Threatened state criminal prosecutions (i.e., where state court proceedings have not already been
instituted) will be enjoined only when necessary to prevent irreparable harm which is clear and
imminent and where appellate remedies in the criminal case are clearly inadequate to provide
relief Such injunctions are almost invariably denied, except where a federal right of free speech or
assembly or a federally protected civil right is threatened by the state criminal proceeding, and it is
shown that the prosecution is in bad faith or is for the purpose of harassment. Relief by declaratory
judgment will ordinarily be denied if an injunction would be denied.

D. INJUNCTIONS AGAINST STATE TAX PROCEDURES


28 U.S.C. section 1341 prohibits injunctions against the assessment, levy, or collection of state
taxes "where there is a plain, speedy and efficient remedy . . . in the courts of such State." [See
Rosewell v. LaSalle National Bank, 450 U.S. 503 (1980)]

VII. THE FEDERAL RULES OF CIVIL PROCEDURE


A. COMMENCEMENT OF THE ACTION
Rule 3 provides that an action is commenced byfiling a complaint with the court. Federal courts may
adopt local rules to permit filing by fax or other electronic means. Filing a complaint before the
statute of limitations has run will satisfy the statute of limitations in federal question cases and in
diversity cases where the state rule is similar. However, the Supreme Court has held that a state rule
that an action is commenced for purposes of the statute of limitations only upon service of process must
be applied in diversity cases. [Walker v. Armco Steel Corp., 446 U.S. 740 (1980)]
B. SERVICE OF PROCESS
1. Who May Serve
Rule 4 authorizes any person who is at least 18 years old and not a party to the action to serve
the summons and complaint (together known as "process"). A party may request that service be
made by a United States marshal or by another person appointed by the court for that purpose.
2. How Service Is Made
An individual may be served by (i) personal service, (ii) service left at the defendant's usual
CIVIL PROCEDURE-FEDERAL 43.

place of abode with one of suitable age and discretion residing therein, or (iii) service upon an
authorized agent of the defendant. A minor or incompetent person must be served in accordance
with the rules of the state in which service is to be made. A corporation, partner ship, or association
may be served by serving an officer, a managing or general agent, or an authorized agent of the
corporation. Alternatively, service may be made under state rules or by mail under the waiver of
service provision. Service within the United States ordinarily must be made within 90 days of
the complaint being filed; however, the court must extend this period for good cause shown.
[Fed. R. Civ. P. 4]
a. Service Under State Rules
Rule 4 provides that service-other than on a minor, incompetent person, or a person
whose waiver has been filed-may alternatively be made as provided by the rules of the
state in which the federal court sits or the state in which service is to be effected,
regardless of the basis of subject matter jurisdiction. Hence, federal courts can use state long
arm provisions.
b. Waiver of Service (Service by Mail)
The plaintiff may also request the defendant to waive service of process. To request a waiver
of service, the plaintiff must mail the defendant certain items, the most impor tant of
which are a formal request to waive service (that also informs the defendant of the
consequences of failing to waive service), two copies of the waiver form, and a copy of the
complaint. The defendant generally has 30 days (60 days if outside the United States) from
the date that the request was sent to return the waiver.
1) Effect of Waiver
A defendant who waives formal service of process has 60 days (90 days if outside the
United States) from the date the request was sent, instead of the usual 21 days (see
F.3.b., infra) to answer the complaint. The waiver of service does not waive the
defendant's right to object to venue and jurisdiction.
2) Effect of Failure to Waive
If the defendant does not waive service of process, the plaintiff must serve him using
one of the methods described in 2., supra. However, a defendant who is located in the
United States is liable for the cost of such service if he does not have good cause for
failing to waive service.
3. Parties Served Outside State
The court will acquire personal jurisdiction over parties served outside the state:
a. Under statute and rules for extraterritorial service of the state in which the federal court sits
(domiciliaries, long arm jurisdiction, and in rem jurisdiction);
b. If they are third-party defendants [Fed. R. Civ. P. 14] or required to be joined for just
adjudication [Fed. R. Civ. P. 19], if served within 100 miles from the place where the
summons was issued (but within the United States);

c. If out-of-state service is permitted by federal statute (e.g., interpleader); and


d. For cases that involve afederal question, when a defendant is served with process (or waiver
thereof), provided that the defendant is not subject to generaljurisdiction in
44. CIVIL PROCEDURE-FEDERAL

any state court, that the defendant has sufficient contacts with the United States to warrant the
application of federal law, and that the exercise of jurisdiction is not prohib ited by statute.
4. Parties Served in Foreign Country
Unless a federal law provides differently, a court will acquire personal jurisdiction over a party served in a foreign
country:
a. As provided in an international agreement;
b. In absence of an agreement, as provided by theforeign country's law or as directed by aforeign offi
cial in response to a letter of request (but the method must be reasonably calculated to provide notice);
c. Unless it is prohibited by the foreign country's law, by personal service or by mail, signed return receipt
requested. (However, a corporation may not be served by personal service, and a minor or incompetent person may
not be served by either of these methods); or
d. Any method the court orders (so long as the method is not prohibited by international agreement).
5. Immunity from Process
The federal courts recognize the immunity from service of process of parties, witnesses, and attorneys who enter a state
to appear in another action. In addition, if a party was induced by the plaintiff's fraud or deceit to enter a state so that he
could be served, the service is invalid and the court does not acquire personal jurisdiction.
C. TIME PERIODS
1. Counting Time
Under Rule 6(a), when a time period is expressed in days, the day of the event that triggers the period is excluded,
and the last day of the period is included, when determining on what day an action must be taken. Intermediate
Saturdays, Sundays, and legal holidays are
included, except that if the last day of the period falls on a Saturday, Sunday, or legal holiday, the due date becomes the
next workday. Three days are added to the period when service of the paper is made by mail, by leaving the paper with
the clerk, or by other means to which the parties have consented.
2. Extensions of Time
Rule 6(b) gives the district court power to extend the period within which actions under the Federal Rules must be
performed (e.g., when a paper is delivered late in the day). However, certain time periods may never be extended. The
following motions must be filed, with no extensions, within 28 days after entry ofjudgment: a renewed motion for
judgment as a matter of law, a motion to amend judgment, a motion for a new trial, a motion to amend findings of
fact in a nonjury case, and a grant of a new trial on the court's initiative.
D. INTERLOCUTORY INJUNCTIONS
An injunction is an equitable remedy by which a person is ordered to act or to refrain from acting in a specified manner.
Interlocutory injunctions are granted to maintain the status quo until a trial on the merits may be held.
CIVIL PROCEDURE-FEDERAL 45.

1. Preliminary Injunction
A preliminary injunction is sought by a party prior to a trial on the merits of the complaint.
A preliminary injunction may not be issued without notice to the adverse party. [Fed. R.
Civ. P. 65(a)]

a. Requirements for a Preliminary Injunction


A preliminary injunction ordinarily will be granted when: (i) the plaintiff will suffer
irreparable harm if the injunction is not granted; (ii) the harm to the plaintiff if the
injunction is not granted outweighs the harm to the defendant if the injunction is granted;
(iii) the plaintiff shows that he is likely to be successful on the merits; and (iv) the public
interest favors granting the injunction. Irreparable harm is the most impor tant factor. If the
party seeking injunctive relief has an adequate remedy at law (e.g., money damages will
adequately compensate the party), the injunction will be denied.

2. Temporary Restraining Order


A temporary restraining order ("TRO") is granted by a court when it is necessary to prevent
irreparable injury to a party, and the injury will result before a preliminary injunction hearing can
be held.

a. Requirements for Ex Parte Temporary Restraining Orders


Generally, notice of the hearing for the issuance of the TRO must be given before a TRO
is issued. However, a court may grant a TRO without notice ofthe hearing to the adverse
party if three requirements are met [Fed. R. Civ. P. 65(b), (c)] :

1) Specific Facts Showing Immediate and Irreparable Injury


The moving party must give specific facts in an affidavit or in the verified complaint
to establish that immediate and irreparable injury will result to the moving party
before the adverse party can be heard in opposition.
2) Efforts to Give Notice
The moving party must certify in writing all efforts she made to give notice of the
hearing to the adverse party and the reasons why notice should not be required.
3) Security
The moving party must provide some security, the amount of which is determined by
the court, to pay for any costs and damages incurred by the adverse party if
he was wrongfully enjoined or restrained. The United States, its officers, and its agencies
are not required to give security.
b. Notice of Hearing vs. Actual Notice
Although a TRO may be issued without notice of the hearing, due process requires that a
person must receive actual notice (through service of process or otherwise) of the TRO (or
any other injunction for that matter) before he may be held in contempt for violating it.
[See Fed. R. Civ. P. 65 (d)]

c. Time Limit
The TRO will expire within 14 days unless the restrained party consents to an extension or
good cause is shown for an extension.
46. CIVIL PROCEDURE-FEDERAL

E. PROVISIONAL REMEDIES
"Provisional remedies" provide for the pretrial seizure of property for the purpose of securing
satisfaction of ajudgment that may be entered in the case. Federal Rule 64 specifically authorizes the
use of provisional remedies but notes that the remedy's precise name and the precise proce dure to
be used will be governed by state law. Some of the more common remedies (which are also listed
in Rule 64) are:

(i) Garnishment-A court order directing that money or property in the hands of a third party
(e.g., wages) be seized;
(ii) Attachment-A process by which another's property is seized in accordance with a writ or
judicial order for the purpose of securing a judgment yet to be entered. (See I.E.2.a.3), supra,
and I.E.2.b., supra, for some restrictions on the use of attachment.); and
(iii) Replevin-A process by which the plaintiff takes possession of and holds disputed property
during the lawsuit (possession pendente lite).

Procedure will vary from state to state, but the party seeking the remedy generally must make out a
prima facie case on the underlying claim and show that the property will likely not be available after
trial if relief is not granted.
F. PLEADINGS
Pleadings serve the function of giving notice to the opposing parties.
1. Complaint
Each claim for relief should contain:

(i) A short statement of the grounds for the court's jurisdiction;


(ii) A short statement of the claim showing that the pleader is entitled to relief; and
(iii) A demand for judgment for relief, which may be in the alternative.

The federal pleading rules generally require only that a pleader put the other side on notice of
the claim being asserted; detailed assertions of facts underlying the claim generally are not
required. However, the Supreme Court in recent years has required that the plaintiff state facts
supporting a plausible (not just possible) claim. [Bell Atlantic Corp. v. Twombly, 550 U.S. 544
(2007); Ashcroft v. Iqbal, 556 U.S. 662 (2009)]
2. Pre-Answer Motions
a. Rule 12(b)
Prior to filing an answer, the defendant may, ifhe chooses, file a motion and raise any or
all of the following defenses:

(i) Lack of subject matter jurisdiction;


(ii) Lack of personal jurisdiction;
(iii) Improper venue;

CIVIL PROCEDURE-FEDERAL 47.

(iv) Insufficient process;


(v) Insufficient service of process;

(vi) Failure to state a claim upon which relief can be granted (i.e., even if plaintiff's
allegations are taken as true, relief could not be granted); or

(vii) Failure to join a party needed for a just adjudication (includes necessary and indis-
pensable parties).

The first defense may be raised at any time-even for the first time on appeal. The
defendant must raise defenses (ii) through (v) at the time hefiles a motion or his answer
(or an amendment as ofright thereto)-whichever is first. If he does not, the defendant
waives these defenses. The last two defenses (if limited to failure to join an "indispensable
party") can be made at any timeprior to trial or "at trial." The defen dant may choose not to
file a motion and instead raise these defenses in his answer. A motion to dismiss for failure to
state a claim that raises issues outside of the pleadings will be treated as a motion for
summary judgment.

b. Motion for More Definite Statement


A party may move for a more definite statement before responding (by filing an answer or
reply) to a pleading (a complaint) that is so vague or ambiguous that a responsive pleading
cannot reasonably be framed. The opposing party has 14 days after notice of an order to obey
unless the court fixes a different time. If not obeyed, the court may strike the pleading or
issue any other appropriate order. [Fed. R. Civ. P. 12(e)]

c. Motion to Strike
Before responding to a pleading or, if no responsive pleading is permitted, within 21 days
after service of the pleading, a party may move to have stricken any insufficient defense,
or any redundant, immaterial, impertinent, or scandalous matter. Such motion may also be
made upon the court's initiative at any time. [Fed. R. Civ. P. 12(f)]

And note: An objection of failure to state a legal defense to a claim is not waived merely
because a Rule 12(f) motion is not made. Such a defense can be made by motion for
judgment on the pleadings, or at the trial. [Fed. R. Civ. P. 12(h)]
3. Answer
a. Must Contain Denials or Admissions and Any Affirmative Defenses
The answer must contain a specific denial or admission of each averment of the complaint,
or a general denial with specific admissions to certain averments. Where the defendant is
without knowledge or information sufficient to form a belief, a statement to that effect constitutes
a denial. Afailure to deny constitutes an admission. The answer must also state any affi rmative
defenses the defendant may have, such as statute of limitations, Statute of Frauds, res judicata,
etc.
b. Time
If no Rule 12 motion is made, a defendant who was formally served with a summons and
complaint must present an answer within 21 days after service; a defendant to
48. CIVIL PROCEDURE-FEDERAL

whom the complaint was mailed and who waives formal service must answer within 60
days after the request for waiver was mailed to her. If a Rule 12 motion is made and the
court does not fix another time, the responsive pleading is to be served within 14 days of
the court's denial or postponement of the motion. The answer is due within 14 days of
service of a more definite statement if the court grants a Rule 12(e) motion (see 2.b.,
supra.) The same timing rules apply to answers to counterclaims and cross-claims.

c. Counterclaims
Claims that the defendant may have against the plaintiff may be pleaded in the answer as
counterclaims. If a counterclaim arises out of the same transaction or occurrence as one
of the plaintiff's claims, it is a compulsory counterclaim and must be pleaded or it will be
barred. Any other counterclaim is permissive and may be asserted (assuming there is
subject matter jurisdiction) even though there is no connection at all between it and the
plaintiff's claim.

d. Effect of Failure to Answer-Default and Default Judgment


A default is simply a notation in the case file by the clerk that there has been no answer
filed within the time permitted by the rules. A defaultjudgment is a judgment, with the
same effect as any other judgment, that is entered because the defendant did not oppose the
case.
1) Default
If a party against whom a judgment for relief is sought has failed to plead or other
wise defend, and that fact is made to appear by affidavit or otherwise, the clerk must
enter the default of that party. Once the default has been entered, the party may not
proceed with the action until the default has been set aside by the court. [Fed. R. Civ.
P. 55]
2) Default Judgment
A defendant against whom a default is entered loses the right to contest liability.
However, the amount of damages must still be determined before a default judgment
may be entered, and the defaulting party can be heard at the hearing for damages. A
default judgment may be entered against a minor or incompetent person only if she
has a personal representative who has appeared in the case.
a) Default Judgment Entered by the Clerk
On request of the plaintiff, supported by an affidavit as to the amount due,
the clerk may sign and enter judgment for that amount and costs against the
defendant if: (i) the plaintiff's claim against the defaulted defendant is
for a sum certain; (ii) the default was entered because the defendantfailed to
appear; (iii) the defaulted defendant is not an infant or incompetent
person; and (iv) the damages amount requested is not greater than the amount
requested in the complaint. [Fed. R. Civ. P. 54(c), 55(b)(l)]

3) Notice Required
If the defendant has "appeared," even though he has not answered, he must be notified
of the request for a default judgment by first-class mail at least seven days before the
hearing on the application for a default judgment. Appearance includes
CIVIL PROCEDURE-FEDERAL 49.

any actual formal appearance before the court and any other action that clearly indicates
that the defendant intends to contest the case on the merits (e.g., the defendant's
continued settlement negotiations). [Fed. R. Civ. P. 55(b)(2)]

4) Setting Aside a Default or a Default Judgment


An entry of default may be set aside for "good cause shown." Although not specifi
cally required by the rules, a majority of courts will require that the defendant have a
meritorious defense. A default judgment may be set aside as provided in Rule 60
(relief from judgments) (see VIII.A., infra).

4. Inconsistent Claims or Defenses


A party may set out as many alternative claims or defenses as he may have regardless of
consistency.

5. Special Pleading
The general rule of pleading is for short and plain statements, but there are certain rules for
special circumstances. [See Fed. R. Civ. P. 9] Note that in some of these situations (notably
concerning fraud, mistake, and special damages), the Federal Rules require a party to state
more detail than simply a short and plain statement. These situations requiring greater
specificity are narrow, however, and the Supreme Court has emphasized that courts have no
power to impose such rigorous pleading requirements outside the areas addressed by Federal
Rule or statute. [Swierkiewicz v. Sorema N.A., 534 U.S. 506 (2002) -lower court erred by
requiring detailed pleading of employment discrimination claim; Leatherman v. Tarrant County,
507 U.S. 163 (1993)-lower court erred by requiring detailed pleading of civil rights case
against municipality]

a. Capacity
Capacity or authority to sue or be sued need not be alleged. A person wishing to challenge
a party's capacity has the duty to raise the issue by specific negative averment, including
such particulars as are within his knowledge.

b. Fraud or Mistake
Circumstances that establish fraud or mistake must be stated withparticularity. By statute
(the Private Securities Litigation Reform Act), plaintiffs in federal securities fraud cases
must plead with particularity facts relating to the defendant's acting with the required
scienter.

c. Conditions of the Mind


Malice, intent, knowledge, or other conditions of the mind may be averred generally.

d. Conditions Precedent
The performance of conditions precedent may be alleged generally. Denial of perfor mance or
occurrence must be made specifically and with particularity.

e. Official Document or Act


When dealing with an official document or act, it is sufficient to aver that it was issued
or the act was done in compliance with the law.
50. CIVIL PROCEDURE-FEDERAL

f. Judgment
It is not necessary to averjurisdiction when a domestic or foreign court or a board or
officer renders ajudgment or decision.
g. Timing
Time and place averments are material for the purpose of testing the sufficiency of a
pleading.
h. Special Damages
Elements of special damages must be specifically stated.
6. Reply
A reply by the plaintiff to the defendant's answer is required only if the court orders the
plaintiff to file one. A plaintiff need not reply to an affirmative defense; he is deemed to
deny or avoid the allegation of the defense. [Fed. R. Civ. P. 7, 12]
7. Amendment and Supplemental Pleadings
a. Amendment
As a matter of course, a pleading may be amended once within 21 days of serving it
or, if the pleading is one to which a responsive pleading is required, 21 days after
service of a responsive pleading or a pre-answer motion. Thereafter, a pleading may be
amended only by the written consent of the adverse party or by leave of the court
upon motion.
Leave of the court is "freely given when justice so requires." [Fed. R. Civ. P. 15]
1) Relation Back
For statute of limitations purposes, an amendment to a pleading that arises from
the same conduct, transaction, or occurrence that was set forth (or was attempted
to be set forth) in the original pleading generally is deemed filed on the date
that the original pleading was filed. (In other words, the filing of the amendment
relates back to the filing date of the original pleading.) Amendments also relate
back if relation back is permitted by the law that provides the statute of
limitations applicable to the action. [Fed. R. Civ. P. 15(c)] Of course, the
original complaint must have been filed within the applicable statute of limitations
period.
2) Changing Party
An amendment changing the party or the naming of the party against whom a
claim is asserted relates back if the amendment concerns the same conduct,
transaction, or occurrence as the original pleading and if, within the period for
filing a complaint and serving process, the party to be brought in by amendment:

(i) Has received such notice ofthe action that she will not be prejudiced in
maintaining her defense on the merits; and
(ii) Knew or should have known that, butfor a mistake concerning the proper
party's identity, the action would have been brought against her.

[Fed. R. Civ. P. 15(c)(l)(C)] The Supreme Court has emphasized that it is the
knowledge of the party to be brought in by amendment (not of the plaintiff) that
CIVIL PROCEDURE-FEDERAL 51.

is relevant. [Krupski v. Costa Crociere S.p.A., 560 U.S. 538 (2010)] And again, the original
complaint must have been filed within the applicable statute of limitations period.

3) Conform to Evidence
A pleading may be amended during or after trial, or even after judgment, to conform to
the evidence, reflect an issue actually tried by the express or implied consent of the
parties, or permit the raising of new issues at trial. However, a party may not raise a new
claim or defense for which the opposing party had no oppor tunity to prepare and which
would result in prejudice in maintaining his action or defense. [Fed. R. Civ. P. 15(b)]

4) Due Process Limitation


Amendments to pleadings must satisfy due process. For example, in Nelson v. Adams
U.S.A. Inc., 529 U.S. 460 (2000), the trial court permitted a post-verdict amendment to
add a defendant, and simultaneously entered judgment against
that new defendant. The Supreme Court held that this procedure violated the new
defendant's due process rights. The Federal Rules are meant to provide an oppor tunity for
an added defendant to respond to a claim, and do not permit such "swift passage from
pleading to judgment in the pleader's favor."
b. Supplemental Pleadings
Supplemental pleadings relate to matters occurring after the date of the original pleading.
The permission of the court, upon motion, is required. Permission may be granted even
though the original pleading is defective in its statement of a claim for relief or a defense.
[Fed. R. Civ. P. 15(d)]

8. Rule 11

a. Certification upon Presenting Paper to Court


In federal civil cases, the attorney (or unrepresented party), by presenting to the court a
pleading, written motion, or other paper, certifies that to the best of her knowledge,
information, and belief formed after an inquiry reasonable under the circumstances:
(i) The paper is not presented for any improper purpose (harassment, delay, etc.);
(ii) The legal contentions therein are warranted by existing law or a nonfrivolous
argument for the modification ofexisting law or the establishment of a new law;
(iii) The allegations and factual contentions either have, or upon further investigation or
discovery are likely to have, evidentiary support; and
(iv) Denials of factual contentions are warranted on the evidence or, where specified, are
reasonably based on a lack of information and belief.

The certification applies anew each time an attorney or unrepresented party "later advocates" a
position contained in a pleading, motion, etc. Thus, a paper that was not sanctionable when first
presented may become sanctionable if the attorney or party later
52. CIVIL PROCEDURE-FEDERAL

advocating a position contained in the paper has since learned or should have learned that
the position no longer has merit.
b. Sanctions
The court has discretion to impose sanctions, "limited to what is sufficient to deter
repetition of such conduct," against a party who presents a paper to the court in viola tion of
the above requirements, either on the court's own initiative or on motion of the opposing
party. When appropriate, sanctions may be imposed against parties, attor neys, or law firms,
and may consist of nonmonetary directives or monetary penalties including payment of
expenses and attorneys' fees incurred because ofthe improper paper. However, a monetary
sanction may not be imposed on a represented party for violation of a.(ii), supra.
1) Court's Initiative
A court on its own initiative may enter an order describing the matter that appears to
violate Rule 11 and direct the proponent to show cause why sanctions should not be
imposed.
2) Party's Motion
A party who believes that his opponent has presented a paper in violation of Rule 11
may serve a motion for sanctions on the party. If the party does not withdraw
or correct the matter within 21 days, the moving party may then file the motion for
sanctions with the court.
G. JOINDER
1. Joinder of Parties
a. Capacity
An individual's capacity to sue or be sued is determined by the law of her domicile; the
capacity of an organization (e.g., an association or partnership) is determined by the law of the
state where the federal court sits, except that a partnership or unincorpo rated association
always has capacity where a substantive federal right is asserted by or against it.
b. Compulsory Joinder (Indispensable Parties)
In certain situations, a plaintiff must join all interested parties or face dismissal of the
lawsuit. The dismissal is usually sought under Rule 12(b), and the issue may be raised any
time up until judgment. [Fed. R. Civ. P. 19] Analysis of a compulsory joinder issue follows a
three-step process:
(i) Should the absentee be joined?
(ii) Can the absentee be joined?
(iii) If not, should the action proceed in his absence (i.e., is the absentee "indispens able")?
1) Should the Absentee Be Joined?
The absentee should be joined as a party when:

CIVIL PROCEDURE-FEDERAL 53.

(i) Complete reliefcannot be accorded among the other parties to the lawsuit without
the absentee being made a party; or
(ii) The absentee has such an interest in the subject matter of the lawsuit that a
decision in his absence will:
i. As a practical matter, impair or impede his ability toprotect the
interest; or
ii. Leave any of the other parties subject to a substantial risk ofincurring
multiple or inconsistent obligations.
2) Can the Absentee Be Joined?
Assuming that the absentee should be joined under the analysis above, the next question
is whether he can be joined, i.e., whether the court can obtain personal jurisdiction over
the absentee and whether the court will still have subject matter jurisdiction over the
action after joinder of the absentee. (28 U.S.C. section 1367 does not permit
supplemental jurisdiction over claims by or against parties joined
under Rule 19, so joinder of a party who would destroy diversity is not possible
under the compulsory joinder rules.) If the court has personal jurisdiction over the
absentee, and his joinder will not destroy diversity or venue, he must be joined.
3) Should the Action Proceed Without the Absentee?
If the absentee cannot be joined (e.g., plaintiff cannot serve process on the absentee),
the court must determine whether "in equity and good conscience" the action
shouldproceed among the parties before it, or should be dismissed, the absentee thus
being regarded as indispensable. The decision requires consideration of:
(i) The extent ofprejudice to the absentee or availableparties of ajudgment;
(ii) The extent to which the prejudice can be reduced or avoided by means of
protective provisions in the judgment, the shaping of relief, or other measures;
(iii) The adequacy ofajudgment rendered without the absentee; and
(iv) Whether the plaintiff will have an adequate remedy (e.g., in anotherforum)
ifthe case is dismissed for nonjoinder.
Note: The Supreme Court has held that ajoint tortfeasor subject to joint and several
liability is not a person needed for just adjudication. [Temple v. Synthes Corp., 498
U.S. 5 (1990)]

c. Permissive Joinder-Requirements
Parties may join as plaintiffs or be joined as defendants whenever:

(i) Some claim is made by each plaintiff and against each defendant relating to or
arising out of the same series ofoccurrences or transactions; and
(ii) There is a question offact or law common to all the parties.
54. CIVIL PROCEDURE-FEDERAL

Example: It is very common for all persons injured in an automobile accident to


join as plaintiffs. The common issue is the defendant's negligence; the
other issues of contributory negligence and damages are tried individu ally for
each plaintiff.
The court is given wide discretion to order separate trials where joinder would be unfair to
a party not sufficiently involved in all the claims. Note that the court still must have subject
matter jurisdiction over the claim.
2. Joinder of Claims
The policy of the Federal Rules is to permit the adjudication of all claims between the parties and
all claims arising out of a single transaction. A plaintiff can join any number and type
of claims against a defendant; when multiple plaintiffs or multiple defendants are involved, it is
essential only that at least one of the claims arise out of a transaction in which all were
involved.
a. Successive Claims
Rule 18 (b) permits the plaintiff to join two claims when success on the first is a prereq
uisite to the second, such as a claim for money damages and a suit to set aside a convey
ance that was fraudulent because of the debt asserted in the first claim.
b. Jurisdiction
When jurisdiction is based on the diversity of citizenship between the plaintiff and defendant,
the plaintiff may aggregate all claims she has against the defendant to satisfy the
jurisdictional amount. When jurisdiction is based on a "federal question" claim,
and diversity jurisdiction is not available, a nonfederal claim can be joined only if the court
has supplemental (pendent) jurisdiction over it. The court will have supplemental (pendent)
jurisdiction over the claim if it is regarded as part of the same case or contro versy as the
federal claim.
Example: Plaintiffs claimed that the defendant appropriated plaintiffs' literary work in
such a way as to (i) infringe federal law copyright, and (ii) constitute state
law unfair competition. There was federal pendent juris diction over the state
claim. [Hurn v. Oursler, 289 U.S. 238 (1933)]
c. Class Actions
1) Prerequisites
Rule 23 takes a functional approach to the class action device. Named representa
tives will be permitted to sue on behalf of a class if:
a) The class is so numerous that joinder of all members is impracticable;
b) There are questions of law orfact common to the class;
c) The namedparties' interests are typical of the class;
d) The named representatives will ensure thefair and adequate representation
of the interests of absent members of the class [Fed. R. Civ. P. 23 (a)]; and

CIVIL PROCEDURE-FEDERAL 55.

e) The action meets the definition of any of the following three types of class
actions found in Rule 23(b):

(1) Separate actions by class members would create a risk ofinconsistent


results or, as a practical matter, would impair the interests ofother absent
members ofthe class; or
(2) A to act on grounds applicable to the class and injunctive or declaratory
defe reliefis appropriate for the class as a whole (e.g., employment
nda discrimination claims; but note that in Wal-Mart Stores, Inc. v. Dukes,
nt 131 S. Ct. 2541 (2011), the Supreme
has Court rejected an effort to recover monetary relief in such a class action); or
acte
d or (3) There are questions offact or law common to members of the class that
refu predominate over individual issues and a class action is superior to the
sed alternative methods of adjudication.

2) Consideration in Treating Case as a Class Action


The court should determine at an early practicable time whether the action may be
maintained as a class action, i.e., whether to "certify" a class, but may determine at
any time thereafter that the action is not an appropriate one for class action treat ment.
In determining whether to treat the case as a class action, the court should consider,
inter alia, the following factors: (i) the interest of individual control,
(ii) the extent and nature of litigation elsewhere on the same subject, (iii) the
desirability of having the whole package in this court, and (iv) the difficulties in
managing the class action.

a) Court Must Define Class Claims, Issues, or Defenses


The court, in certifying a class, must "define the class and the class claims,
issues, or defenses."

b) Appointment of Class Counsel


The court must appoint class counsel for every certified class and expressly
mandates that the attorney must fairly and adequately represent the interests of
the class. [Fed. R. Civ. P. 23(g)(l)]

3) Effect of Judgment
All members of a class will be bound by the judgment rendered in a class action
except those in a "common question" class action [Fed. R. Civ. P. 23(b)(3); see c.1)
e)(3), supra] who notify the court that they do not wish to be bound ("opting out").
Members of Rule 23(b)(l) and 23 (b)(2) classes cannot opt out. Note, however, that if
the substantive claim of the individual representing the class is mooted, this does not
render the class action moot. [United States Parole Commission v. Geraghty, 445 U.S.
388 (1980)-release from prison of named plaintiff in class action suit challenging
parole procedure did not moot entire class action suit]
56. CIVIL PROCEDURE-FEDERAL

a) Personal Jurisdiction Over Absent Class Members Not Required


The minimum contacts requirements that must be met for the assertion of
personal jurisdiction need not be satisfied to bind absent members of the
plaintiff class in a Rule 23 (b)(3) ("common question") suit who chose not to
opt out. [Phillips Petroleum Co. v. Shutts, 472 U.S. 797 (1985)] This allows a
state or federal court to bind persons to a class action judgment under Rule
23(b)(3) even though they have no contact at all with the state.
4) Notice
a) Notice Required in Common Question Suits
Notice to all members of the class ofthe pendency of the class action is
required under Rule 23 only in "common question" suits [Fed. R. Civ. P.
23(b)(3)], so that class members can opt out. The notice must state (i) the
nature of the action, (ii) the definition of the class, (iii) the class claims,
issues, or defenses, and (iv) the binding effect of a class judgment.

b) Notice Discretionary in Other Types of Class Action Suits


Notice to members of the class of the pendency of the class action in other class
suits is discretionary with the court. [See Eisen v. Carlisle & Jacquelin, 417
U.S. 156 (1974); Oppenheimer v. Sanders, 437 U.S. 340 (1978)]

Note: Notice of dismissal or compromise is a separate notice. Itmust be given


to class members of all types of class actions if the case is dismissed or
settled. (See 6), infra .)
5) Jurisdiction
a) Diversity Action
In class actions founded on diversity, only the citizenship ofthe named
representatives of the class is taken into account to establish diversity. The
Supreme Court held that the amount in controversy requirement is satisfied in a
class action that invokes diversity of citizenship if any named representa tive's
claim exceeds $75,000. The class action may proceed in federal court even
ifclass members' claims do not exceed $75,000. The claims by the class members
that do not exceed $75,000 invoke supplemental jurisdiction. [Exxon Mobil
Corp. v. Allapattah Services, 11.C.3., supra]

b) Federal Question Action


If the class asserts a claim arising under federal law, it can invoke federal
question jurisdiction. In that sort of case, of course, the citizenship of the
parties and the amount in controversy are irrelevant. (See III., supra.)

6) Court Approval
The court must approve the dismissal or settlement of a class action. The class
must satisfy the requirements for certification under Rule 23 (a) and (b) before a
court can approve a class settlement (see 1), above). [Amchem Products, Inc. v.
Windsor, 521 U.S. 591 (1997)]

CIVIL PROCEDURE-FEDERAL 57.

a) Notice of Dismissal or Settlement


Moreover, notice of the proposed dismissal or settlement must be given to all
memb manner as directed by the court. [Fed. R. Civ. P.
ers of 23 (e)] This notice is required in all types of class actions, unless the judgment
the will not bind the class. The purpose of notice to class members is to allow them
class to object to the proposed dismissal or settlement when the court holds
in a a "fairness hearing" to determine whether to approve the dismissal or settle ment.
(See below.)

b) Procedures for Settlements of Class Action Su.its


Rule 23(e) requires a settlement hearing (usually called a "fairness hearing") if
the judgment will bind the class and permits settlement only if the court finds
the terms to be fair, reasonable, and adequate. The court must make
a finding supporting its conclusion that the settlement meets that standard.
Parties seeking approval ofa settlement must inform the court ofany collateral
agreements made in connection with the class settlement.

(1) "Opt Out" Provision


The court may refuse to approve a settlement of a Rule 23(b)(3) class
action if members are not provided a new opportunity to opt out. Thus,
members who received notice of the pendency of the class action (see 4),
above) but declined to opt out may be permitted a second opportunity to
opt out, essentially to reject the terms of the settlement and proceed on their
own.

c) Appeal of Approval of Settlement


A class member who objects to the approval of settlement may bring an appeal
of the approval of settlement. [Devlin v. Scardelletti, 536 U.S. 1 (2002)]

7) Appeal of Class Action Certification Decision


Although a court's order granting or denying the certification of a class is not a final
judgment in the case, a party may seek review of the decision in the court of appeals
under Rule 23(£). (See IX.C.6., infra.)

d. Class Action Fairness Act


The Class Action Fairness Act ("CAFA'') relaxes federal jurisdictional requirements
for some class actions in an effort to make it easier for class action plaintiffs to file in
federal court and for class action defendants to remove class actions from state to
federal court. (Congress had concluded that some state courts had certified class actions
inappropriately and that greater access to federal courts would protect defendants from such
perceived abuses.)

1) Subject Matter Jurisdiction Under the CAFA


Under the CAFA, subject matter jurisdiction is established if:

a) Any class member (not just the representative, but anyone in the plaintiff
class) is of diverse citizenship from any defendant;
58. CIVIL PROCEDURE-FEDERAL

b) The amount in controversy in the aggregate (i.e., adding all the class claims
together) exceeds $5 million; and
c) There are at least 100 members in the proposed class or classes.
Removal Under the CAFA
Additionally, in a case falling under the CAFA, any defendant, rather than all
defendants, may remove the case from state to federal court. Moreover, there is no
in-state defendant limitation on removal-the case may be removed under the CAFA
even if a defendant is a citizen of the forum.
3) Excluded Actions
a) Primary Defendants Are States or Governmental Entities
There is no federal court jurisdiction under the CAFA if the primary defen
dants are states, state officials, or other governmental entities against whom the
court may be foreclosed from ordering relief.
b) Claims Based on Securities Laws or Regarding Corporate Governance
There is no federal court jurisdiction under the CAFA over a class action that
solely involves a claim underfederal securities laws, or that relates to the internal
affairs ofa corporation and is based on the laws ofthe state of incorporation.
4) Local Considerations May Defeat Jurisdiction
The CAFA has some provisions designed to defeat federal jurisdiction in class
actions that are relatively local in nature. These provisions contain some unclear terms.
a) Mandatory Decline of Jurisdiction
A district court must decline jurisdiction provided by the CAFA if: (i) more than
two-thirds of the members of the proposed plaintiff class are citizens of the state in
which the action wasfiled; (ii) a defendant from whom "signifi cant relief' is
sought is a citizen of that state; (iii) the "principal injuries" were incurred in the
state in which the action wasfiled; and (iv) no similar class action has been filed
within theprior three years.
b) Discretionary Decline of Jurisdiction
A district court may decline jurisdiction provided by the CAFA if more than
one-third but less than two-thirds of the proposed plaintiff class are citizens
ofthe state in which the action wasfiled and the "primary defendants" are also
citizens ofthat state. In that case, the court considers a variety of factors,
including whether the claims involve matters of national interest, whether the
claims will be governed by the law of the state in which it was filed, and
whether the state has a "distinct nexus" with the class members, the alleged
harm, or the defendants.
5) Protections Underthe CAFA
The CAFA adds a number of protections that apply to settlements in all class
actions in federal court.

CIVIL PROCEDURE-FEDERAL 59.

a) Coupon Settlements
Sometimes, class action settlements provide that the class members are to
receive coupons good for purchase of further goods or services from the
defendant. The court may approve such a settlement only after holding a
hearing and making afinding that the settlement isfair, and it may also require
that unclaimed coupons be distributed to charitable organizations. If attorneys'
fees in such cases are to be based on the value of the settle ment to the class,
they amount available to class members. Alternatively, attorneys' fees can be based
must on the amount of time class counsel reasonably expended on the action.
be
limite b) Protection Against Loss by Class Members
d to
the In some consumer class actions, some class members have actually lost money,
value because attorneys' fee awards required them to pay the lawyers more than they
of the received from the settlement. A court may approve a settlement that would have
coupo that effect only if it makes a written finding that nonmonetary benefits to the
ns class member substantially outweigh the monetary loss.
actuall
y c) Protection Against Discrimination Based on Geographic Location
redee The court may not approve a settlement that provides larger payouts for some class
med members than others solely because the benefitted class members are located closer
by to the court.
class
memb
ers, d) Notification of Federal and State Officials
rather Settling defendants are required to give notice of proposed settlements to identified
than federal and state officials. Final approval of the proposed settlement may not be
the issued until at least 90 days after the notice is served. A class member who
total demonstrates that required notice was not provided may choose not to be bound
by the settlement.

e. Shareholder Derivative Suits

1) Minority Shareholder Allegations


Under Rule 23.1, a minority shareholder, suing on behalf of other minority share
holders to enforce some right of the corporation which the corporation refuses to
assert, must allege in a verified complaint that:

(i) She was a shareholder at the time of the transaction complained of (or
received her shares thereafter by operation of law);

(ii) The action is not a collusive effort to confer jurisdiction on the court that it
would otherwise lack; and

(iii) She made a demand on the directors and, if required by state law, on the
shareholders, or the reasons why she did not make such demands. For this
requirement, facts must be pleaded with particularity.
60. CIVIL PROCEDURE-FEDERAL

Rule 23.1, like Rule 23, requires that the class representative be able to fairly and
adequately represent the class.

2) Corporation Named as Defendant


The corporation must be named as a defendant if those who control the corpora
tion are antagonistic to the action sought by the plaintiffs. If not so named, the
court will align the corporation as a defendant to reflect the antagonism.

3) Jurisdictional Amount and Venue


The judgment runs to the corporation; therefore, the jurisdictional amount looks to
the damages allegedly suffered by the corporation. By statute, venue is proper
wherever the corporation could have sued the same defendants (i.e., usually in the
state of its incorporation). [28 U.S.C. §1401]

4) Court Approval
The court must approve the dismissal or settlement of a derivative suit.
f. lnterpleader
1) Purpose Is to Avoid Double Liability
Interpleader permits a person in the position of a stakeholder to require two or
more claimants to litigate among themselves to determine which, if any,
has the valid claim where separate actions might result in double liability on a
single obligation. Interpleader is available under Rule 22 and under the Federal
Interpleader Statute. [28 U.S.C. §1335]

2) Rights of Plaintiff Stakeholder


The plaintiff stakeholder does not have to admit liability to any claimant and the
claims do not have to have common origin. Once the court has allowed inter pleader,
a trial by jury is available to determine the issues of fact.
3) Jurisdiction
a) Rule 22 Interpleader
If Rule 22 interpleader is relied on, the normal rules as to subject matter
jurisdiction apply. Therefore, there must be either a federal question claim, or
complete diversity between the stakeholder and the claimants and more than
$75,000 in controversy.

b) Federal lnterpleader Statute


Under the Federal Interpleader Statute, on the other hand, the jurisdictional
requirements are less restrictive. The federal statute permits jurisdiction where
the amount in controversy is $500 or more and where there is diver sity
between any two contending claimants. Venue lies where any claimant resides,
and process may be served anywhere in the United States under the statute
(but not under Rule 22). The plaintiff stakeholder must deposit the amount in
controversy (or a bond) with the court.
CIVIL PROCEDURE-FEDERAL 61.

g. Intervention
Intervention may be granted to a party of right or permissively. [Fed. R. Civ. P. 24]

1) Intervention of Right
Intervention of right is available whenever the applicant claims an interest in the
property or transaction that is the subject matter of the action, and the disposition of
the action without her may impair her ability to protect that interest (unless her
interest is already represented). The possible stare decisis effect of a judgment may be
sufficient "interest" to authorize intervention of right. Traditionally, interven tion of right
invoked ancillary jurisdiction, so that no independent basis of subject matter
jurisdiction was required over claims by or against the intervenor of right.
Under the supplemental jurisdiction statute, however, it appears that there is no
supplemental jurisdiction (with limited excpetions that are unlikely to be tested) over
claims by or against one seeking to intervene in a diversity action. The United States has a
right of intervention in all cases where the constitutionality of a United States statute is
raised.

2) Permissive Intervention
Permissive intervention is available when the applicant's claim or defense and the main
action have a question of fact or law in common; no direct personal or
pecuniary interest is required. A claim in permissive intervention must not destroy
complete diversity (if it does, intervention will be denied), and must be supported by
its ownjurisdictional ground. Permissive intervention is discretionary with the court.

3) Caveat
In all cases of intervention, the application must be timely, a matter within the
court's discretion.
h. Third-Party Practice (Impleader)
1) Claims for Indemnity or Contribution
A defending party may implead a nonparty, but only if the nonparty is or may be
liable to her for any part of a judgment that the plaintiff may recover against
her. Usually, such an impleader claim will be for indemnity or contribution. If the
indemnity or contribution claim by the defending party against the third-party
defendant does not meet the requirements for diversity of citizenship or federal question
jurisdiction, it will invoke supplemental (ancillary) jurisdiction, because such claims will
meet the "common nucleus ofoperativefact" requirement
ofsupplemental (ancillary)jurisdiction. (See 11.B.5., 6.e., supra.) Thus, the
defending party may assert an indemnity or contribution claim in federal court even if
there is no diversity between the defending party and the third-party defen dant and the
third-party claim is based on state law. Furthermore, venue need not be proper for the
third-party defendant.
2) Non-Indemnity or Non-Contribution Claims
As part of the third-party complaint, the third-party plaintiff (i.e., the original defending
party) may join other (non-indemnity or non-contribution) claims she
62. CIVIL PROCEDURE-FEDERAL

may have against the third-party defendant. Ifthese other claims cannot invoke
diversity ofcitizenship orfederal questionjurisdiction, they would also need to invoke
supplemental (ancillary)jurisdiction (see 11.B.5., 6.e., supra), although it is less likely
that the "common nucleus" test could be met.
3) Severance of Third-Party Claims
In any event, even i jurisdiction exists, the court may sever any third-party claim
to be tried separately ifit isjust to do so (e.g., if addition of those claims would
f to one of the parties).
lead to unfair prejudice
4) Response of Impleaded Party
After he is joined by the third-party complaint, the third-party defendant may assert
defenses to the plaintiff's original claim, as well as defenses to the third party liability
asserted against him.
5) Impleading Insurance Companies
In some states, a defendant may not implead its own insurance company, but if the
insurance company denies coverage and refuses to defend, then the defendant may
implead the company and have that issue decided in the same case.
i. Cross-Claims
Co-parties may assert claims against each other that arise out of the same transac- tion or
occurrence as the main action by filing cross-claims. Since a cross-claim is, by
definition, transactionally related to the existing action, it is commonly considered to come
within the court's supplemental jurisdiction at least if the claim is by a defendant against a
co-defendant.
H. DISCOVERY
1. Duty of Disclosure
Rule 26 requires parties to disclose certain information to other parties without waiting for a
discovery request. However, Rule 26 also has provisions allowing stipulation of the parties or
court order to modify some disclosure requirements.
a. Types of Disclosure Required
Before making her disclosures, a party has an obligation to make a reasonable inquiry into
the facts of the case. Rule 26 requires parties to disclose all information "then reasonably
available" that is not privileged or protected as work product. A party is not relieved from
her obligation to disclose merely because she has failed to complete her investigation or
because another party has not made his disclosures or has made inade quate disclosures.
Three types of disclosure are required: initial disclosures, disclosure of expert testimony,
andpretrial disclosures.
1) Initial Disclosures
Without waiting for a discovery request, a party must provide to other parties (unless
stipulation or court order provides otherwise):
(i) The names, addresses, and telephone numbers of individuals likely to have
discoverable information that the disclosing party may use to support its
claims or defenses, unless the use would be solely for impeachment;
CIVIL PROCEDURE-FEDERAL 63.

(ii) Copies or descriptions of documents, electronically stored information, and


tangible things that are in the disclosing party's possession or control and that the
disclosing party may use to support its claims or defenses, unless the use would
be solely for impeachment;

(iii) A computation of damages claimed by the disclosing party and copies of


materials upon which the computation is based; and

(iv) Copies of insurance agreements under which an insurer might be liable for all or
part of any judgment that might be entered.

These disclosures must be made within 14 days after the meeting of the parties
required by Rule 26(f) (discussed at 1. 1., infra) unless a different time is set by
court order or by stipulation.

a) Exemptions from Initial Disclosure Requirement


Initial disclosures are not required in particular types of cases, such as actions to
review an administrative record, actions to enforce an arbitration award, pro se
litigation brought by prisoners, actions to quash or enforce subpoenas, or habeas
corpus petitions. [Fed. R. Civ. P. 26(a)(l)(B)]

2) Disclosure of Expert Testimony


A party must also disclose to other parties the identities of expert witnesses expected
to be used at trial. If the expert witness has been specially retained to provide expert
testimony, or if the expert's duties as the party's employee regularly involve giving
expert testimony, the disclosure must be accompanied by a report prepared and signed
by the expert witness stating her qualifications, the opinions to be expressed, and the
basis for those opinions, along with a list of cases in which he has testified in the
past four years and his compensation, among other items. Otherwise, no report is
required, and instead these disclosures are required: the identity of the expert, the
subject matter on which the expert is expected to present evidence, and a summary of
the facts and opinions to which the witness
is expected to testify. This disclosure must be made at the time directed by the court
or, in the absence of any directions or any stipulations among the parties, at least 90
days before trial; if the evidence is intended solely to rebut another party's disclosure
of expert testimony, it must be made within 30 days after disclosure of the evidence
being rebutted.

3) Pretrial Disclosures
At least 30 days before trial, a party must disclose to the other parties and file with
the court a list of (i) the witnesses she expects to call at trial, (ii) the witnesses she
will call if the need arises, (iii) the witnesses whose testimony will be presented by
means of a deposition and a transcript of pertinent portions of the deposition, and
(iv) documents or exhibits she expects to offer or might offer if needed. Evidence or
witnesses that would be used solely for impeachment need not be disclosed. Within
14 days after this disclosure, a party may serve objections to use of the depositions
at trial and to the admissibility of disclosed documents and exhibits. Such objections
are waived if not made at this point, except for objections that the
64. CIVIL PROCEDURE-FEDERAL

evidence is irrelevant, prejudicial, or confusing under Federal Rules of Evidence 402


and 403.
Discovery of Electronically Stored Data
The Rules require parties to discuss the discovery and preservation of electronically stored data
and to report to the court on those discussions. Electronically stored information need not be
produced if the responding party identifies it as from a source not reasonably acces sible
because of undue burden or cost. On motion to compel or for a protective order, that party must
show to the court's satisfaction that its assertion is justified. Even then, the court may order the
information produced for good cause, but it may also impose conditions such as cost-shifting or
cost-sharing. [Fed. R. Civ. P. 26(b)(2)(B)]

a. Format for Producing Electronic Documents


A requesting party may specify the form or forms for producing electronically stored
information, and the responding party must use that form unless it objects (but the party
must still produce other items to which it has no objections). The court will determine if
the objection is valid. If the request does not specify the form for producing electroni cally
stored information, the responding party may use any form in which the informa tion is
maintained or a form that is reasonably usable by the requesting party. [Fed. R. Civ. P.
34(b)]
b. Destruction of Electronically Stored Information
Rule 37(e) creates remedies for discovering parties and sanctions against parties who fail
to take reasonable steps to preserve electronically stored information when litiga tion is
reasonablyforeseeable. (If reasonable steps were taken, or if litigation was not reasonably
foreseeable, no sanctions may be imposed for the destruction of discover able
information.) Additionally, if unpreserved electronically stored information can
be restored or adequately replaced through additional discovery, no other remedial
measures need be imposed. If the information cannot be restored or replaced, and the
discovering party is prejudiced by the lack of information, the court may order
remedial measures no greater than necessary to cure the prejudice, taking into
consideration the importance of the information to the case and the expenses resulting
from restoring or replacing the information. Ifthe disclosing party acted with intent to
deprive the discovering party ofthe information, the court may (i) presume the lost
informa-
tion was unfavorable to the party; (ii) instruct the jury that it may or must presume the
information was unfavorable to the party; or (iii) dismiss the action or enter a default
judgment. The imposition of these measures is discretionary with the court.
3. Scope of Disclosure and Discovery
a. In General
Unless the court limits discovery by order, discovery may be had of "any nonprivileged
matter that is relevant to any party's claim or defense" and proportional to the needs of
the case, considering the importance of the issues at stake in the action, the amount in
controversy, the parties' relative access to information, the parties' resources, the importance
of the discovery in resolving the issues, and whether the burden or expense
of the proposed discovery outweighs its likely benefits. ''Any matter" encompasses both
documentary evidence and individuals with knowledge of any discoverable matter.
CIVIL PROCEDURE-FEDERAL 65.

Furthermore, information need not be admissible at trial to be discoverable. [Fed. R.


Civ. P. 26(b)(l)]
b. Trial Preparation Materials
Work product of a party or a representative of a party (e.g., a lawyer) made in antici
pation oflitigation, is discoverable only upon showing "substantial need" and to avoid
"undue hardship" in obtaining materials in an alternative way. If the court orders the
disclosure of work product, it must take steps to avoid the disclosure of mental
impressions, conclusions, opinions, or legal theories of the disclosing party.
However, a party may obtain, without a court order and without showing need and hardship,
a copy of any statement or recording previously made by that party. Draft reports and draft
disclosures of "trial" experts are work product. Confidential commu nications between such
experts and counsel for the party are generally protected
as trial preparation materials, except for communications relating to the expert's
compensation or to facts or data the attorney provided to the expert. [Fed. R. Civ. P. 26(b)]

1) Procedure for Claiming Work Product


When a party claims that certain discoverable information is privileged trial
preparation material, he still must disclose the existence of the material in suffi cient
detail to the opposing party so that the opposing party may assess the claim of
privilege.
c. Inadvertent Disclosure of Trial Preparation or Privileged Materials
If a party inadvertently discloses trial preparation or privileged material to opposing
parties, he may still invoke a claim of work-product protection or any privilege by
notifying the opposing parties of the inadvertent disclosure and the basis for the claim of
work product or privilege. Once so notified, the opposing party may not use or disclose
the trial preparation or privileged material until the claim is resolved, and he must take
reasonable steps to retrieve the material if he disclosed it to others.
d. Experts
A party may depose experts who are expected to be called at trial (testifying experts). If a
report from the expert is required, the deposition may take place only after the report is
provided. The opinions of experts who are retained in anticipation of litigation but who are
not expected to testify at trial (consulting experts) may be discovered only upon a showing
of exceptional circumstances under which it is impracticable to obtain facts or opinions by
other means. [Fed. R. Civ. P. 26(b)(4)]

e. Protective Orders
Protective orders may be obtained under Rule 26 (c) to limit the nature and scope of
examination or to terminate examination if discovery is abused. The court also may
allocate the expenses of discovery among the parties.
Supplementation of Disclosures and Discovery Responses
A party must timely supplement required disclosures and prior responses to interroga
tories, requests for production, or requests for admissions if she learns that the informa tion
disclosed was materially incomplete or incorrect and the new information has not
66. CIVIL PROCEDURE-FEDERAL

been made known to the other party in discovery or in writing. The duty to supplement also applies to an expert's
reports and information from any deposition of an expert. [Fed. R. Civ. P. 26(e)]
4. 'fypes of Discovery
a. Pre-Action Depositions
Prior to a lawsuit being filed, or while an appeal is pending, a potential party or party to an appeal may ask the
court to order the deposition of any person in order to perpetuate her testimony. To do so, the potential party
must file a verified petition in the federal court for the judicial district in which any expected adverse
party resides.
1) Contents of Petition
The request for a court order is included within the petition itself. The petition also must show that,
among other things, the petitioner expects to be a party to an
action cognizable in a court in the United States but is presently unable to bring it or cause it to be
brought. All expected adverse parties must be named.
Notice and Appointed Counsel
At least 21 days before the hearing date for the court order, the potential party must serve each expected
adverse party with a copy of the petition and a notice of hearing. The manner of service is the same as for
an original petition (see VII.B., supra). If the expected adverse party cannot be so served, the court must
appoint counsel for that party.
3) Court Order
If the court finds that ordering a deposition may prevent a failure or delay of justice, it will issue an
order that specifies the person being deposed, the subject matter ofthe deposition, and the manner of
the deposition.
b. Depositions
A party may not take more than 10 depositions, nor may she depose the same person more than once, without leave
of court or stipulation of the parties.
1) Oral Deposition of a Witness, Including a Party-Witness
A common form of discovery is the oral deposition under Rule 30. If the deponent is not available at trial,
the deposition may be used in lieu of her appearance as
a witness. The deposition may be recorded by sound, sound and visual, or steno graphic means.
Depositions may be taken by telephone or through other remote electronic devices. All parties may pose
questions to the deponent. A deposition may not exceed "one day of seven hours" absent court order or
stipulation to the contrary.
a) Compulsory Appearance of Witnesses
(1) Subpoena Not Needed for Parties
It is not necessary to serve a subpoena on an adverse party or an officer, director, or
managing agent of a party to compel appearance; the notice
CIVIL PROCEDURE-FEDERAL 67.

of deposition is sufficient to compel attendance. For organizations, the


notice may name the organization and state with "reasonable particu larity"
the matters to be covered. The organization then designates individuals to
testify. [Fed. R. Civ. P. 30(b)(6), 37]

(2) Nonparties Should Be Subpoenaed


If the witness to be deposed is not a party to the action, he should be
subpoenaed. The subpoena may be served by any person who is not a party
and is not less than 18 years old. Service is made by delivering a copy
ofthe subpoena with any necessary fees to the person named in the
subpoena. A nonparty organization may be required by subpoena to
designate individuals to testify, as in (1), supra. A subpoena may
command a person to appear only (i) within 100 miles from where he
resides, works, or regularly transacts business in person; or (ii) within the
state in which he resides, works, or regularly transacts business in
person so long as he would not incur substantial expense. A subpoena is
enforceable in the court it was issued or the court in which the action is
pending on transfer from the issuing court. [Fed. R. Civ. P. 30, 45]

(3) Costs When Notifying Party Fails to Attend


When the party who notices the deposition does not appear (in person or by an
attorney) to take the deposition, and the other party does appear, the latter can
obtain his costs of attending, including reasonable attor neys' fees.

2) Deposition of Witnesses on Written Questions


Rule 31 provides for written questions to witnesses (including parties) and is
designed to facilitate the depositions of witnesses living a great distance from the
parties. All parties can pose questions to the deponent, which must be served on all
parties before the deposition.

c. Interrogatories to the Parties


Rule 33 provides for written interrogatories to other parties and written answers by the party to
whom the interrogatories are directed. The party must respond not only with facts which
she herself knows, but also with facts that are available to her. The party may also be
asked to give opinions, even on the application of law to facts. Initially, the requesting
party may not serve more than 25 interrogatories including subparts without court order or
stipulation, and leave may be granted to serve additional ones.

1) Option to Produce Business Records


If the answer to an interrogatory may be ascertained from the responding party's business
records (including electronically stored information), and the burden of ascertaining the
answer will be essentially the same for the parties, the responding party may (i) specify the
records that must be reviewed, in sufficient detail to enable the requesting party to locate
and identify them as readily as the responding party could; and (ii) give the interrogating
party a reasonable opportunity to examine and audit the records and to make copies,
compilations, abstracts, or summaries.

68. CIVIL PROCEDURE-FEDERAL

d. Production of Physical Material; Inspection


Rule 34 provides (i) for the production by aparty (or, if accompanied by a subpoena, a
nonparty) of physical material, including documents and electronically stored infor
mation, relevant to the pending action; and (ii) that a party be required to permit entry
onto land for relevant testing. Notice must be given to all other parties before a nonparty
subpoena issues. A subpoena is limited to production at a place within 100 miles of where
the person resides, works, or regularly transacts business in person.

e. Physical and Mental Examinations


1) Order for Examination
Rule 35 provides for an independent physical or mental examination of a party
when that party's physical or mental condition is in controversy. Such exam is
available only if ordered by the court, on showing of good cause. Traditionally, the
Rule has allowed exams only by "physicians." Now, however, it allows exams by a
"suitably licensed or certified examiner," which would include, for example,
doctors, dentists, occupational therapists, and any others required to be licensed and
qualified to comment on a physical or mental condition.

2) Report of Findings
The person examined may request a copy of the examiner's report, but if that
person so requests or takes a deposition of the examiner, she waives any privilege
and must produce, upon demand, copies of her own doctor's reports of any other
examinations of the same condition.

f. Requests for Admission


Any party may serve on any other party a written request for admission as to the truth or
genuineness of any matter or document described in the request. The matters will be
considered admitted unless the party upon whom the request was served returns a sworn
statement denying the truth of the matters set forth in the request, or explaining why she
cannot admit or deny them. Alternatively, the party upon whom the request
was served can file written objections to those requests that she has a legal basis for not
answering. A party may be asked to admit matters that are genuine issues for trial. The
admission is for the purpose of the pending action only and may not be used against the
party in any other proceeding. [Fed. R. Civ. P. 36]

5. Enforcing Disclosure and Discovery


a. Motion for an Order Compelling Disclosure or Discovery
On notice and motion, a party may seek an order compelling discovery from the court.
(An evasive or incomplete response to discovery essentially is treated as a failure to make
discovery.) The movant must certify that he has, in good faith, conferred or attempted to
confer with the person or party failing to make discovery in an effort
to avoid court intervention. If the motion is to compel a party to make discovery, the
motion must be filed in the court in which the action is pending; if the order is sought
against a nonparty, the motion must be made in the court in which discovery will be taken.
[Fed. R. Civ. P. 37(a)]

1) CIVIL PROCEDURE-FEDERAL 69.

Motion Granted or Discovery Is Provided After Filing


If the motion is granted, or if the discovery is provided after the motion was filed
(and, of course, after the movant conferred or attempted to confer to resolve the
2) dispute), the court must, after giving an opportunity to be heard, require the party or
deponent whose conduct necessitated the motion, the party or attorney advising that
conduct, or both, to pay the moving party's reasonable expenses
incurred in making the motion, including attorney's fees, unless the nondisclosure
was substantiallyjustified or other circumstances exist that make an award of
expenses unjust.
Motion Denied
If the motion is denied, the court may issue a protective order and must, after giving
an opportunity to be heard, require the movant and/or the attorney filing the motion to
pay the deponent's reasonable expenses, including attorney's fees, incurred in opposing the
opposing motion unless the motion was substantiallyjustified or other circumsta.nces exist that
party's or make an award ofexpenses unjust.

3) Motion Granted In Part and Denied In Part


If the motion is granted in part and denied in part, the court may issue a protec tive
order and/or, after giving an opportunity to be heard, apportion the reasonable
expenses for the motion.

b. Failure to Comply with a Court Order


A party or deponent may be held in contempt of court for failing to obey a court order
(except for an order for a mental or physical examination). There are further sanctions that
may be available against a party. The court in which the action is pending may:
(i) take designated facts as true for the purposes of the action; (ii) prohibit the nondis
closing party from supporting designated claims or defenses; (iii) strike pleadings; (iv) stay
the proceedings until the order is obeyed; (v) dismiss the action in whole or in part;
(vi) render a default judgment against the nondisclosing party; and/or (vii) order the party
and/or his attorney to pay reasonable expenses, including attorney's fees. [Fed. R. Civ. P.
37(b)]

c. Failure to Disclose or Supplement an Earlier Response


If a party fails to provide information or identify a witness, the party is not allowed to
use that information or witness to supply evidence in a motion, or at a hearing or at
trial, unless the failure was substantiallyjustified or harmless. In addition to or
instead of this sanction, the court, on motion, may (i) order payment of the reasonable
expenses, including attorney's fees, caused by the failure; (ii) inform the jury of the
party's failure; and (iii) impose other appropriate sanctions. A motion to compel is not
required for these sanctions. [Fed. R. Civ. P. 37(c)(l)]
d. Failure to Admit
If a party fails to admit a fact that is the subject of a request for admissions, and the
requesting party later proves the matter true, the requesting party may move that the
party who failed to admit pay the reasonable expenses, including attorney's fees,
incurred in making that proof. The court must order payment of expenses unless: (i)
the request was held objectionable; (ii) the admission sought was of no substantial
70. CIVIL PROCEDURE-FEDERAL

importance; (iii) the party failing to admit had a reasonable ground to believe that it
might prevail on the matter; or (iv) there was other good reason for the failure to admit. A
motion to compel is not required for this sanction. [Fed. R. Civ. P. 37(c)(2)]

e. Party's Failure to Attend His Own Deposition, Serve Answers to Interrogatories, or


Respond to a Request for Inspection
If a party, after being served with proper notice, fails to attend his own deposition, or if a
party fails to provide any response to interrogatories or a request for inspection, the
requesting party may, instead of moving to compel, apply for immediate sanctions against
the nondisclosing party. However, the movant (requesting party) must certify that he made
a good faith attempt to resolve the discovery dispute. That a request is objectionable does
not serve as a defense unless the nondisclosing party has a pending motion for a
protective order. Permissible sanctions are the same as those described above. [Fed. R.
Civ. P. 37(d)]
6. Use of Depositions at Trial or Hearing
Subject to the rules of evidence, a deposition may be used (at trial or in a hearing) against any
party who was present at the deposition or had notice of it:
(i) To impeach the testimony of the deponent as a witness;
(ii) For any purpose if the court finds that the deponent (including a party-deponent) is dead,
at a distance greater than 100 miles from the place of trial (unless the absence was
procured by the party offering the deposition), unable to testify because of age,
sickness, or imprisonment, or the party offering the deposition could not procure the
witness's attendance by subpoena;
(iii) For any purpose if, on notice and motion, thejudge permits the deposition to be so used
due to exceptional circumstances; or
(iv) For any purpose if the deponent is an adverse party.
[Fed. R. Civ. P. 32]

7. Errors and Irregularities in Depositions


Rule 32 governs the waiver of errors and irregularities in the taking of depositions.

a. As to Notice
Errors and irregularities relating to the notice of deposition are waived unless written objection
is promptly served on the party giving notice.
b. As to Manner of Taking
Errors of any kind which could have been obviated if promptly presented are waived
unless seasonable objection is made at the time of taking the deposition (applies to form
of questions, oath, conduct of parties, etc.).
c. As to Completion and Return
Errors and irregularities as to the completion and return of the deposition are waived
unless a motion to suppress is made with reasonable promptness after the error was or
should have been discovered (applies to signing, sealing, certification, and transmittal).

CIVIL PROCEDURE-FEDERAL 71.

d. As to Form of Written Questions


Objections to the form of written questions are waived unless served on the party
propounding them within the time for serving succeeding questions and within five days after
service of the last questions authorized.
I. PRETRIAL CONFERENCES
1. Rule 26(f) Conference of Parties-Planning for Discovery
As soon as practicable, and in any event before the Rule 16(b) scheduling conference (see below),
the parties must confer to consider their claims and defenses, the possibility of settlement,
initial disclosures, any issues concerning the preservation of evidence, and a discovery plan.
The parties must submit to the court a proposed discovery plan within 14 days after the conference
addressing the timing and form of required disclosures, the
subjects on which discovery may be needed, the timing of and limitations on discovery, and relevant
orders that may be required of the court.

2. Rule 16(b) Scheduling Conference


The court must (except in classes of cases exempted by local rule) hold a scheduling confer ence
among the parties or counsel. The scheduling order will limit the time forjoinder, motions, and
discovery. The order may also include dates forpretrial conferences, a trial date, and any other
appropriate matters. This schedule cannot be modified except by leave of court upon a showing of
good cause. [Fed. R. Civ. P. 16(b)]

3. Pretrial Conferences
The court may also hold pretrial conferences as necessary to expedite trial and foster settle ment.
A.finalpretrial conference, if any, is held as close to the time of trial as reasonable, and is for the
purpose of formulating a plan for the trial, including the admission of evidence. This conference is
to be attended by at least one of the lawyers for each side who will actually be conducting the
trial, and by any unrepresented parties. After a pretrial confer ence, an order must be entered that
controls the subsequent course ofevents in the case.
Thus, the final pretrial conference order is a blueprint for the trial, usually listing witnesses to be
called, evidence to be presented, factual and legal issues needing resolution, and like matters. It is
thus said to supersede the pleadings and may be modified only to prevent manifest injustice.

4. Sanctions
A party or counsel may be sanctioned for failure to attend a conference or obey an order entered
pursuant to a conference, for being substantially unprepared to participate in a conference, or for
acting in bad faith. The court has a broad range of available sanctions including contempt, striking
pleadings, and prohibiting the introduction of evidence. In addition, the court must require the
disobedient party or counsel to pay expenses incurred (including attorneys' fees) by other parties,
unless the court finds that circumstances make such an award unjust.

J. ALTERNATIVE DISPUTE RESOLUTION


Alternative dispute resolution (''ADR") is a process in which a neutral person resolves a dispute or helps the
parties to resolve their dispute. Examples of these processes include contractual arbitra tion, judicial
arbitration, and mediation.
72. CIVIL PROCEDURE-FEDERAL

1. Contractual Arbitration
The Federal Arbitration Act ("FAA'') governs written arbitration agreements involving inter state
or international commerce and preempts conflicting state law. [9 U.S.C. §§1 et seq.]
a. Procedure
A written agreement to arbitrate a dispute is valid and enforceable unless a contractual
ground for revocation exists (e.g., fraud in the inducement of the arbitration clause,
illegality or unconscionability of the arbitration clause). Court proceedings are stayed until
the arbitration proceedings are completed. The appointment of the arbitrator usually will be
provided for in the arbitration agreement. At the arbitration proceeding, the arbitrator can
subpoena witnesses and require them to bring documentary evidence to the hearing. After
the arbitrator renders the award, a party can move to have the court confirm the award. The
opposing party may move to vacate the arbitration award on the grounds below. If the
award is confirmed, it is considered to be final and binding, and it is enforceable as a court
judgment.
1) Judicial Review of Award
An arbitration award may be vacated, even on appeal, only on narrow statutory grounds,
such asfraud or evident partiality ofthe arbitrator, the arbitrator's refusal to delay
proceedingsfor sufficient cause, or the arbritrator'sfailure to follow the arbitration
agreement to such a degree as to affect the outcome. A party may also move to modify
the award to co"ect miscalculations, to modify awards that go beyond the scope ofthe
arbitration agreement, or to correct minor imperfections ofform. Additionally, an
arbitration award also may be overturned when it represents a manifest disregard ofthe
law, a judicially created
and extremely deferential standard that requires the complaining party to show that the
arbitrator knew the applicable law but chose to disregard it.
2. Judicial Arbitration
"Judicial arbitration" is a dispute-resolution process conducted by a neutral person under the
auspices of the court in an attempt to resolve the action without trial. Judicial arbitration may be
employed by federal courts under local district court rules. At the federal level, participation in
arbitration procedures is voluntary, and even then certain actions (cases involving a violation of
constitutional rights, certain civil rights actions, and cases alleging an amount in controversy
of more than $150,000) may not be referred to ADR even if the parties consent. Within 30 days
of the arbitration award, a party may reject the award and request trial de novo. Any evidence that
the case was arbitrated is generally excluded. [28 U.S.C. §§654 et seq.]

3. Mediation
Mediation involves the use of a neutral person to help parties to a dispute reach a mutually
acceptable agreement. The mediator does not have decisionmaking power; his role is to facilitate
the process by which the parties reach their own voluntary agreement. In federal court, mediation
is accomplished by local district rule. By local rule, mediation may be made mandatory for
certain cases. [28 U.S.C. §§ 651, 652]
K. TRIAL
1. Jury Trial Problems
Rule 38 requires a party who desires a jury trial (on some or all fact issues) to file a written
CIVIL PROCEDURE-FEDERAL 73.

demand with the court and serve it on the parties. (Such demand may be indorsed upon a
pleading ofthe party.) Failure to make such a demand within 14 days after the service of the
last pleading directed to the jury-triable issue constitutes a waiver by that party of any right to
trial by jury. A court may, within its discretion, order a trial by jury if the plaintiff's waiver was
not intentional. In the absence of compelling reasons to the contrary, a court should grant relief
from waiver if the issue is one normally tried by a jury. [Cox v. Masland & Sons, Inc., 607
F.2d 138 (5th Cir. 1979)] A jury demand may be withdrawn only if all parties consent.

a. Right to Jury Trial


The Seventh Amendment preserves the right to a jury trial in federal courts of facts in all
"suits of common law" where the amount in controversy exceeds $20. The determi nation
is historical and turns initially on whether the claim or relief was available at law or in
equity in 1791. The Supreme Court has demonstrated a clear preferenceforjury trial in
doubtful cases by holding that:
1) If legal and equitable claims are joined in one action involving common fact issues,
the legal claim should be triedfirst to thejury and then the equitable claim to the court
(the jury's finding on fact issues will bind the court in the equitable claim);

2) If a procedure formerly available only in equity, such as a class suit, interpleader, or


derivative action, is now permitted under the Federal Rules for determining a
"legal" claim, ajury should try thefact issues;
3) If damages are claimed as part of an action seeking an injunction, the defendant
cannot be denied ajury on the damages issues on the ground that they are
"incidental" to the equitable relief; and

4) If a new claim is created that did not exist at common law, a right to a jury trial will
exist if the claim is similar to a claim for common law rights and remedies, unless
the statute creating the right provides otherwise. [See, e.g., Feltner v. Columbia
Pictures Television, Inc., 523 U.S. 340 (1998)-statutory damages under Copyright
Act to be tried to jury]

b. Jury Trials in Diversity Cases


1) Right to a Jury Trial
The federal court mustpermit ajury trial in any diversity "suit at common law" even
though the state court would deny a jury (the Seventh Amendment prevails over state
law in Erie situations); and a federal court will generallyfollow the federalpractice of
submitting issues of fact to the jury even though the state
law assigns the issue to the court. [Byrd v. Blue Ridge Electric Cooperative, Inc.,
11.D.2.b., supra] If the state rule requires submission of a fact issue to the jury,
the federal court may nonetheless direct a verdict under the usual standards or
otherwise follow a federal practice that calls for the court to be the trier of fact.
Likewise, state law is disregarded in determining the sufficiency of the evidence to
create a jury issue; i.e., the directed verdict standards are always federal.

74. CIVIL PROCEDURE-FEDERAL

2) Motion for New Trial Based on Excessiveness of Verdict


The Supreme Court has required federal trial courts to apply a state standard when
considering a motion for a new trial based on excessiveness of the verdict.
[Gasperini v. Center for Humanities, Inc., II.D.3., supra] Under the Seventh
Amendment, federal appellate review of whether a trial court properly denied
a motion to set aside a verdict as excessive is limited to whether the trial court
abused its discretion in denying the motion. In contrast, a jury's determination of
the amount of a punitive damage award is reviewed de novo on appeal. [Cooper
Industries, Inc. v. Leatherman Tool Group, Inc., 532 U.S. 424 (2001)]
c. Jury Size
In federal civil cases, a jury must have at least six jurors and not more than 12 jurors.
[Fed. R. Civ. P. 48] There is no provision for alternate jurors.
d. Jury Selection
1) Jury Venire
Jury selection process begins with individual potential jurors being summoned to appear
in court. At this stage, the potential jurors together are called a "venire," and the
jurors who will actually hear the case are chosen from the venire. The venire must
be a reasonable cross-section of the community.
2) Voir Dire
Typically, the potential jurors will be asked to fill out a questionnaire to discover if
the juror has some potential bias in the case (e.g., whether the juror knows any of
the parties or attorneys involved in the case, whether the juror has been involved in a
similar case, whether the juror owns stock in one of the parties, etc.). The juror likely
will be asked questions individually also. This process is known as "voir dire."
3) Jury Challenges
If the questioning of a potential juror reveals that the juror is biased, that juror may be
excused/or cause. Bias may be actual or implied. Actual bias may be inferred by the
juror's deliberately lying during voir dire. Implied (or presumed) bias
arises when it is very unlikely that an average person in the juror's shoes would be
impartial. Examples of implied bias include when the juror is closely related to one
of the parties or attorneys, when thejuror is employed by one of the parties, when the
juror was the victim of a similar crime or suffered a similar injury, or when
the juror has a financial stake in one of the parties. In such cases, the juror must be
excused, even if the juror states that she can be fair and impartial.
e. Jury Instructions
At the close of the evidence, or sooner at the court's direction, a party may file proposed
instructions. If a party fails to object to an instruction or the failure to give an instruc tion
before thejury retires to consider a verdict, any issue is waived on appeal. [Fed. R. Civ. P.
51]
f. Jury Deliberations
Jurors may take into the jury room allpapers or exhibits in evidence and their own
CIVIL PROCEDURE-FEDERAL 75.

notes. Instructions, pleadings, or other matters are generally improper for use in the jury
room, except when they are formally admitted into evidence. A jury may not engage in
experiments in the jury room, and jurors may not make private studies of documents or items
outside of the jury room. Jurors may not view property or places involved in the case, except
by court order. Jurors must not communicate with any nonjuror regarding the trial; in fact,
any private communication between jurors and counsel or parties is serious misconduct that
may lead to a new trial. It is error for a juror, in the jury room, to state facts not in
evidence; however, jurors are entitled to evaluate evidence presented in light of their general
knowledge and experience.
g. Jury Verdicts
The jury verdict must be unanimous unless the parties agree otherwise. The trial court
has discretion to decide the type of verdict to be used (e.g., general vs. special; see
below). [Fed. R. Civ. P. 49] Jurors cannot decide a verdict by flipping a coin or
averaging (although averaging may be proper as a starting point for discussion). A juror
may be excused for good cause (e.g., illness) without causing a mistrial, so long as at least
six jurors participate in reaching the verdict.

1) General Verdict
In a general verdict, the jury finds for the plaintiff or defendant and gives the amount
of damages or relief due. A general verdict implies that all essential issues were
found in favor of the prevailing party.

2) Special Verdict
In a special verdict, a jury is asked to make a finding on all material conclusions of
fact, and the court applies the law. The procedure for a special verdict is to submit to
the jury a series of questions regarding each ultimate fact. The court then makes legal
conclusions based on those facts. Each question must deal with a single fact only and
must not assume the existence of facts in dispute. A party waives objec tions to the
form of the questions if she does not object when they are given. If
the court fails, on request, to submit an issue to the jury, the case will be reversed
unless the omission was harmless. If no request was made, ajury trial on the issue
is generally held to be waived, and the court will decide it. [See Fed. R. Civ. P. 49]
3) General Verdict with Special Interrogatories
In a general verdict with interrogatories, the jury is asked to give a general verdict and
also to answer specific questions concerning certain ultimate facts in the case. The
purpose is to ensure that thejury properly considered the important issues.
Interrogatories must be submitted with the general verdict to test the verdict's validity.
[See Fed. R. Civ. P. 49(b)]

4) Erroneous Verdicts and Inconsistent Findings


Inconsistent determinations in a verdict make the verdict erroneous if the deter
minations are irreconcilable (e.g., when a verdict is rendered against a person
vicariously liable and the principal wrongdoer is exonerated) . Any clear compro
mise verdict is also erroneous, as is any verdict that simply finds for the plaintiff
"for actual damages suffered." When a verdict is erroneous, the jury may be asked
to reconsider it, or a new trial may be ordered. When answers to special
76. CIVIL PROCEDURE-FEDERAL

interrogatories are consistent with each other, but one or more is inconsistent with the
general verdict, the court may (i) enter judgment in accord with the special
interrogatories; (ii) have the jury reconsider its answers or verdict; or (iii) order a
new trial. When the answers to the special interrogatories are inconsistent with each
other and one or more is inconsistent with the general verdict, judgment cannot be
entered, and the court must either have the jury reconsider its answers and verdict, or
the court may order a new trial. A jury may completely change its answers or verdict
when redeliberation is ordered.

5) Juror Misconduct
A new trial is appropriate if a juror gavefalse testimony on voir dire or concealed
materialfacts relating to his qualifications to serve. A verdict will not be set aside if
the alleged misconduct was harmless. Nonjurors may give evidence of misconduct
except as to declarations ofjurors to them. Under Federal Rule of Evidence
606(b),jurors may not testify as to any matter occurring during delib erations, except on
the question of whether extraneous prejudicial information was improperly brought to
the jury's attention, or whether any outside influence was brought to bear on any juror.

2. Consolidation and Separate Trials


Rule 42(a) allows the court to consolidate actions then before it when the actions have a
common question of law or fact. Rule 42(b) allows the court to order separate trials of any
claim, cross-claim, counterclaim, or other issues when such separation will foster judicial
economy.

3. Involuntary Dismissals
On the defendant's motion or on its own motion, a court may order an involuntary dismissal
against a plaintiff for failure to: (i) prosecute; (ii) comply with the Federal Rules; or (iii) comply
with a court order. [Fed. R. Civ. P. 4l(b)] An involuntary dismissal is with prejudice, meaning that
it operates as adjudication on the merits, unless the court orders otherwise.

4. Voluntary Dismissals
The plaintiff can give up his case voluntarily by way of a voluntary dismissal, either with or
without leave of court. [Fed. R. Civ. P. 4l(a)]

a. Without Leave of Court


If the defendant has not answered or filed a motion for summary judgment, the plaintiff
may dismiss her case by filing a notice of dismissal. A dismissal by notice is without
prejudice unless the plaintiff has previously dismissed any federal or state court action on
the same claim, in which case the dismissal by notice is with prejudice. (This is known as
the "two dismissal rule.") The parties may also stipulate to a voluntary dismissal at any
time. A stipulated dismissal is without prejudice unless the stipulation states otherwise.
The plaintiff is charged with costs only if she files the action again after the dismissal.
b. With Leave of Court
When a voluntary dismissal without leave of court is not available (i.e., there has been an
answer, motion, or previous dismissal), the court has discretion to grant dismissal on
CIVIL PROCEDURE-FEDERAL 77.

such terms and conditions as the court deems proper. The dismissal is without prejudice
unless the court specifies otherwise. If there is a counterclaim pending in the action, there
can be no dismissal over the defendant's objection unless the counterclaim remains pending.
5. Offer of Judgment
Under Rule 68, a party defending against a claim or counterclaim may serve, at least 14 days
before trial, aformal offer to have ajudgment entered against it on specified terms with costs
then accrued, thereby settling the case out of court. A defending party may also serve an offer
ofjudgment after it has been determined to be liable to another party, but before actual damages
have been set; such an offer must be made at least 14 days before the hearing on damages. If
the claiming party rejects the offer, and the ultimate judgment is less favor able to him, he must
pay costs incurred after the offer was made. So long as the offer does not explicitly or implicitly
exclude costs, it is a valid offer. Evidence of an unaccepted offer is not admissible except in a
proceeding to determine costs. Unlike the practice in a number of states, Rule 68 allows only
defending parties to make a formal offer to have a judgment entered voluntarily.

6. Summary Judgment
a. Standard
Summary judgment may be granted if, from the pleadings, affidavits, and discovery
materials, it appears that there is no genuine dispute ofmaterialfact and the moving party
is entitled to judgment as a matter of law. [Fed. R. Civ. P. 56] The court may not decide
disputed fact issues on a motion for summary judgment; if there is a genuinely disputed
material fact (meaning a dispute backed by evidence on both sides of the issue), the case
must go to trial.

b. Applicable to All Civil Actions


Rule 56 applies to all parties and civil actions that are subject to the Federal Rules
including actions by and against the United States, and to all types of claims that appear in
a civil action (counterclaim, cross-claim, declaratory judgment, injunction, and inter
pleader).

c. Time
Unless local rule or court order dictates otherwise, a party may file a motion for summary
judgment any time until 30 days after close of all discovery. [Fed. R. Civ. P. 56(b)] If a
motion is premature, the court may defer ruling on it.

d. Partial
Summary judgment may be partial (as well as complete) .
Example: Summary judgment may be rendered on the issue of liability alone
although there is a genuine issue as to the amount of damages.

e. Support
The motion may be supported or opposed with affidavits or other declarations made
under penalty of perjury, depositions, sworn pleadings, admissions, answers to
interrogatories, or other materials in the record.

78. CIVIL PROCEDURE-FEDERAL

f. Affidavits
1) Affidavits or declarations must: (i) be made onpersonal knowledge; (ii) set forth
such facts as would be admissible in evidence; and (iii) show the affiant is compe
tentto testify.
2) A party may object that the material cited to support or dispute a fact cannot be
presented in aform that would be admissible in evidence.
3) If a party fails to support an assertion of fact or fails properly to address another
party's assertion of fact, the court may consider thefact undisputed for purposes of
the motion, grant summaryjudgment if appropriate, give an opportunity to address
the fact, or issue any other appropriate order.

4) When the party opposing the motion shows by affidavit or declaration that he cannot
present facts, he may state the reasons for their unavailability or declara tions. The
court may then deny the motion, order a continuance to permit affida vits to be
obtained or depositions to be taken, or make such other order as is just.
5) When affidavits or declarations are made in badfaith, the court may:
a) Order the party using them to reimburse the other party for those expenses
that the affidavits caused him, including attorneys' fees.
b) Adjudge in contempt the offending party or attorney.
g. Nonappealability
The denial of a motion for summary judgment is generally not appealable.

h. Relationship to Motion to Dismiss


A motion pursuant to Rule 12(b)(6) to dismiss a complaint for failure to state a claim
upon which relief can be granted differs from a motion for summary judgment in that
the former is addressed only to the legal sufficiency ofthe complaint.

i. Relationship to Motion for Judgment on the Pleadings


A motion for judgment on the pleadings presents the moving party's contention that on
theface ofthe pleadings, he is entitled to judgment. Theoretically, matters outside the
pleadings are irrelevant to a decision on either of these motions. However, a party
making such a motion and accompanying it with an affidavit or other matters outside the
pleadings may in reality be making a motion for summary judgment, putting the wrong
label on the motion. The court is expressly authorized to treat such a motion as one for
summary judgment and to conduct subsequent proceedings thereon in accor
dance with the rule on summary judgment, giving the parties full opportunity to present
material made relevant by that rule.

7. Motion for Judgment as a Matter of Law (Formerly Directed Verdict)


Historically, a judge could direct a particular verdict whenever the evidence-viewed in the light
most favorable to the party against whom the verdict was directed (including legitimate
CIVIL PROCEDURE-FEDERAL 79.

inferences in that party's favor) and without considering the credibility of witnesses-was such
that reasonable persons could come to only one conclusion. Today, this can be done pursuant
to a party's motionforjudgment as a matter of law ("JMO:v'). The motion may be made by any
party any time before submission of the case to the jury, and the moving party must specify the
judgment sought and the law and facts on which the party is entitled to judgment. The motion
may be granted only after the nonmoving party "has beenfully heard" on the matter. To grant
the motion, the court must find that "a reasonablejury would not have a legally sufficient
basis to find for the [nonmoving] party on that issue." [Fed. R. Civ. P. 50(a)]

8. Renewed Motion for Judgment as a Matter of Law (Formerly Judgment


Notwithstanding the Verdict ("JNOV"))
Historically, a party against whom judgment was entered could move for JNOV if the judgment
was based upon a verdict that reasonable persons could not have reached and if the moving party
had sought a directed verdict at the close of all the evidence. Now, the motion for JNOV is
called a renewed motionforjudgment as a matter oflaw. It must be filed no later than 28 days
after entry of judgment and the party making the renewed motion must have moved for
judgment as a matter of law at some time during the trial. In theory, a party may raise only
those issues raised in the motion for a JMOL. The standard is the same as for the motion for
judgment as a matter of law. [Fed. R. Civ. P. 50(b)]

9. Motion for New Trial


A motion for a new trial must be filed no later than 28 days after judgment is entered. Within that
period, the court may order a new trial on its own motion. [Fed. R. Civ. P. 59]

a. Reasons for Granting New Trial


The court may grant a new trial because of an error during the trial (usually going to the
admissibility of evidence or the propriety of the instructions), because the verdict is
against the weight of the evidence (limited to cases where the judge finds the verdict
seriously erroneous), because ofjuror misconduct, or because the verdict is excessive or
inadequate.

1) Remittitur
If the trial judge believes that the jury's compensatory damages award is so
exces sive as to "shock the conscience" (or in a diversity case if the award
meets the state standard for excessiveness), the judge may order a new trial or
may offer the alternative of remittitur. When offered remittitur, the plaintiff is
given the choice between accepting an award less than that given her by the jury
or submitting to
a new trial. Note that the court cannot simply lower the award given by the jury. It
must offer the plaintiff the alternative of a lower award or a new trial. [Hetzel v.
Prince Williams County, 523 U.S. 208 (1998)]
2) Additur
If the trial judge believes that the jury's compensatory damages are inadequate, she
may not offer the defendant the choice of accepting a higher award or submitting
to a new trial. ''Additur" has been held to violate the Seventh Amendment (which
is not applicable to the states). However, inadequate damages may be a basis for a
new trial.
80. CIVIL PROCEDURE-FEDERAL

Renewed Motion for Judgment as a Matter of Law with Motion for New Trial
When a renewed motion for judgment as a matter of law and a motion for a new trial are
made in the alternative and the renewed motion is granted, the court must rule
hypothetically on the new trial motion so that no remand is required if the ruling on the
judgment as a matter of law is subsequently reversed on appeal.

10. Effect of Failure to Move for a Renewed Judgment as a Matter of Law or for a New
Trial
If a party fails to move for either a renewed judgment as a matter of law or for a new trial on
the basis of insufficiency of the evidence, that party is precluded from raising the question of
evidentiary sufficiency on appeal, to support either judgment as a matter of law or a new trial.
[Unitherm Food Systems, Inc. v. Swift-Eckrich, Inc., 546 U.S. 394 (2006)]
11. Judgment on Partial Findings
In a nonjury trial, the judge may enter ajudgment as a matter of law against a party on any issue
whenever there are sufficientfacts to resolve the issue, provided that the party has been fully
heard on the issue. If the issue is dispositive of a claim or defense, the judge may enter
judgment as a matter of law against a party on that claim or defense. The judge may also wait
until the close of all evidence to render judgment. Because the judge is acting as the trier of
fact, she decides issues of disputed facts, and she may consider the credibility of witnesses.
The judgment must be supported by findings of fact and conclusions of law. [Fed.
R. Civ. P. 52]

VIII. ATTACK ON THE JUDGMENT AT THE TRIAL COURT LEVEL


A. RELIEF FROM JUDGMENT OR ORDER
1. Clerical Mistakes
A clerical error is one arising from oversight or omission, and may occur in judgments, orders,
or other parts of the record. The court can correct clerical errors on its own motion or the
motion ofany party. [Fed. R. Civ. P. 60(a)] There is no time limit for the correction of clerical
errors, and the court order correcting the error dates back to the time judgment was entered. As a
result, the battle over what constitutes a clerical error is acute.

2. Motions to Amend Prior Orders or Renew Prior Motions


Motions to amend a final order (i.e., one that disposes of the litigation) may be brought under
Federal Rule 59(e) when: (i) new evidence has been discovered that was not previously avail
able; (ii) there has been an intervening change in controlling law; (iii) there is a need to correct a
clear error of law or fact; or (iv) there is a need to prevent manifest injustice. Such
a motion must be brought within 28 days of the order; otherwise, the party must seek relief
from judgment under Rule 60 or appeal. It should be noted that an order that does not dispose of
all parties and claims is not a final order and may be modified under Federal Rule 54(b).

3. Other Grounds for Relief from Judgment


On motion and just terms, the court may relieve a party from a final judgment or order on the
following grounds:
CIVIL PROCEDURE-FEDERAL 81.

(i) Mistake, inadvertence, surprise, or excusable neglect;


(ii) Newly discovered evidence that by due diligence could not have been discovered in
time to move for a new trial;
(iii) Fraud, misrepresentation, or other misconduct of an adverse party;
(iv) The judgment is void;
(v) The judgment has been satisfied, released, or discharged; apriorjudgment on which it is
based has been reversed or otherwise vacated; or it is no longer equitable that the
judgment should have prospective application; or
(vi) Any other reason justifying relief from the operation of the judgment.
For grounds (i), (ii), and (iii), the motion must be made within a reasonable time not to exceed
one year; for the other grounds, the motion must be made within a reasonable time. [Fed. R.
Civ. P. 60(b)] Note: Ground (iv) does not apply simply because the judgment was erroneous;
such errors are to be remedied on appeal. Ground (iv) applies only if there was a fundamental
flaw such as lack ofjurisdiction or deprivation ofdue process by failure to give notice or
opportunity to be heard. [United Student Aid Funds, Inc. v. Espinoza, 559 U.S. 260 (2010)]

B. INDEPENDENT ACTION IN EQillTY TO SET ASIDE THE JUDGMENT


A court, in its discretion, may entertain an independent action to relieve a party from a judgment or
order. The plaintiff must show that he is likely to win if a new action is allowed. The only advantage
of an independent action is that it will not be barred by the specific time limits outlined in A., above.
However, the aggrieved party must act promptly once he knows or should know of the ground for
relief. An independent action will be rejected if a motion to set aside the judgment has been rejected
on the merits.

IX. FINAL JUDGMENT AND APPELLATE REVIEW


A. JUDGMENT
1. Relief that May Be Given
Except in default cases, the court is not limited to the demand for relief in the pleadings and
may give any reliefthat is appropriate based on the evidence. Thus, damages may exceed the
plaintiff's demand and an injunction may be entered although not requested. Interest on a money
judgment is awarded at the rate provided under state law from the date ofjudgment.

2. Judgment on Multiple Claims or Parties


When multiple claims or multiple parties are involved in an action, the court may enter a final
judgment as to fewer than all ofthe claims or parties only on an express determina tion that
there is nojust reason for delay and an entry ofjudgment. Unless the trial judge makes such
an express determination, the order determining the merits of fewer than all of the claims or
dismissing fewer than all of the parties is not afinaljudgment and is not
82. CIVIL PROCEDURE-FEDERAL

appealable. This is in accord with the traditional policy against piecemeal appeals. [Fed. R. Civ. P. 54 (b)]
3. Final Decision on Merits May Be Valid Despite Lack of Subject Matter Jurisdiction Occasionally, lack
of subject matterjurisdiction is not raised until the decision is final and all appeals are completed. The question then is
whether the decision may be collaterally attacked-i.e., be set aside in an independent proceeding or treated as invalid in a
later case. The factors that must be balanced in making this determination are: (i) lack ofjurisdiction is clear; (ii)
jurisdiction depends on a question of law, not fact; (iii) the court is of limited, not general, jurisdiction; (iv) the question
ofjurisdiction was not litigated; and (v) strong policy exists against the court acting beyond its jurisdiction.
B. TIME FOR APPEALS
Under Rules 3 and 4 of the Federal Rules of Appellate Procedure, an appeal may be taken by filing a notice of appeal
with the district court within 30 days from the entry of the judgment appealed from (60 days where the United States
is a party to the action). However, if a timely (within 28 days) renewed motion for judgment as a matter of law, a
motion for new trial, or a motion to set aside or amend the judgment is made, the running ofthe 30 days is terminated.
Upon the entry of an order based on such post-trial motions, a new 30 day period begins to run. A notice of appeal filed
during the pendency of such a post-trial motion will become effective on final disposition of the motion by the trial court.

1. Extension of Time for Appeal


Upon a showing of excusable neglect, made within 30 days after the time to appeal has expired, the district court may
extend the time for filing a notice of appeal by 30 days from the time it would otherwise have run, or 14 days from
the date of the order granting the extension, whichever is later. The court also may reopen the time to appeal for a
period of 14 days when (i) the party did not timely receive notice of entry ofjudgment; (ii) the motion
for extension is filed within 180 days of the judgment or within 14 days of receiving notice of judgment, whichever is
earlier; and (iii) the opposing party will not be prejudiced.
C. REVIEWABLE ORDER
Generally, onlyfinal orders are reviewable on appeal. A final order is one that disposes of the whole case on its merits,
by rendering final judgment not only as to all the parties but as to
all causes of action involved. [Cunningham v. Hamilton County, 527 U.S. 198 (1999)-order imposing sanctions on attorney
is not a final order even when the attorney no longer represents a party to the case] However, certain interlocutory orders
are also reviewable:
1. Interlocutory Orders as of Right
a. Injunction
A party may appeal as of right any order granting, continuing, modifying, refusing, dissolving, or refusing to
dissolve or modify an injunction.
b. Receivers
A party may appeal as of right any order appointing a receiver, or refusing to wind up or take steps to
accomplish purposes of receiverships (e.g., directing sales or other disposals of property).
CIVIL PROCEDURE-FEDERAL 83.

c. Admiralty
An order finding liability but leaving damages to be assessed later in admiralty cases may
be appealed.

d. Patent Infringement
A patent infringement order where only an accounting is wanting may be appealed.

e. Property Possession
A party may appeal as of right any order whereby possession of property is changed or
affected, such as orders dissolving writs of attachment and the like.

2. Interlocutory Appeals Act


Review under the Interlocutory Appeals Act [28 U.S.C. § 1292] is discretionary and may be
available when: (i) the trial judge certifies that the interlocutory order involves a control ling
question of law, as to which there is substantial ground for difference of opinion, and
immediate appeal from the order may materially advance the ultimate termination of the
litigation; and (ii) the court of appeals then agrees to allow the appeal. A party obtaining such a
certificate from the trial judge must, within 10 days, apply to the court of appeals, where two
out of three judges must agree to hear the appeal.

3. Fewer than All Claims or Parties


(See A.2., supra.)

4. Collateral Order Rule


If the claim or issue is separable from and collateral to the main suit and is too impor- tant to
require deferring appellate review, it may be classified as a judgment in a separate
("collateral") proceeding and thus be appealable. [Puerto Rico Aqueduct & Sewer Authority
v. Metcalf & Eddy, Inc., 506 U.S. 139 (1993) -govemmental entity's claim of Eleventh
Amendment immunity from suit denied; issue appealable immediately under collateral order rule
because failure to permit interlocutory appeal would effectively eviscerate Eleventh Amendment
immunity from suit in federal court by requiring entity to litigate to final judgment before
appealing]

5. Review of Nonappealable Orders by Writ


In exceptional cases, nearly all jurisdictions allow some circumvention of the final judgment rule
through the appellate writs of mandamus and prohibition. Mandamus commands a trial judge to
act, and prohibition commands the judge to refrain from acting. The writs are avail able only if
an appeal will be insufficient to correct a problem and the trial court's actions constitute a
serious abuse of power that must be immediately corrected.

6. Certification of Class Actions


A district court's order granting or denying certification of a class action can be appealed within
14 days of entry of the order. [Fed. R. Civ. P. 23 (f)] The court of appeals has complete
discretion in deciding whether to hear the appeal. If the court decides to hear the appeal,
proceedings are not stayed at the district court unless the district court or court of appeals so
orders.

84. CIVIL PROCEDURE-FEDERAL

D. STANDARDS OF REVIEW
1. On Matters of Law
On appeal, when it is alleged that the trial judge erred on a pure matter of law, the appellate court
may substitute its judgment for that of the trial judge. This is called a de novo review.
2. On Questions of Fact
In a bench trial, the trial judge will make findings of fact (see VII.K.11., supra). The trial
judge's findings of fact will not be disturbed on appeal unless they were "clearly erroneous."
[Fed. R. Civ. P. 52(a)(6)] A factual determination by the jury is afforded even greater
weight on appeal. Findings of fact by a jury will be affirmed on appeal if, while viewing the
evidence in the light most favorable to affirming the jury's verdict, a reasonable jury could have
reached the same conclusion. (This is similar to the standard a trial judge faces when deciding
whether to grant a judgment as a matter of law; see VII.K.7., supra.)
3. On Mixed Questions of Law and Fact
Mixed questions of law or fact are reviewed de novo. It is often difficult to determine whether the
question is purely factual, purely legal, or mixed. Generally speaking, whether a set offacts
meets a legal definition (e.g., whether the use of copyrighted material is "fair use" under
copyright law) is considered to be a mixed question of law and fact.
4. On Discretionary Matters
Many decisions a trial judge makes are left to her discretion (e.g., whether to consolidate or
sever cases, whether to grant leave to amend a pleading, etc.). On appeal, the standard of
review is whether the judge "abused her discretion" in making her decision. This means that the
judge's ruling will not be overturned on appeal unless is it plainly wrong or without an
appropriate basis.
E. STAY PENDING APPEAL
Stays are governed generally by Rule 62.
1. Execution
No execution on judgments is allowed for 14 days after entry except injunctions or receiv erships,
which are not held up unless otherwise ordered by a court. Thereafter, judgments become
enforceable duringpendency ofpost-trial motions unless a court otherwise orders in its
discretion and on such conditions for the security of the adverse party it deems proper.
2. Bond
A supersedeas bond is required in sufficient size to satisfy the judgment, costs, interest, and
damages for delay, should the appeal be dismissed or affirmed. Upon filing such a bond, an
appellant has a stay pending appeal-unless the order was for an injunction or receivership.
3. Injunction Order
Power of Trial Court
When appeal is taken from an interlocutory or final judgment granting, dissolving, or
denying an injunction, the court may suspend, modify, restore, or grant an injunction during
the pendency of the appeal upon such bond as it considers proper for the security of the
adverse party.

CIVIL PROCEDURE-FEDERAL 85.

b. Power of Appellate Court


An appellate court has similar power to grant a stay or injunction pending appeal, or to
vacate one granted by the trial court, or to make any order appropriate to preserve the status
quo or the effectiveness of the judgment subsequently to be entered. Ordinarily such a stay
or injunction pending appeal must be sought in the trial court before the appellate court
will entertain it.

F. SUPREME COURT JURISDICTION


The Supreme Court has direct appeal jurisdiction from any order granting or denying an injunc tion
in any proceeding required to be heard by a three-judge court. [28 U.S.C. § 1253]
1. Court of Appeals Cases
Cases in the courts of appeals may be reviewed by the Supreme Court:
(i) By certiorari granted upon petition of any party to any civil or criminal case, before or after
rendition ofjudgment or decree; or

(ii) By certification by the court of appeals of any question of law in any civil or criminal case
as to which it desires instructions. Upon such certification, the Supreme Court may give
binding instructions or may require the entire record to be sent to it for decision of the entire
case.
[28 u.s.c. §1254]
2. Cases from Highest State Court
Final judgments rendered by the highest court of a state in which a decision could be had may
be reviewed by the Supreme Court by certiorari in the following circumstances:
(i) Where the validity of a treaty orfederal statute is drawn into question; or
(ii) Where the validity of a state statute is drawn into question on the ground that it is
repugnant to the federal Constitution or to a treaty or federal statute; or
(iii) Where any title, right, privilege, or immunity is claimed under the federal Constitution or
treaty or federal statute.

[28 U.S.C. §1257] Only the Supreme Court may hear appeals coming from the state court
system.

X. EFFECTS OF JUDGMENT ON FUTURE CASES


A. CLAIM PRECLUSION (RES JUDICATA)
1. Definition
Once a.final judgment on the merits has been rendered on a particular cause of action, the
claimant is barred by claim preclusion (res judicata) from asserting the same cause of action in a
later lawsuit.

86. CIVIL PROCEDURE-FEDERAL

2. Terminology Used to Describe Effect-"Merger" and "Bar"


When the claimant wins the earlier lawsuit, the cause of action is said to have been "merged''
into the judgment. When the defendant wins, the claimant is said to be "barred" by the earlier
adverse judgment. Both terms simply mean that the claimant cannot sue again on the same
cause.
3. Requirements for "Merger" and "Bar"
Before merger or bar apply, it must be shown that (i) the earlier judgment is a valid, final
judgment "on the merits"; (ii) the cases are brought by the same claimant against the same
defendant; and (iii) the same "cause ofaction" (or "claim") is involved in the later lawsuit.
4. Valid, Final Judgment
Claim preclusion (res judicata) flows from the entry in an earlier case of a valid, final judgment "on
the merits." A judgment is valid as long as it is not void (e.g., for lack of subject matter
jurisdiction). Whether a judgment is final for these purposes is generally the same as whether it is
final for purposes of taking an appeal. (See IX.C., supra.)
5. "On the Merits"
Usually, the more difficult issue is whether the valid, final judgment is considered "on the
merits" for purposes of claim preclusion. Often, a judgment will be based on actual litigation
between the parties, but it can also be a default judgment entered as a penalty against a party
(such as a dismissal for willful violation of discovery orders) or an involuntary dismissal closely
related to the merits (such as for failure to state a claim upon which relief may be granted). In
contrast, other involuntary dismissals not involving the merits (such as those based on lack
ofjurisdiction, improper venue, or failure to join an indispensable party) are not a judgment on
the merits and do not have claim preclusive effect. Although Federal Rule 41 (b) indicates that
all dismissals are to operate "as an adjudication on the merits" unless based on jurisdiction,
improper venue, or failure to join an indispensable party, the Supreme Court has held that Rule
4l(b) does not govern whether the judgment is "on the merits"
for purposes of claim preclusion. [Semtek, Inc. v. Lockheed Martin Corp., 531 U.S. 497
(2001)] Thus, jurisdictions may take different views of whether a particular dismissal-e.g.,
dismissal because the statute oflimitations has run-is deemed "on the merits" for purposes of
claim preclusion.
6. Same Claimant Versus Same Defendant
Traditionally, claim preclusion applies only if the earlier case and the latter case are brought by
the same claimant against the same defendant. It is not enough that the same litigants were also
parties in the previous case; they must have been in the same configuration of one asserting a
claim against the other.
Examples: 1) In Case One, A sues Z to recover damages for personal injuries suffered
in an automobile collision between the two. A valid, final judgment on the
merits is entered. Now A sues Z again, this time to recover damages for
property damage inflicted in the same wreck. Assuming that both cases
involve the same "cause of action" (discussed immediately below), claim
preclusion would apply, because both cases were brought by A against Z.
2) In Case One, A sues Z to recover damages for personal injuries suffered in
an automobile collision between the two. A valid, final judgment on the
CIVIL PROCEDURE-FEDERAL 87.

merits is entered. Now Z sues A to recover for her personal injuries suffered in
the same wreck. Claim preclusion does not apply. Here, the second case is
brought by Z against A, while thefirst case was brought by A against Z.
Note: Z may be barred from asserting her claim because of the compulsory counterclaim rule,
but not because of claim preclusion. The compulsory counterclaim rule requires a defending
party to assert against the claimant in thepending case any claims arising from the same
transaction or occurrence as the claimant's claim. (See VII.F.3.c., supra.)

7. "Cause of Action"
While various tests have been used to define "cause of action," the modem approach is to
require assertion of all claims arising out of the same transaction or occurrence that is the
subject matter of a claim asserted by the claimant.
a. Common Examples
1) Accidents
The claimant seeks to recover separate damages from the same accident in
separate actions. The claimant may not seek damages for neck injuries in one
action and leg injuries in another. Likewise, most courts would not permit the
claimant to sue for personal injuries and property damage in separate actions.
However, ifthe claimant is insured for property damage and, after payment of the
claim, the claimant assigns her cause of action for property damage to the
insurance company, most courts would consider the property damage claim and
personal injury claim as two separate causes of action.

2) Installment Obligations
In the situation of a series of obligations, such as installment payments on a debt or
lease, the claimant is required to sue on all installments due at the time of suit, but
not later installments. But if the contract has an acceleration clause that makes all
installments due if earlier ones are not paid, the claimant must sue for all
installments (unless the acceleration clause is optional and the claimant elects not to
exercise the option). This rule does not apply if the installment obligations are
represented by separate notes; in such cases, suit as each note comes due repre sents
suit on a separate cause of action.
B. ISSUE PRECLUSION (COLLATERAL ESTOPPEL)
1. Definition
A judgment binds the plaintiff or defendant (or their privies) in subsequent actions on different
causes of action between them (or their privies) as to issues actually litigated and essential
to thejudgment in the first action. This conclusive effect of the first judgment is called issue
preclusion (or collateral estoppel). Note that issue preclusion is narrower than claim preclusion.
Claim preclusion focuses on something relatively large-the scope of a "cause ofaction." If it
applies, the result is usually to bar the claimant from asserting a second case. Issue preclusion,
in contrast, focuses on something relatively narrow-an issue that was litigated and deter mined in
the first case, and that is relevant in a second case. With issue preclusion, the issue is deemed
established in the second case without need to proffer evidence on it.
88. CIVIL PROCEDURE-FEDERAL

2. Requirements
a. Final Judgment
The traditional view is that the final judgment requirement for issue preclusion (collat eral estoppel) is very similar
to the final judgment requirement for claim preclusion. (See A.4.-5., supra.) However, recent decisions have
relaxed the final judgment rule for issue preclusion. A judgment may be "final" even if it is subject to post-trial
motions or appeals. Several factors may be weighed in determining whether to give preclu sive effect to a
judgment not entered as final, such as whether the prior decision was adequately deliberated, whether the court's
decision is supported by reasoned opinion, and whether the prior decision was subject to appeal or is on
appeal.
b. Issue Actually Litigated and Determined
The issue on which issue preclusion applies must actually have been litigated and deter mined in the previous case.
Thus, if a default or consent judgment is entered, there is generally no issue preclusion as to the fact issues
that would have been tried had the case gone forward.
c. Issue Was Essential to the Judgment
1) It must be clear exactly how the issue was decided by the trier of fact.
Example: P sues for personal injuries based on D's negligence. D pleads contributory
negligence as a defense. If the jury renders a general verdict for D, the decision will
have no issue preclusive effect in a subsequent case involving either P or D's negligence,
because there is no way of knowing whether the jury found that D was not negli
gent or that P was contributorily negligent, or both.
Compare: However, if the jury found for P for the full amount of his injuries, it clearly had
to decide that D was negligent and P was not. Thus, both issues could have issue
preclusive effect in a later case.
2) The judgment must depend on the issue of fact decided.
Example: If, in a personal injury action, the jury specially finds that neither P nor D was
negligent-thereby rendering a verdict for D-the finding that P was not negligent
was not essential to judgment and will have no issue preclusive effect in a later
suit.
3) Note that the "essential fact" rule tends to reduce the number of cases in which issue preclusion can be
applied, thus eliminating some of the burden from the first suit.
d. Due Process and Mutuality Considerations
1) Against Whom Is Issue Preclusion Used?
Issue preclusion may be asserted only against someone who was a party (or in privity with a party) to
the previous case (the case in which the issue was actually litigated and determined). This requirement is
imposed by due process, and thus is the rule in every jurisdiction.

2) CIVIL PROCEDURE-FEDERAL 89.

3) By Whom Is Issue Preclusion Used?


Under the traditional "mutuality" rule, only someone who was a party (or in privity
with a party) in the previous case can use issue preclusion. This require ment is not
imposed by due process, however, and has been subject to modification in certain
circumstances to allow nonparties to take advantage of a prior judgment.

Exceptions to Mutuality When Judgment Used Defensively


When a nonparty wishes to utilize a prior judgment to avoid liability in a subse
quent suit, there are often compelling reasons for allowing her to do so. Fairness to
4) the nonparty will also be considered.
Example: If P unsuccessfully sues a person primarily liable (e.g., an employee),
P's later suit person primarily liable should be subjected to a separate suit, and most
against a courts so hold.
person
secondarily Exceptions to Mutuality When Judgment Used Offensively-Consider Fairness
liable (e.g., Courts have been very reluctant to permit a nonparty to use issue preclusion to aid
against the him (as a plaintiff) to obtain relief.
employer for Example: Suppose one of many passengers in a public vehicle successfully sues
the the driver for injuries received in an accident, and other passen gers
employee's wish to utilize the judgment to establish liability. While a few courts
acts) will be have permitted such use, others refuse. They fear a situation in which
barred by 10 plaintiffs each sue and lose, and the 11th plaintiff wins, and all
issue other potential plaintiffs seek to ignore the first 10 suits and rely solely
preclusion in on the 11th; application of issue preclusion in such a situa tion is
virtually all considered unfair and demeaning to the legal system.
courts.
Similarly, if P However, the United States Supreme Court, in Parklane Hosiery Co. v. Shore, 439
unsuccessfull U.S. 322 (1979), upheld the use of issue preclusion offensively. In the first action,
y sues a brought by the Securities and Exchange Commission, the defendant was held to have
person violated the federal securities laws. The second suit, brought by a private plaintiff
secondarily against the same defendant, alleged damages resulting from the same violation
established in the first action. The Court allowed the latter plaintiff to rely on issue
liable, there is preclusion to establish the existence of the violation since under all the circumstances
little reason it wasfair to the defendant to do so. Thus, in cases of nonmu tual issue preclusion
why the used offensively, the key often is whether such use isfair and equitable.
C. CLAIM AND ISSUE PRECLUSION IN SPECIAL SITUATIONS

1. Judgments for Specific Performance


Rules of bar and issue preclusion apply in actions brought for specific performance and the
like. However, merger does not apply because such a judgment, unlike one for money,
cannot be enforced by bringing a suit on the judgment. Thus, if the defendant fails to obey
the first judgment, the claimant may sue again.
90. CIVIL PROCEDURE-FEDERAL

2. In Rem Judgments
If a court exercises in rem jurisdiction over some property or status within its control, and if
proper notice has been given to all interested persons, the judgment as to title or status is
binding on allpersons.
3. Quasi In Rem Judgments
A quasi in rem judgment determines the rights of the parties only in the specific property before
the court. No personaljudgment is granted against anyone, and no otherproperty is affected.
D. WHICH PERSONS ARE BOUND BY A JUDGMENT?
1. Parties Are Bound
Parties are persons named as parties who have the power to control the action or who, if they
lack capacity, are represented by guardians. Nonparties normally are not bound. Even where the
lawsuit raises an issue as to performance or rights, nonparties normally are not bound by the
judgment; e.g., an assignor who has no control over the suit and no interest in the outcome, or an
employee who allegedly was negligent, where the suit is filed only against the employer.
2. Privies to Parties Are Bound by Issue Preclusion
Persons who control the litigation and who will be affected by the outcome are bound by
issue preclusion as to all issues litigated. For example, if the owner of a patent assumes
control of an infringement suit brought by her licensee against a competitor, and the court
holds the patent invalid, the owner is barred on that issue should she sue the same compet
itor. The owner has had her day in court.
3. Represented Parties May Be Bound by Claim Preclusion
Persons whose interests are represented and privities also may be bound by claim preclu sion.
For example, beneficiaries are bound by an action brought or defended on their behalf by the
fiduciary, provided the fiduciary is operating within her authority. Holders offuture interests are
bound. Unborn or unascertained persons having future interests in property are bound by
judgments as to the property if their interests are identical to those of parties to the action, or if a
special representative is appointed for them. This rule reflects public policy favoring free
marketability of property. Members ofa class are bound by a valid class action judgment.
Successors in interest are bound. Transferees ofproperty are in privity with prior owners and
thus are bound by a prior judgment concerning the property. This rule protects the public as to
security of titles.
Note, however, that one is not barred from asserting a claim simply because she is asserting
the same claim that a previous claimant has already litigated.
Example: Citizen A sues to challenge a tax as unconstitutional and loses. Citizen B is
not barred from suing to challenge the same tax on the same basis unless
Citizen A and Citizen B are in privity or Citizen A represented Citizen B in
bringing the first suit. [Richards v. Jefferson County, 517 U.S. 793 (1996)]
In vicarious liability situations (employer-employee, principal-agent, insurer-insured) a judgment
exonerating either generally is held to preclude an action on the same claim against the other.

Example: CIVIL PROCEDURE-FEDERAL 91.

P sues Principal claiming injuries as a result of Agent's negligence. P alleges


Agent was acting in the course and scope of agency at the time of harm. A
judgment in favor of Principal on the ground that Agent was not negligent would
in some jurisdictions preclude suing Agent thereafter on the theory that Agent was
negligent in causing the harm. If P first sued Agent and Agent was found not
negligent, P should thereafter be barred from suing Principal for
the negligence of Agent causing the harm.
E. WIDCH CHOICE OF LAW RULES APPLY TO PRECLUSION QUESTIONS? Preclusion
questions-whether claim preclusion or issue preclusion-always involve at least two cases. One case
has gone to a valid, final judgment on the merits. Preclusion law determines
whether thatjudgment (in "case one") precludes litigation of any matters in a pending case ("case
two"). (By the way, note that "case one" was not necessarily filed first; it is "case one" because
it went to judgment first.) A choice of law issue arises when case two is brought in a different
jurisdiction than case one; i.e., what law does the court in case two use to determine whether the
judgment in case one is entitled to claim or issue preclusion?

1. Case One Decided in State Court


When case one has been decided in state court, the court in case two (whether state or federal)
generally will apply the claim or issue preclusion law of the jurisdiction that decided case one.
Example: Judgment is entered in a case in Kansas. A second case is brought in
Missouri. To decide whether that case is subject to claim or issue preclusion,
the judge in Missouri should generally apply Kansas law on claim or issue
preclusion.

2. Case One Decided in Federal Court Under Diversity Jurisdiction


What if case one was decided in a federal court under diversity jurisdiction? Here, the Supreme
Court held that the court in case two should apply federal law (because a federal court decided
case one). However, it also held that usually the federal law in such an instance would be the
state law of the state in which the federal court sat.
Example: After plaintiff files suit in a California state court, defendant removes the case to
federal court based on diversity jurisdiction. Plaintiff's federal court case
is then involuntarily dismissed because it is barred by California's statute of
limitations. Plaintiff files the same claims in state court in Maryland (which has a
longer statute of limitations). In determining whether to dismiss the case under
claim preclusion, the Maryland state court should look to federal law. But,
despite the language of Federal Rule 4l(G), federal law would adopt the
California law (unless "state law is incompatible with federal interests").
Because California law would allow the plaintiff to file in a jurisdiction with a
longer statute of limitations, the Maryland court should not dismiss under claim
preclusion. [Semtek, Inc. v. Lockheed Martin Corp., A.5., supra]
CIVIL PROCEDURE CHARTS
1.

REMOVAL ISSUES

The key points to remember are:


� A federal court must have jurisdiction over the case; jurisdiction need not have been
proper in the state court.

� Removal is to the federal district court whose territory encompasses the state court.

� Only defendants can remove; all defendants generally must join in the removal.
l!f A case based on diversity may not be removed if any defendant is a citizen of the forum
state.
i!f• Notice of removal must be filed within 30 days of the date defendant receives a copy of the
initial pleading.

� If a case later becomes removable (as by dismissal of a nondiverse defendant), the case may be
removed within 30 days of the date it becomes removable, but (for diversity cases) not
more than one year affer it was brought in state court.
l!f The one year rule may be disregarded if the plaintiff has acted inbad faith to defeat
removal.
2. CIVIL PROCEDURE CHARTS ® barbrr

� CHART
TIMING OF PRE-ANSWER MOTIONS

Motion
Timing

1. Lack of jurisdiction over the subject-------------------------11 May be raised any time, even on
matter appeal

2. Lack of jurisdiction over the person


3. Improper venue Waived if not raised by motion or
answer, whichever is first
4. Insufficiency of process

5. Insufficiency of service ofprocess

May be raised any time before


trial or at trial
6. Failure to state a claim upon which---------------------------J
relief can be granted
7. Failure to join a party under Rule 19
(indispensable party)
CIVIL PROCEDURE CHARTS 3.

FEDERAL CLASS ACTION REQUIREMENTS


CHART

A federal class action must meet all four requirements on the left side of the chart and
one of the requirements on the right. The three alternatives on the right determine the
type of federal class action. Only the third type, i.e., the common question type, requires
notice to all class members and allows opting out.

Numerous class
and
Risk of inconsistent results
Common questions
or
and
Injunctive or declaratory relief appropriate
Typicality
or
and
Common questions predominate and a
Fair and adequate class action is superior to alternate
representation methods ofadjudication
4. CIVIL PROCEDURE CHARTS

PROCEDURAL DEVICES THAT MAY TERMINATE CASE

Metho Circumstances Timin


d g
Pre-Answer Motion Addresses the following (See summary chart supra
[Rule 12(b)] preliminary matters: defects in for timing of Rule 12
subject matter jurisdiction, defenses.)
personal jurisdiction, venue,
process, and service of
process; failure to state claim;
failure to join needed party.
Voluntary Dismissal by Without prejudice once as a If dismissed as a matter of
Plaintiff [Rule 41 (a)] matter of right; also possible right without prejudice, must
by stipulation or court order. be done before defendant
files answer or motion for
summary judgment.
Involuntary Dismissal Plaintiff fails to prosecute Any time.
[Rule 41 (b)] the case or to comply with
the Rules or a court order.
Motion for Judgment on On the face of the pleadings After pleadings are closed
the Pleadings [Rule 12(c)] (without considering matters but not so late as to
outside the pleadings), the delay trial.
moving party is entitled to
judgment. Treated as motion
for summary judgment if
accompanied by outside
matters.
Summary Judgment No genuine dispute of Unless local rule or court
[Rule 56] material fact and moving order dictates otherwise, a
party is entitled to judgment as party may file a motion for
a matter of law. May support summary judgment at any
by pleadings, affidavits, time until 30 days after close
discovery materials. of discovery. If a motion is
premature, the court may
defer ruling on it.
Judgment on Partial In a nonjury trial, the judge During trial, once the judge
Findings [Rule 52] may enter a judgment as a has heard sufficient evidence
matter of law if she makes to make dispositive findings
dispositive partial findings on and all parties have been
the claim. fully heard on the issue.
Motion for Judgment as Evidence viewed in light After opponent has
a
Matter of Law (Directed most favorable to motion's presented case but before
Verdict) [Rule 50(a)] opponent leads reasonable submission of case to jury.
person to conclusion in favor
of moving party.
Renewed Motion for The verdict returned could not Within 28 days after entry of
Judgment as a Matter have been reached by judgment.
of Law ("JNOV") [Rule reasonable persons. Moving
50(b)] party must have previously
sought judgment as a matter of
law sometime during the trial.
CIVIL PROCEDURE MULTIPLE CHOICE QUESTIONS 1.

CIVIL PROCEDURE MULTIPLE CHOICE QUESTIONS INTRODUCTORY

NOTE
You can use the sample multiple choice questions below to review the law and practice your under
standing of important concepts that you will likely see on your law school exam. To do more questions, access
StudySmart Law School software from the BARBRI website.

Question 1 Question 2
The plaintiff and defendant in a lawsuit were in A writer registered under federal copyright law
an accident in which the plaintiff was injured. As his copyright in certain song lyrics he wrote. The
a result of the accident, the plaintiff incurred writer later entered into a contract with an
medical expenses of $ 100,000. At the time of the advertiser in which the writer granted the adver
accident, the plaintiff and defendant both lived in tiser a license to use the lyrics in radio advertise
State A. Before the action was filed, the plain tiff ments. When the writer heard the advertisement
moved permanently to State B. The plaintiff then using the lyrics, the writer was incensed at how
filed a negligence action against the defen dant in the lyrics had been used. Believing that the
federal district court, with subject matter advertiser had lied to him about how the lyrics
jurisdiction being based on diversity of citizen would be used, the writer filed an action in federal
ship. After the action was filed but before the district court claiming that the advertiser had made
defendant was served with process, the defen dant false representations that fraudulently induced the
was transferred by his employer and moved writer into entering the contract to license the
permanently to State B. lyrics. The writer is a citizen of State
A. The advertiser is a partnership comprised of
For purposes of evaluating the court's diver sity partners who are citizens of State A, State B, and
of citizenship jurisdiction, what are the State C. The partnership's headquarters and most of its
citizenships of the two parties? operations are in State B.

(A) Both are citizens of State A. Does the federal court have subject matter
jurisdiction over the action?
(B) The plaintiff is a citizen of State B and the
defendant is a citizen of State A. (A) No, because the action does not arise under
federal law and the parties are citizens of the
(C) The plaintiff is a citizen of State A and the same state.
defendant is a citizen of State B.
(B) Yes, because the plaintiff and defendant are
(D) Both are citizens of State B. citizens of different states.

(C) Yes, because the action arises under federal law.

(D) Yes, because the transaction involves inter state


commerce.
2. CIVIL PROCEDURE MULTIPLE CHOICE QUESTIONS

Question 3 Question 4
A homebuyer filed a breach of contract action While driving in State A, the defendant, a State
in a State A state court against the contractor (a B resident, was in an automobile accident with the
corporation) that agreed to build the home and sell plaintiff, a resident of State A The plaintiff filed a
it to the buyer. The homebuyer's complaint joined negligence action against the defendant in a State
as an additional defendant the contractor's agent A state court consistent with State Ns long arm
who negotiated the contract and signed it on behalf statute, and properly served the State B defendant
of the contractor. The action sought pursuant to the State A long arm statute. The State
$150,000 in damages from both defendants. The B defendant immedi ately filed a motion to
contractor is a citizen of State B. The homebuyer dismiss the action on the grounds that the State A
and the contractor's agent are citizens of State A court does not have personal jurisdiction.
Six months after filing the action, the homebuyer
dismissed the claims against the agent, leaving
only the claims against the contractor. The How should the court rule on the motion to
corporation immediately and appropriately filed a dismiss?
notice of removal.
(A) The court should deny the motion, because
May the contractor remove the action to federal the defendant has purposeful contacts with
district court? State A that are directly related to the claim
being asserted.
(A) Yes, because the action can be properly
removed at any time after the action was (B) The court should deny the motion because
filed. the plaintiff is a State A resident.

(B) Yes, because, while the action was not (C) The court should grant the motion unless the
properly removable when it was filed, it defendant is subject to general jurisdic tion
now can be properly removed to federal in State A
district court.
(D) The court should grant the motion because
(C) No, because, while the requirements for State A courts lack constitutional authority
diversity of citizenship jurisdiction now are to assert jurisdiction over defendants outside
satisfied, the time in which removal is of State A unless such defendants consent to
allowed has passed. the State A courts' jurisdiction.

(D) No, because removal is allowed only if


federal subject matter jurisdiction exists at
the time the complaint is filed.
iJ barbrr CIVIL PROCEDURE MULTIPLE CHOICE QUESTIONS 3.

Question 5 Question 6
The plaintiff, who resides in the Southern A pedestrian filed an action against a driver in
District of State A, was involved in a three-car federal district court, alleging negligence. The
accident in the Northern District of State A. The attorney for the driver has interviewed an
plaintiff intends to file a negligence action against eyewitness whose testimony will clearly indicate
the other two drivers in federal district court. One that the driver was at fault.
defendant resides in the District of State B and
the other resides in the District of State C.
Must the driver disclose the existence and
In which federal district(s) is venue proper? identity of the eyewitness to the pedestrian?

(A) The Northern District of State A only. (A) No, because the identity of the eyewitness is
protected from discovery under the work
(B) The District of State B and the District of product doctrine.
State C.
(B) No, because the driver is not likely to use
(C) The Northern District of State A, the District the eyewitness as part of her case.
of State B, and the District of State C.
(C) Yes, because the driver must disclose all
(D) The Northern District of State A, the witnesses who have discoverable informa
Southern District of State A, the District of tion, even without a specific request.
State B, and the District of State C.
(D) Yes, but only in response to an appropriate
interrogatory.
4. CIVIL PROCEDURE MULTIPLE CHOICE QUESTIONS

Question 7 that of the individual. The individual's lawyer later


served on the corporation a request for production
The plaintiff in a negligence action in federal
of documents. When the corpora tion objected to
district court was injured when the defendant's car
some of the requests, the lawyer filed a motion to
crashed into his. The defendant's friend was a
compel production. Shortly before the hearing on
passenger in the defendant's vehicle at the time of
the motion to compel, the individual advised the
the accident. The defendant's attorney sent
lawyer that the signature on the contract was in
an email to the passenger asking the passenger to
fact hers, but she and
describe in detail what the passenger remem
bered about the accident and the events leading the lawyer agreed that she nonetheless should
not be liable on the contract for other reasons.
up to it. In response, the passenger sent the
The lawyer thus continued to assert the motion
defendant's attorney an email describing the
to compel production. At the hearing on the
events. The plaintiff served on the defendant the
motion, the lawyer referred to the complaint and
following request for documents: "Please
the answer to justify the relevancy of the
produce for inspection or copying any and all
requests for production, but he did not mention
statements obtained by you or your attorney that
the signature in any way.
relate in any way to the events and/or issues that
are the subject of this legal action."
Has the lawyer violated Rule 11 of the Federal
Rules of Civil Procedure?
Must the defendant produce the passenger's
statement?
(A) No, because when the lawyer signed the
complaint, he had evidentiary support in
(A) Yes, because the statement is relevant to the
parties' claims and defenses. the expected testimony of the individual
who denied signing the contract.
(B) Yes, because the work product doctrine does
not apply to emails and other digitally stored (B) No, because the lawyer failed to state in the
motion hearing that the signature on the
information.
contract was not that of the individual.
(C) No, because the statement is protected from
discovery pursuant to the work product (C) Yes, because when the lawyer referred to
doctrine. the answer in the motion hearing, he
renewed his certification that facts in the
(D) No, because the statement is privileged. answer had evidentiary support at the time
of the hearing.
Question 8
(D) Yes, because the lawyer should not have
A corporation filed a breach of contract action signed the answer without having the
against an individual in federal district court. individual sign a sworn statement that the
Attached to the complaint was a written contract signature on the contract was not hers.
providing that the individual agreed to pay the
corporation $100,000 over a period of years for
specified services. The contract also contained the
individual's signature. The individual told his
lawyer that the signature was a forgery and that
she had never signed or entered into the contract.
On the basis of the individual's statement, the
lawyer drafted, signed, and filed an answer. The
answer denied the claim on a number of grounds
and denied that the signature on the contract was
iJ barbrr CIVIL PROCEDURE MULTIPLE CHOICE QUESTIONS 5.

Question 9 Question 10
While driving a new car he recently A State A citizen and a State B citizen were
purchased from an authorized dealer, a involved in a car accident. The State B citizen
consum er's car caught fire for an unknown filed a negligence action against the
reason. The fire not only damaged the car, but State A citizen in federal district court, seeking
also injured the consumer. The consumer filed
$500,000 for injuries incurred in the accident.
a products liability action against the
The State A citizen believes that she was not at
manufacturer of the car in federal district court,
fault and that the accident was caused by the
seeking to recover compensatory damages for
negligence of the State B citizen.
his injuries. The complaint alleged that parts of
the electrical system in the car were defective
and that the defects caused the fire. The May the State A citizen assert in the pending
manufacturer filed an answer that denied the action a negligence claim against the State B
existence of any defects and denied that any citizen, seeking $400,000 for the injuries the State
defects caused the fire, but stated that it lacked A citizen suffered in the accident?
sufficient knowl edge and information to know
what caused the fire. During discovery, the (A) No, joinder is improper under these cir
consumer served an interrogatory on the cumstances.
manufacturer that asked the manufacturer to
"identify and summarize all evidence of which (B) Yes, and the court has discretion to grant or
the manufacturer [was] aware that indicated that deny the motion in the interest ofjustice.
the fire was not caused by a defect in the car."
The manufacturer's response stated that it did (C) Yes, as a counterclaim in the pending
not have, and was not aware action, or she may assert it as an indepen
of, any evidence indicating that the fire was not dent action.
caused by a defect. Based on that interrogatory
response, the consumer filed a motion for partial (D) Yes, but only as a counterclaim in the
summary judgment on the issue of causation to pending action and will be barred from
establish that any fire was caused by defects. asserting it as an independent action.
How should the court rule on the motion?

(A) Grant the consumer's motion for summary


judgment.

(B) Grant the consumer's motion for summary


judgment unless the manufacturer files
affidavits or other evidence indicating that
the fire was not caused by a defect.

(C) The court should deny the consumer's


motion, because it addresses an ultimate
issue in the case.

(D) The court should deny the consumer's


motion, because, unless the consumer has
presented evidence that a defect caused the
fire, the manufacturer does not need to
present evidence regarding causation.
6. CIVIL PROCEDURE MULTIPLE CHOICE QUESTIONS

Question 11
Which ofthe following statements is
A plaintiff sued a defendant in federal district correct regarding the defendant's right to
court for breach of contract. Based on a prelimi amend his answer to add an affirmative
nary investigation, the defendant's attorney defense?
believes that no legally enforceable contract ever
existed. Unfortunately for the defendant, (A) The defendant has a right to amend
however, his attorney is unsure whether she can his answer any time before trial.
prove that point at trial. If a valid contract did
exist, the defendant's attorney believes that the (B) The defendant must obtain leave of the
defendant did not breach it. She also thinks that court to amend his answer, but the
she has a better chance of prevailing on that court should freely grant leave.
point.
(C) The defendant may amend his answer to
Which of the following is true regarding the add the affirmative defense only if he can
defendant's ability to assert as defenses both show he could not, with due diligence, have
that no contract existed and that the defendant discovered the defense prior to serving his
did not breach the contract if one did exist? answer.

(A) The defendant may plead only one of (D) The defendant may not amend his
these defenses because they are answer more than 21 days after serving
inconsistent. it.

(B) The defendant may plead both defenses,


but the plaintiff will then be able to have
one defense stricken.

(C) The defendant may plead both


defenses, as long as he labels them as
affirmative defenses.

(D) The defendant may plead both


defenses regardless of how they are
labeled.

Question 12

A plaintiff filed a breach of contract action


against a defendant in federal district court,
invoking the court's diversity of citizenship juris
diction. The defendant filed an answer denying
the material allegations in the complaint.
Approximately two months after the answer was
served, the court entered a scheduling order that
required the parties to complete all discovery
within 10 months after the entry of the sched
uling order. Two months later (four months
after service of the answer and two months into
the discovery period), the defendant sought to
amend his answer to add an affirmative
defense that the plaintiff's claim was barred by
the statute of limitations.
ANSWERS TO MULTIPLE CHOICE QUESTIONS 7.

ANSWERS TO MULTIPLE CHOICE QUESTIONS


Answer to Question 1
(B) The plaintiff is a citizen of State B and the defendant is a citizen of State A. In addition to an amount
in controversy that exceeds $75,000, diversity of citizenship jurisdiction requires complete diversity,
meaning that each plaintiff must be a citizen of a different state from every defendant. Whether
complete diversity exists is determined when the suit is filed, not when the cause of action arose or
when the defendant is served with process. The citizenship of a natural person
is the state in which he is domiciled. A new state citizenship may be established by (i) physical
presence in a new place; and (ii) the intention to remain there permanently. In this question, the
plaintiff was originally from State A, but then moved permanently to State B before suit was filed.
After suit was filed, the defendant also moved to State B from State A. Because the plaintiff's move
to State B was before he filed suit, he is considered to be a citizen of State B for purposes of
diversity jurisdiction, whereas the defendant is considered to be a citizen of State A because his move
did not occur until after suit was filed. Thus, complete diversity exists. Choices (A), (C), and
(D) are incorrect for the reasons stated above.
Answer to Question 2
(A) The federal court does not have jurisdiction because neither diversity of citizenship jurisdiction nor
federal question jurisdiction exists. Diversity of citizenship jurisdiction is available when (i) there is
complete diversity of citizenship, meaning that each plaintiff is a citizen of a different state from every
defendant; and (ii) the amount in controversy exceeds $75,000. A natural person's citizenship is the
state that is the person's domicile. A partnership is a citizen of each state of which its partners, both
limited and general, are citizens. Here, the writer is a citizen of State A, and the advertiser's partners
are citizens of State A, State B, and State C. Given the State A-State A connection, complete diversity
does not exist. Hence, (B) is an incorrect answer choice as to diversity jurisdiction. Federal question
jurisdiction is available when the plaintiff, in his well pleaded complaint, alleges a claim that arises
under federal law. Anticipation of a federal defense or the fact that federal law is implicated by the
plaintiff's claim do not give rise to federal question
jurisdiction; the plaintiff's claim must arise under federal law. Here, although federal copyright law is
peripherally involved, the writer's cause of action is actually based on state contract law. As a result,
no federal question has been presented by the writer's complaint, making (C) incorrect.
Note too that federal question jurisdiction does not have a complete diversity requirement, making
(B) incorrect as to federal question jurisdiction. (D) is incorrect because federal question jurisdic tion
does not arise merely because interstate commerce is affected.
Answer to Question 3
(B) The contractor may remove the action. A defendant may remove an action that could have origi nally
been brought in the federal courts (because subject matter jurisdiction based on either a federal
question or on diversity of citizenship was present). Diversity of citizenship jurisdiction is available
when (i) there is complete diversity of citizenship, meaning that each plaintiff is a citizen of a different
state from every defendant; and (ii) the amount in controversy exceeds $75,000. Generally speaking, a
defendant has 30 days from the date he receives the initial summons or complaint to remove a case.
However, if a case becomes removable on the basis of diversity at
a later date, he has 30 days to remove the case from the date the defendant is served with the
document that first makes the case removable. That said, for cases based on diversity, removal may
not take place more than one year after the case was filed. In the instant case, the homebuyer
8. ANSWERS TO MULTIPLE CHOICE QUESTIONS ® barbrr

(from State A) initially sued the agent (also from State A) and the contractor (from State B) for
$150,000. Thus, the case was not initially removable. The case then became removable when the
homebuyer dismissed the agent from the case, leaving the State B corporation as the sole defen dant.
At this point, the corporation has 30 days to remove the case, and the one-year restriction does not
come into play because the facts state that only six months have passed since the case was filed. As
a result, (B) is the correct answer. (A) is incorrect because the case was not initially removable. The
claim was for a state law breach of contract, so no federal question was presented, and complete
diversity was initially lacking, so diversity of citizenship jurisdiction was not avail able. (C) is
incorrect because the 30-day period has been met (since the corporation immediately filed a notice of
removal), and only six months have passed since the case was filed in state court, making the case
removable on the basis of diversity of citizenship jurisdiction. (D) is an incor rect statement of the law.
The grounds for removal need not exist at the time the case is filed. If grounds for removal come up
later, the case may still be removed, subject to certain restrictions.

Answer to Question 4
(A) The court should deny the motion. The question provides that service was made pursuant to the State
A long arm statute. The assertion of personal jurisdiction is statutorily authorized, in that it is
consistent with State J>:s long arm statute, and the exercise of personal jurisdiction is constitu tional,
given that the claim arises from the defendant's purposeful activities (using roads) in State A Thus, a
court in State A may properly exercise personal jurisdiction over the defendant from State B, even if
the defendant does not consent to personal jurisdiction and is not domiciled there.
(B) is incorrect because personal jurisdiction cannot be exercised against a defendant based on the
plaintiff's domicile. (C) is incorrect. There are two types of personal jurisdiction: general and specific.
Here, the defendant's activity in the state allows State A to exercise specific jurisdiction Uurisdiction
for the specific claim arising from the activity) . General jurisdiction Uurisdiction
for all causes of action) is not needed. (D) is incorrect because consent is not necessary to assert
personal jurisdiction over a defendant, as explained above. Consent is one basis for exercising personal
jurisdiction over a defendant, but not the only basis.

Answer to Question 5
(A) Venue is proper only in the Northern District of State A Federal venue is proper in (i) the district in
which any defendant resides if all defendants reside in the same state; and (ii) the district in which a
substantial part of the events or omissions giving rise to the claim occurred. Here, the accident
occurred in the Northern District of State A, making that district a proper venue under prong (ii).
However, given that the defendants here reside in different states, venue cannot be based on the
residence of the defendants. Thus, choices (B), (C), and (D) are incorrect. Choice
(D) is also incorrect because venue is not based on the residence of the plaintiff.

Answer to Question 6

(D) The driver must disclose the identity ofthe witness in response to an appropriate interrogatory.
Federal Rule 26(a) requires, as an initial disclosure, a party to reveal the name and contact infor
mation of individuals who are likely to have discoverable information and who the disclosing party
may use to support his claims or defenses (unless the use would be solely for impeach ment) . After
initial disclosures are made, discovery proceeds, and the parties may continue with discovery of
nonprivileged information that is relevant to any party's claim or defense, including the names and
contact information of any person who knows of any discoverable matter. Here, the eyewitness would
not need to be disclosed as an initial disclosure because the driver obviously
ANSWERS TO MULTIPLE CHOICE QUESTIONS 9.

will not use the eyewitness to support the driver's claim or defense. However, the identity of the
eyewitness would need to be disclosed eventually, assuming the pedestrian submits a proper discovery
request. This makes (D) correct and (B) incorrect. (A) is incorrect because the work product doctrine
does not prevent the disclosure of the existence of the eyewitness. Any materials generated by the
attorney would probably be protected under the work product doctrine (unless
a showing of substantial need and undue hardship can be made); however, the eyewitness's name and
contact information would not be protected. (C) is an overbroad description of the initial disclosure
requirements and is, thus, incorrect.
Answer to Question 7
(C) The defendant does not need to produce the passenger's statement. Work product prepared in
anticipation of litigation is discoverable only on a showing of substantial need and undue hardship in
obtaining the substantial equivalent of the work product. Here, there is no indication that the
passenger is unavailable or cannot recall the accident. Thus, it is unlikely that the plaintiff will
be able to show substantial need and undue hardship in obtaining a statement from the passenger as to
her recollection of the accident. As a result, the statement is protected from discovery.
(Note that this analysis is applicable to the statement only; the existence of the passenger as a
witness must be disclosed either as an initial disclosure-assuming the defendant is going to use
passenger to support her claim or defense-or in response to a properly submitted interrogatory.)
(A) is an overbroad statement of the requirements for discovery. Federal Rule 26(b)(3) specifically
exempts documents prepared in the anticipation of litigation from discovery, and the passenger's
statement falls into this category. (B) is an incorrect statement of the law; the fact that the state ment
was electronic does not prevent it from becoming protected under the work product doctrine.
(D) is incorrect because the statement is not "privileged" per se (such as the doctor-patient evidentiary
privilege), but rather is exempt from discovery under the work product doctrine under Federal Rule
26(b)(3). As described above, there are exceptions to the work product doctrine.
Answer to Question 8
(C) The lawyer has violated Rule 11. By "presenting" a document to the court, Rule 11 provides that a
lawyer certifies that he believes the denials of factual contentions in the document are warranted on
the evidence and that his belief is formed after a reasonable inquiry. One "presents" a document not
only by signing or filing it, but also by "later advocating" it. Thus, Rule 11 imposes a continuing
certification requirement, applicable any time a matter is presented to the court. At the time he signed
the complaint, the lawyer believed there was evidence that the individual did not sign the contract based
on the individual's own statement. When the lawyer referred to the answer in the motion hearing, the
lawyer "presented" the answer to the court anew and renewed his certification, despite the fact that he
never discussed the signature. At that time, the lawyer
no longer believed that all the facts in the answer had evidentiary support. He knew that he lacked
evidence to support the answer's denial that the signature on the contract was that of the individual.
Thus, (C) is correct and (B) is incorrect. (A) is incorrect because it does not take into
consideration the "later advocated" basis for presenting a document under Rule 11. (D) is incorrect
because an attorney need not have his client sign a sworn statement for every fact that the client tells
the attorney.
Answer to Question 9
(D) The court should deny the consumer's motion. The plaintiff must prove the elements of the prima
facie case for her claim. Absent proof of an element of the prima facie case, summary judgment
10. ANSWERS TO MULTIPLE CHOICE QUESTIONS ® barbrr

for the plaintiff is not appropriate. Thus, (A) is incorrect and (D) is correct. Furthermore, although a
party is generally required to respond to a motion for summary judgment with affidavits, the facts here
indicate that the consumer has not come forward with any evidence pertaining to the manufacturer's
fault for the damage and injury. Thus, because the consumer has not properly supported his motion for
summary judgment with relevant, admissible information, there is no need for manufacturer to produce
any evidence to survive a motion for summary judgment. This makes (B) incorrect. (C) is an incorrect
statement of the law. A motion for summary judgment may be granted even though the motion
addresses an ultimate issue in the case.
Answer to Question 10
(D) The State A citizen must assert her claim. Because the State A citizen's claim arises from the same
transaction or occurrence as the claim asserted against her in the pending action, the State A citizen's
claim is a compulsory counterclaim which must be asserted in the pending action, or it is lost. As a
result, (D) is correct and (A) is incorrect. (B) is incorrect in that it is not a discretionary call with the
judge whether the claim may be filed as a counterclaim. (C) is incorrect because the claim may not be
asserted as an independent action. FRCP 13 states that the pleader need not state a compulsory
counterclaim if the subject matter of the compulsory counterclaim was already pending in another
court. However, if the pleader has not filed suit on the subject matter of the compulsory counterclaim
(which she has evidently not done here), she must file the compulsory counterclaim.
Answer to Question 11
(D) The defendant may plead both defenses regardless of how they are labeled. The Federal Rules of
Civil Procedure expressly permit inconsistent pleadings in the alternative or hypothetically,
reasoning that the discovery process will operate to sort out the viable pleadings. For this reason,
(A) and (B) are incorrect. (C) is incorrect because the Rules do not limit inconsistent claims or
defenses to "affirmative" defenses.
Answer to Question 12
(B) The defendant may amend with leave of the court. A party may amend a responsive pleading
of right within 21 days after serving it. Thereafter, according to Federal Rule 15, the party
may amend only with consent of all parties or with leave of the court, but the "court should
freely grant leave when justice so requires." With so much time left for discovery and before
trial, a
court would almost certainly grant leave to amend. (A) is incorrect because, as stated, the time for
amendment as of right is 21 days after serving it, not any time before trial as (A) implies. (C) is
incorrect in that due diligence in discovering the defense need not be shown. (D) is incorrect. The 21-
day period applies to amendment as of right, but a court may grant leave to amend after that period.
APPROACH TO CIVIL PROCEDURE 1.

APPROACH TO EXAMS
CIVIL PROCEDURE
IN A NUTSHELL: To determine where to file a case, a plaintiff must find a court that has power over
the defendants ("personal jurisdiction") and power over the type of case ("subject matterjurisdic tion"), and
the location (''venue") must be proper. The plaintiff drafts a complaint (sometimes called
a "petition") that will inform the defendant of the plaintiff's claims; the plaintiff files the complaint with
the court. Once filed, the plaintiff must provide the defendant with timely notice the complaint has been
filed ("service of process"), using a method authorized by law. After the defendant is served with
process, the defendant may challenge the merits of the case or defects in the complaint or petition by filing
an answer and/or various motions (e.g., a motion to dismiss). The defendant may also file
a claim against the plaintiff. Thereafter, the parties disclose to each other the evidence each may have
("discovery"). If a party discovers that his opposing party may not be able to prove a claim or defense
at trial, he may ask the court to dismiss the case for lack of evidence ("motion for summary
judgment"). If the plaintiff's case survives to this point, it is tried before a jury (if requested) or a
judge. In certain circumstances, the case may be withdrawn from the jury, or thejury's verdict may be set
aside, or the case may be appealed.

I. JURISDICTION OVER THE PERSON


State law must authorize jurisdiction, and the exercise of such jurisdiction must be constitutional

A. Types of Personal Jurisdiction


1. In personam-forum has personal jurisdiction over defendant
2. In rem-forum has power to adjudicate rights of all persons to a particular item of property;
defendant not personally bound
3. Quasi in rem-two types
a. Quasi in rem type I-court adjudicates rights of parties in property based on
property being in forum; close connection between case and property provides
minimum contacts
b. Quasi in rem type II-court attaches property to bring defendant into forum on
unrelated claim; defendant must have minimum contacts with forum

B. Statutory Limitations on Personal Jurisdiction


1. Federal court must analyze personal jurisdiction as if it were a court of the state in which it is
located
a. State law must authorize jurisdiction
b. Most, if not all, states authorize jurisdiction over a defendant who:
Is present in forum state and personally served with process therein;
2) Is domiciled in forum state;
3) Conducts systematic and continuous business in the state such that the defendant is
"essentially at home" therein;
4) Consents to jurisdiction; or
5) Commits an act covered by the long arm statute

Constitutional Limitations on Personal Jurisdiction


1. Traditional rule-physical power
APPROACH TO CIVIL PROCEDURE

2. Modern due process standard-contact and fairness


a. Defendant must have such minimum contacts with the forum such that the exercise of
personal jurisdiction over him is fair and reasonable
1) Consider whether defendant purposefully availed himself of the benefits and
protections of state law and whether he could have anticipated being brought into state
court
b. Notice also required
1) Traditional methods of personal service satisfy notice requirements
2) Requirement that agent notify defendant
3) Requirement for cases involving multiple or unknown parties

II. SUBJECT MATTER JURISDICTION IN FEDERAL COURTS


A. Diversity of Citizenship Jurisdiction
1. Complete diversity
a. Every defendant must be of diverse state citizenship from each plaintiff-this is
"complete diversity"
b. Must have complete diversity when action commenced
1) Interpleader exception
c. Alienage jurisdiction-citizen of U.S. state and foreign citizen
d. Questions of citizenship
1) Individuals-domicile
2) Corporations-every state/country where incorporated and one state/country of
principal place of business, which is the place from which the corporation's high
level officers direct and control its activities
3) Unincorporated associations and limited liability companies-citizenships of its
members
4) Legal representatives-domicile of the represented person
Class actions-domiciles of the named members
6) Nonresident U.S. citizens-not a citizen of any state and not an alien
e. Realignment of parties according to their true interest
1) Shareholder derivative actions
f. Supplemental jurisdiction
1) Must be one claim with original jurisdiction
2) Supplemental claim must arise from a common nucleus of operative fact as the
original jurisdiction claim such that the claims should be tried together
g. Subsequent addition of parties and claims
1) Intervention of right-no supplemental jurisdiction
2) Substitution of parties-citizenship of original party controls
3) Impleader-supplemental jurisdiction as to claim of Defendant v. Third-Party
Defendant; claim of Plaintiff v. Third-Party Defendant must have original jurisdic tion in
diversity case
4) Cross-claims-supplemental jurisdiction
2. Jurisdictional amount in excess of $75,000
a. Amount in controversy does not include interest and costs or counterclaims
1) Attorneys' fees and interest that are recoverable by statute or as part of claim are
included
b. Aggregation
APPROACH TO CIVIL PROCEDURE 3.

1) One plaintiff may aggregate claims against a single defendant


2) One plaintiff may not aggregate separate claims against several defendants One
plaintiff may sue several defendants on a joint liability claim if it exceeds
$75,000
Several plaintiffs may aggregate claims against one defendant if seeking to enforce
single title or right
c. Supplemental jurisdiction over claims by permissively joined plaintiffs not exceeding
$75,000 possible in diversity cases
d. Counterclaims
1) Compulsory counterclaim may invoke supplemental jurisdiction
2) Permissive counterclaim needs original jurisdiction
Citizenship generally not an issue-if diversity exists between Plaintiff v.
Defendant, it exists between Defendant v. Plaintiff
3. Erie doctrine and law applied in diversity cases-state substantive law, federal procedural law
4. Exceptions to diversity of citizenship doctrine-no probate, divorce, alimony, or child
custody cases
5. Multiparty, Multiforum Trial Jurisdiction Act
B. Federal Question Jurisdiction
1. Federal question must appear in complaint
Implied federal right of action possible
3. Supplemental jurisdiction
a. Pendent claims-federal court has discretion to hear state law claim if it arises from a
common nucleus of operative fact as the federal claim such that the claims should be
tried together
b. Pendent parties-must be common nucleus of operative fact
4. Specific statutory grants of exclusive federal jurisdiction

III. VENUE
A. Subject Matter Jurisdiction Distinguished-Venue Is Proper Geographic District
B. General Rules
1. Venue proper in federal district where any defendant resides (if all defendants reside in the
same state), where a substantial part of events or omissions occurred, or where a substantial
part of property situated
Fallback provisions-if no district satisfies above, venue proper in federal district where any
defendant is subject to court's personal jurisdiction
3. Unlike subject matter jurisdiction, venue can be waived
C. Residence for Venue Purposes
1. Individuals-domicile
Business entities-where subject to court's personal jurisdiction
3. Nonresidents-any judicial district
D. Transfer
1. Original venue proper-transfer for convenience to venue where case might have been
brought or to venue to which parties consent
4. APPROACH TO CIVIL PROCEDURE

2. Original venue improper-transfer to venue where case could have been brought to correct error
a. Dismissal if transfer not available or if some extraordinary circumstance exists

IV. REMOVAL JURISDICTION


Defendant can remove an action that could have been brought originally in federal court
A. Requirements
1. Original jurisdiction
a. Diversity jurisdiction
1) Dismissal of nondiverse party allows removal
2) Case cannot be removed based solely on diversity if any defendant is a citizen of the forum state
3) Case cannot be removed based solely on diversity more than one year after case was filed unless defendant
can show plaintiff acted in bad faith to prevent removal
b. Federal question jurisdiction
1) Having a federal defense is insufficient
2) Entire case is removed
3) State law claims may be severed and remanded to state court
2. State court need not have had jurisdiction
3. Only defendants may remove
4. Venue-federal district where state action was filed
B. Procedure for Removal
1. Time-30 days
a. Generally starts to run after formal service of complaint
b. If later pleading, motion, etc., is filed that shows case is now removable, period begins to run on service of
that pleading, motion, etc.
2. Remand to state court ifno federal subject matter jurisdiction

V. CONFLICT OF JURISDICTION BETWEEN STATE AND FEDERAL COURTS


A. Injunctions Against State Court Proceedings
1. Federal court generally may not enjoin pending state proceedings
2. Threatened state criminal proceedings-to prevent clear and imminent harm that cannot be remedied with a criminal
appeal

VI. FEDERAL RULES OF CIVIL PROCEDURE


A. Commencement of Action-Complaint and Service of Process
B. Service of Process
1. Personal service, abode service, service on agent; also state rules and waiver of service (by mail)
2. Parties served outside of state or in foreign country
3. Immunity from process for parties, witnesses, attorneys in another action, or fraud or deceit
APPROACH TO CIVIL PROCEDURE 5.

C. Interlocutory Injunctions-Maintain Status Quo Until Trial


1. Preliminary injunctions-require notice
2. Substantive requirements-irreparable harm, harm to party seeking injunction outweighs harm
to party to be bound, likelihood of success on the merits, public interest factors
3. Temporary restraining orders-necessary to prevent irreparable injury

D. Pleadings
1. Complaint-notice of plaintiff's claim
2. Pre-answer motions
a. Motion to dismiss for (i) lack of subject matter jurisdiction; (ii) lack of personal juris
diction; (iii) improper venue (transfer probably); (iv) insufficient process or service of
process; (v) failure to state a claim; and (vi) failure to join a party
b. Motion for more definite statement
c. Motion to strike
3. Answer-specific or general denials by defendant
a. Compulsory counterclaims-arises out of same transaction or occurrence; must be
pleaded
b. Permissive counterclaims-any other counterclaim
c. Inconsistent claims or defenses allowed
4. Special pleading for, e.g., fraud, mistake, special damages; must be more detail
5. Reply by plaintiff generally not required
6. Amendment of pleading and supplemental pleadings
a. Relates back to filing date of original complaint if it concerns same conduct, transac
tion, or occurrence
b. If party is charged, relates back if, within period for service of process, new party (i)
received notice of the action such that party will not be prejudiced; and (ii) knew or should
have known that, but for plaintiff's mistake concerning identity, new party would have been
made a party originally
7. Rule 11
a. Attorney certifies proper purpose upon presenting paper to court
b. Sanctions-judge has discretion limited by deterrence factor
E. Joinder
1. Compulsory joinder
a. Court cannot accord complete relief without absentee
b. Absentee has interest that will be impaired by lawsuit
c. Parties are at substantial risk for multiple or inconsistent judgments without absentee
d. If a., b., or c. is true, and absentee's presence will not destroy subject matter jurisdiction or
venue, and the court can obtain personal jurisdiction over absentee, he must be joined
e. If absentee can be joined, court must consider whether to proceed without absentee,
looking at:
1) Prejudice to absentee and parties Whetherjudgment
can be shaped to avoid prejudice Adequacy
ofjudgment without absentee
4) Whether another forum can hear entire case
2. Permissive joinder-arises out of same occurrence and transaction and common question of law
or fact
3. Joinder of claims

6. APPROACH TO CIVIL PROCEDURE

a. Class actions
1) Requirements
a) Numerous class so joinder of all is impracticable;
b) Common questions of law or fact;
c) Named parties' interests are typical;
d) Named parties will ensure fair and adequate representation of absent
members; and
e) Either (i) separate actions risk inconsistent results or harm absent members;
(ii) injunctive or declaratory relief is appropriate; or (iii) common questions of
law or fact predominate and class action is superior to other methods
2) Effect ofjudgment-all members bound unless opt out
3) Notice of pendency required in "common question" suit so members can opt out
4) Jurisdiction-for diversity, named parties control whether diversity and amount in
controversy are satisfied
5) Notice of settlement must be given to class members so they can object at "fairness
hearing"
6) Court must approve dismissal or settlement after "fairness hearing"; if class action
based on common question of law or fact, court may refuse settlement unless
members are given second chance to opt out
b. Class Action Fairness Act
1) Federal jurisdiction if:
a) Any member of the plaintiff class is of diverse state citizenship from any
defendant
b) Aggregated amount in controversy exceeds $5 million
c) 100 members to the class
2) Local considerations may defeat jurisdiction
c. Shareholder derivative suits
1) Must have been a shareholder at time of transaction (or received shares by opera tion
of law); not collusive effort to confer jurisdiction; made demand on directors if
required
2) Jurisdictional amount-consider corporation's damages
3) Venue-where corporation could have sued the same defendants
d. Interpleader
1) To avoid double liability
2) Mnemonic: Rule 22 interpleader must follow the regular rules; statutory inter
pleader has special, simple standards
4. Intervention
a. As of right-intervenor has an interest in property or transaction that is subject of the
action and action may adversely affect interest
b. Permissive-intervenor's action has common question of law or fact
c. No supplemental jurisdiction in federal court
5. Impleader-generally to bring in nonparty to get indemnity or contribution
a. Supplemental jurisdiction over claim by defendant/third-party plaintiff against third party
defendant, but no supplemental jurisdiction over claim by original plaintiff against third-
plaintiff defendant
6. Cross-claims-co-partners may sue each other for claims arising out of same transaction
or occurrence
a. Supplemental jurisdiction available
APPROACH TO CIVIL PROCEDURE 7.

VII. DUTY OF DISCLOSURE AND DISCOVERY


A. Disclosure Requirements
1. Types of disclosure required without discovery request
a. Initial disclosures-generally disclosure of witnesses who support claims or defenses
and materials that support claims or defenses
b. Disclosure of expert testimony-testifying experts and their reports must be disclosed
c. Pretrial disclosures-trial witnesses, depositions to be used at trial, exhibits, etc.
2. Scope of disclosure and discovery-generally any relevant nonprivileged matter that is
proportional to the needs of the case
a. Trial preparation materials-only if substantial need and to avoid undue hardship
b. Experts
1) Testifying experts-may depose
2) Consulting experts-only if exceptional circumstances
c. Protective orders may limit or end discovery if abused
3. Supplementation of disclosures and discovery response required if learn information given is
materially incomplete or incorrect
B. Types of Discovery upon Request
1. Pre-action depositions to perpetuate testimony
2. Oral depositions
a. Notice of deposition compels appearance of parties
b. Nonparties must be subpoenaed
3. Written depositions-questions to deponent submitted in wrtiting
Interrogatories to parties-written questions to parties
5. Production of physical material-nonparties need to be subpoenaed
6. Request for admissions as to truth or genuineness of any matter or document
C. Enforcing Disclosure and Discovery
1. Motion to compel disclosure
a. Must certify good faith attempt to resolve dispute with opponent
2. Failure to comply with court order
a. Sanctions
b. No sanctions if substantially justified or unjust
3. Failure to disclose or supplement
a. No use of material unless substantially justified or harmless
b. Sanctions
4. Failure to admit
a. Pay expenses for proving matter unless generally some good reason for not admitting
5. Failure to attend own deposition, serve answers to interrogatories, respond to request for
inspection
a. Must certify good faith attempt to resolve dispute with opponent
b. Sanctions
c. Must seek order of protection to avoid sanctions if request objectionable
D. Use of Depositions at Trial or Hearing
Subject to the rules of evidence, depositions can be used:
1. To impeach the witness
2. For dead, unavailable, or absent witness
3. For any purpose if deponent is the adverse party
Rule
8. APPROACH TO CIVIL PROCEDURE 16(b
)
sche
dulin
g
E. Pretrial Conferences confe
1. Rule 26(f) conference of parties-planning for discovery rence
3. Sanctions for failure to attend, obey an order, etc.

VIII. TRIAL
A. Alternative Dispute Resolution
1. Contractual arbitration-written agreement to arbitrate
Judicial arbitration-voluntary arbitration under auspices of court
3. Mediation-use of neutral person to facilitate voluntary settlement between the parties
B. Trial
1. Jury trial problems
a. Right to jury trial-7th Amendment
b. Jury size-at least six, no more than 12 jurors
c. Jury instructions-objections must be made before jury retires
d. Jury verdicts-general (for plaintiff or defendant and amount of damages) or specific
Uury makes findings on material issues of fact)
2. Involuntary dismissal-with prejudice
3. Voluntary dismissal-by plaintiff, with or without leave of court
4. Summary judgment-if no genuine dispute ofmaterial fact, party entitled to judgment as a
matter oflaw (no trial necessary)
5. Judgment as a matter of law (directed verdict)
a. Evidence viewed in light most favorable to nonmoving party
b. Witness credibility is not considered
c. Standard-evidence is such that a reasonable jury would not have a legally
sufficient basis to find for the party on that issue
6. Renewed motion for judgment as a matter of law (a.k.a. JNOV)
a. Same standards as above
7. Motion for new trial-some error occurred at trial (e.g., juror misconduct)
a. If made with renewed motion forjudgment as a matter of law, and renewed motion is
granted, judge must rule hypothetically on new trial motion
8. Party waives "sufficiency of the evidence" argument on appeal ifhe fails to move for a
renewed motion forjudgment as a matter of law or for a new trial

IX. POST-TRIAL MOTIONS AND APPEALS


A. Attack on the Judgment at the Trial Court Level
1. Relief from judgment or order is given for:
a. Clerical mistakes
b. Other grounds for relief from judgment
Independent action in equity to set aside the judgment
B. Final Judgment and Appellate Review
1. Judgment
a. Appropriate relief that may be given
b. Final decision on merits may sometimes be valid despite lack of subject matter jurisdic tion

APPROACH TO CIVIL PROCEDURE 9.

2. Time for appeals-generally 30 days


3. Reviewable order-final orders reviewable on appeal
a. Interlocutory orders as of right-reviewable
b. Interlocutory Appeals Act-review discretionary
c. Collateral order rule-reviewable
d. Orders may be made appealable (or nonappealable) by writ
e. Certification of class action-can be appealed within 14 days of order
4. Stay pending appeal may be granted
5. Supreme Court has jurisdiction to hear federal appellate cases (certiorari or certification) and state
supreme court cases (by certiorari) where federal issue

X. CLAIM AND ISSUE PRECLUSION


A. Effects of Judgment on Future Cases
1. Claim preclusion (res judicata)-a final judgment on the merits bars claimant from asserting same
claim in later action
2. Issue preclusion (collateral estoppel)-a judgment binds parties (or their privies) in subse quent
actions between them as to issues actually litigated and essential to judgment in first action

B. Who Is Bound by Judgment?


1. Parties and their privies are bound
2. Mutuality rules
a. Traditionally, if a nonparty was not bound by a judgment, he could not use issue preclu sion
against one who was bound by the judgment
b. Some states (and federal courts) have relaxed mutuality rule when issue preclusion is used
defensively
c. Some states (and federal courts) employ a four-part test to determine if issue preclusion can be
used offensively-if all of the following are answered affirmatively, issue preclu sion can be
used offensively
1) Are the issues identical?
Is there a final judgment on the merits?
3) Did the party against whom the judgment is to be used have a fair opportunity to be
heard?
4) Is it not unfair or inequitable to apply issue preclusion?
CIVIL PROCEDURE EXAM QUESTIONS 1.

ESSAY EXAM QUESTIONS


INTRODUCTORY NOTE
The essay questions that follow have been selected to provide you with an opportunity to experience
how the substantive law you have been reviewing may be tested in the hypothetical essay examination
question context. These sample essay questions are a valuable self-diagnostic tool designed to enable you to
enhance your issue-spotting ability and practice your exam writing skills.
It is suggested that you approach each question as though under actual examination conditions.
The time allowed for each question is 60 minutes. You should spend 15 to 20 minutes spotting
issues, underlining key facts and phrases, jotting notes in the margins, and outlining your answer.
Ifyou organize your thoughts well, 40 minutes will be more than adequate for writing them down.
Should you prefer to forgo the actual writing involved on these questions, be sure to give yourself no
more time for issue-spotting than you would on the actual examination.
The BARBRI technique for writing a well-organized essay answer is to (i) spot the issues in a
question and then (ii) analyze and discuss each issue using the "CIRAC" method:
C - State your conclusion first. (In other words, you must think through your answer before
you start writing.)
I - State the issue involved.
R - Give the rule(s) of law involved.
A - Apply the rule(s) of law to the facts.
C - Finally, restate your conclusion.
After completing (or outlining) your own analysis ofeach question, compare it with the BARBRI
model answer provided herein. A passing answer does not have to match the model one, but it should
cover most of the issues presented and the law discussed and should apply the law to thefacts of the
question. Use of the CIRAC method results in the best answer you can write.
® barbrr
CIVIL PROCEDURE EXAM
QUESTIONS

EXAM QUESTION NO. 1


Plaintiff filed a complaint in the United States District Court in State X, alleging in substance the
following:
Plaintiff resides in State X; the shopping area nearest to plaintiff's residence is in State Y.
Plaintiff read, in a national magazine, an advertisement for an electrical device called "Warnem"
which, when installed in a home in accordance with instructions, was designed to sound an alarm
bell and turn on the lights if an intruder entered the premises. "Warnem" was manufactured by
defendant, Warnem Corporation, incorporated and having its company headquarters in State W; it
sells the "Warnem" device f.o.b. its plant in State W to retailers throughout the United States.
Plaintiff purchased a "Warnem" from a retailer in State Y and installed it in his home in State X in
accordance with instruc tions. While the "Warnem" was installed and plaintiff was asleep in his
home, intruders entered the home and destroyed a rare antique vase worth $5,000. The device
failed to function. Plaintiff claims damages of $5,000 for the value of the vase and $100,000 for
mental anguish resulting from its loss.
The summons and complaint, issued by the federal district court in State X, were served on
Warnem by personal delivery to the president of the corporation at the corporate office in State W.
You have been consulted by Warnem Corporation. Your research into State X law establishes that
the Supreme Court of State X has recently held that there can be no recovery of damages for mental
anguish from unintentional conduct unless there has been an impact. What are the various motions
that you might make or pleadings that you might file prior to filing an answer in the case, and how
would you expect the court to rule on each? Discuss.
CIVIL PROCEDURE EXAM QUESTIONS 3.

EXAM QUESTION NO. 2


Company, a manufacturer incorporated and with its plant and headquarters in State X, asks Bookkeeper,
an accountant from State Y, to prepare a financial statement for publication with a nationwide issue of
Company's securities. Company mails its books to State Y, where Bookkeeper prepares a favorable report,
negligently failing to discover that Company has fraudulently concealed several million dollars of debts. Company
mails the report from State X to stockbrokers in every state. Investor, a resident and businessperson of State Z,
reads the report in State Z and in reliance thereon purchases in State Z a number of shares of Company stock. He
is later compelled by the economic instability of Company to sell the shares in State Z at a loss.
Investor sues both Company and Bookkeeper for damages in a State Z court, serving both defen dants
by registered mail, return receipt requested, in their home states in accordance with a State Z statute.
Both Bookkeeper and Company move to dismiss for lack ofjurisdiction over the person. How should the
court rule on the motions? Discuss.
® barbrr
4. CIVIL PROCEDURE EXAM QUESTIONS

EXAM QUESTION NO. 3


Ped is a resident of State A Driver is a resident of State D. Health is a corporation incorporated in
State H with its company headquarters in State A Ped was injured when struck by a motor vehicle being
operated by Driver in State A Ped was hospitalized in the Health hospital and his injuries were
aggravated, allegedly as a result of the hospital's negligence.
Ped sued Driver and Health in state court in State A, claiming damages in the sum of $100,000 and
alleging that he was uncertain as to which defendant was responsible for his damages.
(1) Upon Health's timely notice of removal, the case was removed to the United States
District Court in State A Thereafter Ped moved to have the case remanded to the state court in
State A That motion was granted.
(2) After remand, Health moved to dismiss on the grounds of misjoinder of parties defendant and
improper joinder of several causes of action. The motion was overruled.
(3) Both Health and Driver filed answers denying liability and damages. Ped then filed timely
requests for admissions, asking that each defendant admit liability, reserving for trial only the issue of
damages. Both defendants filed timely objections on the grounds that the requests called for legal
conclusions. The objections were sustained.
(4) Following ajury trial, a verdict was returned in favor ofPed and against both defendants in the
sum of $43,652.89. Ped moved for a new trial on the issue of damages or, in the alternative, on all
issues. He supported his motion by the affidavits of five jurors that stated that: (a) immediately after
entering the jury room, the jurors took a ballot on the issue of whether Ped should recover and the vote
was 12 to zero in favor of Ped and against both defendants; (b) then each juror wrote down his idea of the
amount of the recovery, the figures were totaled, divided by 12, and the result was $43,652.89; and
(c) all jurors then agreed that their verdict would be $43,652.89. Based upon defendants' objections, the
court refused to consider the affidavits and denied Ped's motion for a new trial.
Assume for purposes of this question that State Ns rules of civil procedure are the same as those in
federal court. Discuss the correctness of the court's rulings, setting forth the arguments that might
reasonably be made in support of and in opposition to each of the following:
(1) Health's motion to remand;
(2) Health's motion to dismiss;
(3) Ped's requests for admissions; and
(4) Ped's motion for a new trial.
CIVIL PROCEDURE EXAM QUESTIONS 5.

EXAM QUESTION NO. 4


Valeo is a corporation incorporated in State B with its principal place of business in State A. It
manufactures pressure valves for use on compressed air tanks. It purchases from Mity, a corporation, the
"collars" affixed to the pressure valves, which are used to attach the valve to the air tank. Mity is
incorporated and has its sole place of business in State A.
The Valeo pressure valve on a piece of machinery owned by Peter and Quincy, and used by them in State C,
exploded. Peter and Quincy were seriously injured. At all times Peter was a resident of State
C. At the time of the explosion, Quincy was a resident of State A, but after the accident he moved to
State B, where he now lives.
Peter and Quincy wish to assert claims against Valeo and Mity on the theory that the valve exploded
because of a defective "collar." Valeo has informed Peter and Quincy that it will claim as a defense that a
written notice recalling the valves had been sent to them, and to all other users of this model valve, and that
Peter and Quincy ignored the notice.
(1) If Peter, as sole plaintiff, institutes an action against Valeo, as sole defendant, in United States District
Court in State C, may Valeo object to the failure to join Mity as a defendant? May Valeo bring Mity into
the case as a defendant and, if so, how? Discuss.
(2) If Peter and Quincy institute an action against Valeo and Mity in United States District Court in
State C for $100,000 damages each, what issues might be raised as to joinder of parties plaintiff, joinder of
parties defendant, joinder of causes of action, jurisdiction over the defendants, and jurisdic tion over the
subject matter? How should the trial court rule on each issue? Discuss.
(3) If an action by Peter, as sole plaintiff, against Valeo and Mity is commenced and tried in United
States District Court in State A, should the trial court apply State A law, State C law, or federal law on
the two issues of (a) whether plaintiff must plead freedom from contributory fault, and (b) who has the
burden of persuasion on the issue of contributory fault? Discuss.
CIVIL PROCEDURE EXAM ANSWERS 1.

ANSWERS TO ESSAY EXAM QUESTIONS


ANSWER TO EXAM QUESTION NO. 1
Motion to Dismiss for Lack of Subject Matter Jurisdiction
I could make a motion to dismiss for lack of subject matter jurisdiction, which the court should
grant. The district court's power to hear and determine this case turns on whether it hasjurisdic-
tion over the subject matter, otherwise the suit must be dismissed. Since the facts present no "federal
question," jurisdiction must proceed based on diversity of citizenship, which requires that all plaintiffs be of
diverse citizenship from all defendants, and that there be more than $75,000 in controversy.
Clearly, the diversity element is met (plaintiff residing in State X and Warnem a resident of State W);
however, it is debatable whether the amount in controversy exceeds $75,000. The jurisdictional amount is
ordinarily determined from the prayer of the complaint, subject to a good faith limitation that there be
some reasonable expectation of recovery in excess of $75,000. If it appears to a legal certainty that
recovery cannot go above $75,000, the action must be dismissed.
Although plaintiff's complaint alleges $105,000 in damages, $100,000 ofthis is for mental anguish, and the
recent decision of the State X Supreme Court indicates that those damages are not recoverable when there is no
impact. Since a federal court sitting in a diversity case must apply the same substan tive law as would the
highest court of the state in which it is sitting [Erie v. Tompkins] , the federal court in this case is bound by
the State X decision. This would require disregarding the $100,000 claim for mental anguish, with the result
that there will be a lack of subject matterjurisdiction and the case must be dismissed.

Motion to Dismiss for Lack of Personal Jurisdiction


I could also make a motion to dismiss for lack of personal jurisdiction. The district court's power over
Warnem turns on whether it has in personamjurisdiction over it. Although it is not a certainty that the
motion would be granted (as above), there is a good chance that I would prevail on the motion. Since plaintiff is
seeking to recover money damages from Warnem ("in personam" action), there must be a showing of some
constitutionally sufficient contact between Wamem and State X to subject the company to the court's personal
jurisdiction. Assuming State X statutes empower its courts to exercise jurisdiction up to the constitutional
limits, the question is whether those limits would be exceeded in Warnem's case.
Constitutional limits are couched in terms of fair play and substantial justice and require a finding of some
purposeful forum-related activity by the defendant. This case also presents a "stream of commerce"
scenario. When a manufacturer merely places its product in the stream of commerce, and does not do
anything further, it will not be subject to personal jurisdiction in a particular state.
However, it is unresolved whether a manufacturer is subject to personal jurisdiction in a particular state when it
places its product in the stream of commerce and it knows (or hopes) that its product will end up in a
particular state. When dealing with a "stream of commerce" case, the clear case for personal jurisdiction occurs
when the manufacturer places its product in the stream of commerce and does something else (e.g., advertises
or maintains a sales force within a state, changes its product to conform to state regulation, etc.) to serve the
market in that state.
In the instant case, the facts state Wamem sells its product to retailers throughout the United States, and that
the plaintiff bought Wamem's product in State Y and brought it back to his home in State X. No other
facts detailing Wamem's connection to State X are given, nor is it clear how close State Y is to State X.
Thus, it is unclear whether Warnem would be subject to personal jurisdiction in State X. If State Y is close
to State X, it could be that Warnem could foresee that its product could wind up in State X; but, as
discussed above, it is unclear whether this-the knowledge or hope that its product would end up in a particular
state-would be enough for personal jurisdiction over Wamem in State X. It may be that
CIVIL PROCEDURE EXAM ANSWERS ® barbrr

Warnem has committed other acts that show its intent to serve the market in State X through its sales to
retailers in State Y, especially if the two states are close geographically, but the facts of the question are
silent in this regard.
It could also turn out that Warnem sells to other retailers in State X, but that plaintiff chose to buy the
product in State Y. Although this would show Warnem's intent to serve the market in State Y, it would not
solve the jurisdiction based on a stream of commerce theory, because the injury suffered by plaintiff would
not be related to Warnem's activities within State Y. In other words, there would be no specific jurisdiction,
and we would have to show that Warnem is subject to general jurisdiction in State
X. It would have to be shown that selling to retailers in State X makes Warnem "essentially at home" in
State X. This is an unlikely possibility.
For these reasons, I could move the court to dismiss the claim based on the lack of personal jurisdic tion
over Warnem. Although not entirely clear or certain, I would probably prevail on this motion.

ANSWER TO EXAM QUESTION NO. 2


Bookkeeper's and Company's motions to dismiss should be denied. The issues are whether
Bookkeeper and Company have sufficient minimum contacts with State Z, and whether service by mail,
return receipt requested, is a constitutionally permissible method of service of process.
In order for the court to exercise personal jurisdiction over a defendant, personal jurisdiction must be
authorized by state statute, and the exercise of personal jurisdiction must be constitutional. For the
exercise of personal jurisdiction to be constitutional, the defendant must have sufficient purposeful
contacts with the jurisdiction such that the exercise of personal jurisdiction over the defendant would be
fair and reasonable, and he must be provided with sufficient notice of the action.

Statutorily Authorized
The question does not set forth the provisions of the State Z statute, and mere notice to a nonresi dent
outside the state is not sufficient to establish personal jurisdiction. Assuming, however, that State Z has
enacted a long arm statute, the court could assert jurisdiction over nonresident individuals and
corporations as to causes of action arising out of, among other things, the transaction of business or the
commission of a tortious act within the state. Here, since Investor is bringing suit on a tort theory,
apparently the "tortious act" part of the statute is being relied on to establish jurisdiction.
Company: Company's fraudulent concealment of its debts was a misrepresentation committed in
each state to which it mailed the report and offered its securities for sale. It had a duty to disclose its debts
to prospective purchasers, not merely to Bookkeeper. Thus, since the report was sent to State Z and
Investor purchased shares in reliance thereon, both the wrongful act (failure to disclose) and the injury to
Investor (loss on resale) occurred in State Z, and the statute is applicable.
Bookkeeper: Bookkeeper's negligent failure to discover the concealed debts occurred in State Y, while
Investor's injury occurred in State Z. In this situation-where an act is done outside the state
that causes injury within the state-the cases are in conflict as to whether the long arm statute applies.
Some courts have adopted the "place of effect" theory, holding that a tortious act occurs "within"
the state if injury occurs there and defendant should have known that his acts might take effect there.
Other courts insist that the defendant must be shown to have acted while (physically) within the state. If
State Z follows the "place of effect" approach, Bookkeeper comes within the statute because he was told
that his financial statement was for nationwide issue and thus should have known that his act
would affect investors in State Z. However, if State Z follows the "place of act" approach, the statute
would not apply to Bookkeeper-since he "acted" in State Y-and, hence, there would be no basis for
asserting jurisdiction over him.

CIVIL PROCEDURE EXAM ANSWERS 3.

Service of Process
The defendants, Company and Bookkeeper, were served by registered mail in their home states in accordance
with a statute of State Z. Service by registered mail, return receipt requested, is reasonably calculated to provide the
defendant with actual notice of the action. As a result, service by mail, return receipt requested, is constitutional, and
thus adequate.
Constitutionality
Company: The Court in International Shoe v. Washington set forth the "minimum contacts" approach for
testing whether a state can constitutionally assert personal jurisdiction over a nonresident: Itmust appear that there
are sufficient minimum contacts between the defendant and the forum state such that the "maintenance of suit
locally does not offend traditional notions of fair play and substantial justice." The modem interpretation of this test
merely requires a showing that defendant purposefully engaged in forum-related activity and that the quality of
that activity makes it reasonable to expect him to appear and defend.
In this case (as indicated above), Company has committed a tortious act in State Z that injured a local resident.
Moreover, it purposefully entered the State Z marketplace by mailing its financial report and offering its securities
for sale there. These two factors combined establish constitutionally suffi cient contacts for the exercise of
personal jurisdiction over Company. Therefore, Company's motion to dismiss should be denied.
Bookkeeper: Should it be held that Bookkeeper has not committed a tort within the state, the State Z long
arm statute would not apply and the constitutional question need not be reached. However, if it is concluded that
Bookkeeper has committed a tort within the state, the argument made for jurisdiction over Company applies to
him as well. It is immaterial that Bookkeeper is not a corporation because
the minimum contacts test applies to any nonresident defendant. Bookkeeper's knowledge that his report would
be circulated nationwide and his probable expectation of a fee for a job that would have consequences in State Z
establish the type of purposeful forum-related activity required under the Due Process Clause. Hence, a finding of
an adequate basis for personal jurisdiction would be proper, and Bookkeeper's motion to dismiss should be
denied.

ANSWER TO EXAM QUESTION NO. 3


(1) Removal to Federal Court and Motion to Remand
The remand back to state court was the correct ruling. At issue is whether removal on the basis of diversity
citizenship was permissible. Cases within a state court's jurisdiction that could have been brought originally in
federal court can generally be removed to federal court at defendant's request. However, a case cannot be
removed on the basis of diversity if one of the defendants is a resident of the forum state.
The only possible basis for federal jurisdiction over the case is diversity ofcitizenship (no federal question
was presented). Diversity of citizenship jurisdiction requires an amount in controversy of more than $75,000,
and every plaintiff must be of diverse state citizenship from each defendant.
The citizenship of an individual is determined by his domicile (i.e., his permanent home to which he intends to
return) . A corporation is a citizen of every state in which it is incorporated and the one state in which it has its
principal place of business, defined as the place from which the corporation's officers direct and control the
corporation's activities. Here, Ped is a citizen of State A, and Health is a citizen not only of State H (state of
incorporation), but also of State A, where it has its company headquarters, from which its officers presumably
direct and control its activities. Therefore, diversity did not exist, and there was no removal jurisdiction.
Moreover, even if diversity had existed, removal is not permitted ifany defendant is a citizen of the state
where the action was filed. Because Ped brought suit in State A, of which Health is a citizen, removal should
have been prohibited.
4. CIVIL PROCEDURE EXAM ANSWERS ® barbrr

When an action is improperly removed, the appropriate remedy is a remand back to the state court.
Therefore, the federal court ruled correctly.
(2) Joinder of Defendants
The court properly overruled Health's motion to dismiss as to the joinder of the defendants. At issue is
whether a plaintiff may properly assert claims against different defendants in the same action.
Today, federal courts have liberal joinder rules. Defendants can be joined where the claim against them
arises from the same transaction or series of transactions and involves at least one common question of law
or fact.
Both criteria are met here: There are common questions of negligence law, and aggravation of an injury
during treatment is considered part of the same series of events as the original injury.
As a result, Health's motion to dismiss on the grounds of misjoinder of parties was therefore appro priately
overruled.
Joinder of Claims
The court also properly overruled Health's demurrer as to the joinder of the claims. At issue is whether a
plaintiff may properly assert different claims against different defendants in the same action.
Federal andmost state courts allow for the liberal joinder of claims. The policy permits the adjudi cation
of all claims arising out of a single transaction. (Furthermore, when one claim is properly joined against all
defendants, the plaintiff may join any claim he has against any defendant, subject to the court's power to
order severance. It is essential only that at least one of the claims arose out of a trans action in which all
were involved.)
Here, as stated above, the claims are related and thus may be joined together. (Driver's liability and
Health's liability each will be affected by a determination of the other's liability.)
As a result, the court did not err in overruling the demurrer as to joinder of claims.
(3) Ped's Requests for Admissions
The objections to the request for admissions were properly sustained. At issue is whether a party, by use
of a request for admission, can request another party to admit or deny a bald legal conclusion.
Any party may serve on any other party a written request to admit the truth of any relevant matter, the
purpose being to narrow the issues. Generally, the answering party cannot object solely because the
requested admission calls for a legal conclusion, as long as the legal conclusion relates to the facts of the
case. Thus, for example, it is proper to ask the adverse party to admit that he was "negligent."
Arguably, however, in this case the request to admit "liability" was improper because it calls for a
final legal conclusion. On the other hand, it seems logical that the form of the request should not
govern, and since requesting an admission of liability really only calls for an admission of negligence,
objections to the requests should not have been sustained. Nonetheless, this court might well be sitting in a
jurisdiction that does not permit requests as to ultimate issues. If this is the case, Ped's requests were
inappropriate and discovery was properly withheld.
(4) Motion for New Trial
The court was correct in refusing to consider the affidavits. At issue is whether jurors may give
evidence ofjuror misconduct occurring during deliberations.
Misconduct in jury deliberations is ground for a new trial in whole or in part. One form of such
misconduct is the use of "quotient verdicts""-that is, where the jurors agree beforehand that the measure
of recovery will be arrived at by totaling the amount favored by each juror and dividing that total by 12
(number ofjurors). A quotient, however, can be used as a starting point for discussion, and here the jurors
agreed to the quotient after it was derived; therefore, the verdict is probably proper.
Even if the verdict were improper, however, it is not clear that it could be impeached by use of the
juror's own affidavits. Many states-as well as the Federal Rules-do not permit juror testimony as to
matters occurring during deliberation, so that the quotient verdict apparently could not be attacked by the
affidavits used here.
CIVIL PROCEDURE EXAM ANSWERS 5.

Thus, the court's refusal to consider Ped's evidence in support ofhis motion was proper, so that there
could be no basis for ordering a new trial.

ANSWER TO EXAM QUESTION NO. 4


(1) Peter v. Valeo
Joinder of Defendants: Valeo may not compel Peter to join Mity as a defendant. Whether Valeo has
ground for challenging Peter's failure to join Mity turns on compulsory joinder rules.
To determine whether a party must be (compulsorily) joined as a defendant, a three-step process is used,
first asking whether the proposed defendant should bejoined, followed by determining whether the
proposed defendant can bejoined (i.e., determining whether the proposed defendant is subject
to personal jurisdiction in the chosen forum, and whether his presence would affect subject matter
jurisdiction or venue), concluding with a determination that, if the party cannot be joined, whether the action
should proceed or be dismissed.
A party should bejoined if (i) complete relief cannot be accorded among the other parties to the
lawsuit without the proposed defendant being made a party; or (ii) a decision without joining the
proposed defendant would impair or impede that defendant's ability to protect that interest, or his
absence would leave other parties subject to a substantial risk of incurring multiple or inconsistent
obligations.
Here, it does not appear that Mity's joinder is required. Its interests are not in danger because a
determination in the present suit will not bind Mity in the future. If the state follows jointand several
liability rules, Peter may recover any award in the entirety from either Valeo or Mity. If the state appor tions
liability among joint tortfeasors, liability is typically apportioned among tortfeasors even if one is absent
from the trial. The absent defendant would not be bound by such a determination, as res judicata or
collateral estoppel would not apply because, at a minimum, the absent party would not have had an
opportunity to be heard in court on the matter. Thus, the "should be joined" prong of the compulsory joinder
analysis is not satisfied, as complete relief can be accorded among the other parties and no party would be
subject to inconsistent or multiple liabilities. (Although Peter might be able to collect more money by
joining both defendants, it is his tactical decision to proceed without one of the defen dants unless it
prejudices another party or the absentee, and Valeo can implead Mity (see below).)
Hence, joinder of Mity is not compulsory and an objection by Valeo should be overruled.
Impleader: Valeo should file an impleader action against Mity. At issue is how a defendant may bring
another party in who might be liable to him for some or all of the plaintiff's claim. When a stranger to a
pending lawsuit may be liable for all or part of plaintiff's claim against defendant (i.e., defendant has a
potential right to indemnification), that third party can be brought into the suit by impleader. A third-party
claim is deemed supplemental to the main claim and has no effect on juris diction.
In the instant case, if the state follows joint and several liability, and Valeo faces the possibility of
paying for the entire award, impleader would be appropriate. If the state apportions liability, Valeo would
not need to implead Mity because Valeo would not have to pay more than its apportioned liability.
Here, since it is likely that the underlying cause of the explosion was Mity's fault (allegedly defec tive
"collar"), Valeo could have a valid claim for indemnification, and might be able to implead Mity.
Moreover, the fact that both Valeo and Mity are citizens of A, and hence not diverse, does not destroy the
court's jurisdiction. Valeo can (and should) file an impleader claim against Mity (depending on how the
state apportions tort liability).
(2) Peter and Quincy v. Valeo and Mity
Joinder of Parties: Peter and Quincy would be allowed to join together as plaintiffs. At issue is
6. CIVIL PROCEDURE EXAM ANSWERS ® barbrr

whether two plaintiffs may join together in a single action for separate injuries sustained in the same
accident. The Federal Rules allow liberal joinder of plaintiffs and defendants, provided that any right by
or against them arises from the same transaction or series of transactions, and that there is at least one
question of law or fact common to all.
In this case there are many common questions (e.g., both plaintiffs injured by same defective pressure
valve), and the "same transaction" requirement is met by the fact that one single incident caused the harm.
Hence, joinder on both sides is warranted.
Joinder of Claims: Peter and Quincy would also be allowed to join their claims together. Again,
the issue is whether two plaintiffs may join together in a single action for separate injuries sustained
in the same accident. The policy of the Federal Rules is to permit adjudication of all claims between the
parties and all claims arising out of a single transaction. Here, Peter and Quincy were both injured when
the collar manufactured by Mity, and incorporated into a pressure valve by Valeo, exploded and injured
both Peter and Quincy. As a result, a single trial is warranted, and joinder of the claims was proper.
Jurisdiction over Defendants : It is unclear whether a federal court in State C would have personal
jurisdiction over Valeo and Mity. At issue is whether placing a product in the stream of commerce
subjects the manufacturer to personal jurisdiction wherever that product winds up. Assuming State C
has a long arm statute that would give a state court jurisdiction over Valeo and Mity, the federal court
will follow it, and the only problem is whether application of the statute would violate due
process. Under the standard established in International Shoe v. Washington, it need only appear that
there are sufficient, purposeful minimum contacts between the defendants and the forum state such that the
maintenance of suit locally does "not offend traditional notions of fair play and substantial justice."
This case also presents a "stream of commerce" scenario. Generally speaking, when a manufac turer
merely places its product in the stream of commerce, and does not do anything further, it will not be
subject to personal jurisdiction in a particular forum. However, it is unresolved whether a manufacturer is
subject to personal jurisdiction in a particular state when it places its product in the stream of commerce
and it knows (or hopes) that its product will end up in a particular forum. When dealing with a "stream
of commerce" case, the clear case for personal jurisdiction occurs when the
manufacturer places its product in the stream of commerce and does something else (e.g., advertises
or maintains a sales force within a state, changes its product to conform to state regulation, etc.) to
serve the market in that state.
In the instant case, the facts state both Valeo and Mity have their principal places of business in State
A. Presumably, that is where Mity manufactures the collar and where Valeo incorporates it into the
pressure valve. It is unclear how the collar managed to get into State C. (Furthermore, personal
jurisdiction over each company must be considered separately. They are treated here together because
there are no facts indicating how the valve assembly got into State C-the facts are equally silent as to
both.) If Valeo and/or Mity did nothing purposeful to have its valve wind up in State C, personal
jurisdiction over Valeo and/or Mity would not be permissible in State C. If Valeo and/or Mity only
knew or hoped that its valve would wind up in State C, it is unclear whether personal jurisdiction could be
based on this knowledge or hope. If Valeo and/or Mity did something else to purposefully serve the State
C market, there would unquestionably be personal jurisdiction over Valeo and/or Mity. But again, the facts
are silent, so it is unclear whether there will be personal jurisdiction in State C over either Valeo or Mity.
Jurisdiction over Subject Matter: There is no federal subject matter jurisdiction over this case.
At issue is whether the citizenship of an individual is determined when the cause of action accrues or
when suit is filed. Since no federal question is presented, jurisdiction can only be based on diversity of
citizenship-which requires that more than $75,000 be in controversy and that no plaintiff be a citizen of the
same state as any defendant. The citizenship of an individual is his permanent domicile (i.e., the permanent
home to which he intends to return) . A corporation is a citizen of every state in which it is
CIVIL PROCEDURE EXAM ANSWERS 7.

incorporated and the one state in which it has its principal place of business. Citizenship is determined
when suit is filed, not when the cause of action accrues.
Here, there is not complete diversity between the parties. Since citizenship for diversity purposes is
determined when the suit isfiled, Quincy and Valeo are both citizens of B (state of Valeo's incorpora tion
and Quincy's residence when suit commenced). Therefore, the court has no power to decide the case and
must dismiss for lack of subject matter jurisdiction.
(3) Peter v. Valeo and Mity
What Law Applies? A federal court sitting in the district court in State A would apply federal law as
to the pleading of freedom from contributory negligence, but apply state law as to the burden of
persuasion. When determining whether to apply state or federal law, the court must look at whether the
issue involved is substantive or procedural. If Peter alone sues Valeo and Mity, there would be
complete diversity. In diversity cases, a federal court is bound to apply the law of the state in which it is
sitting on all substantive matters [Erie v. Tompkins] ; however, federal procedural rules control.
Questions of pleading (here, whether plaintiff must plead freedom from contributory fault) clearly
involve the mechanics of the federal court system, and as such, the Federal Rules apply.
Burden of persuasion, on the other hand, involves basic policy considerations regarding the substance of
the case, so that the federal court must follow Erie and apply the same law as would the
highest court in State A. Hence, if State A would apply its own law, so would the district court, but if
on conflict of law principles State C law would govern (place of injury), the federal court would
similarly be bound thereby.

You might also like